Download as pdf or txt
Download as pdf or txt
You are on page 1of 190

TM TM TARGET:JEE

TARGET: JEE(Main
(MAIN+ +Advanced)
ADVANCED)2020
2020
ENTHUSIAST & LEADER
Path to success KOTA (RAJASTHAN) ENTHUSIAST & LEADER
COURSE
Path to success KOTA (RAJASTHAN ) COURSE

ADVANCED QUIZ # 1 FUNC TI ONS MATHEMATICS


SINGLE CORRECT CHOICE TYPE
1. Let f(x) = x2 + 4x – 1 and g(x) = |x|. If h(x) = f(g(x)) + 10, then the range of h(x), is
(A) {y : y ³ 10} (B) {y : y ³ 0} (C) {y : y ³ 5} (D) {y : y ³ 9}
2. Which of the following best represent the graph of function y = ||x – 2|–2| ?

(A) x (B) X
O O

y y

(C) x (D) x
O O

ì -4 ü ì4ü 4x
3. Let f : R - í ý ® R - í ý be a function defined as f(x) = . The inverse of f is the map g :
î3þ î3þ 3x + 4

ì4ü ì -4 ü
R - í ý ® R - í ý is given by
î3þ î3 þ

3y 4y 4y 3y
(A) g(y) = (B) g(y) = (C) g(y) = (D) g(y) =
3 - 4y 4 - 3y 3 - 4y 4 - 3y

The range of the function f(x) = log cosec2 7 p (4 - 2 + 1) is equal to


x x
4.
3

(A) (-¥,1] (B) [-1, ¥ ) (C) (-¥, - 1] (D) [1, ¥)

5. Let ‘n’ be the number of elements in the Domain set of the function f (x) = ln x2 +4 x
C2 x 2 + 3 and ‘Y’ be the

global maximum value of f (x), then [ n + [Y]] is (where [.] = Greatest Integer function)
(A) 4 (B) 5 (C) 6 (D) 7
6. The function f : R ® R is defined by f(x) = cos2 x + sin4 x for x Î R, then the range of f(x) is
æ3 ù é3 ö é3 ù æ3 ö
(A) ç 4 , 1ú (B) ê 4 , 1 ÷ (C) ê 4 , 1ú (D) ç 4 , 1 ÷
è û ë ø ë û è ø

7. If f(x) = [cos (x + 6) cos x – cos2(x + 3)] where [.] denote greatest integer function, then a solution of
equation f(x) = x3 is
(A) –1 (B) 0 (C) 1 (D) 2

MATHEMATICS /AQ # 1 E-1


TM TARGET: JEE (MAIN + ADVANCED) 2020
ENTHUSIAST & LEADER
Path to success KOTA (RAJASTHAN)
COURSE
8. The domain of f(x) = ln (ax3 + (a + b)x2 + (b + c)x + c), where a>0, b2 – 4ac = 0, is

ì bü ì bü ì bü
(A) (–1, ¥) – í - ý (B) (1, ¥) – í - ý (C) (–1, 1) – í - ý (D) None of these
î 2a þ î 2a þ î 2a þ
PASSAGE TYPE QUESTION
PARAGRAPH FOR QUESTION NO. 9 TO 10

ì 1, -2 £ x £ -1
ï
Consider f be a real-valued function defined on interval [–2, 2] as f (x) = í x + 2, -1 < x < 1
ï 4 - x, 1 £ x £ 2
î

9. The range of f(x) is


(A) [1, 2] (B) [1, 3] (C) (1, 3) (D) [–1, 4]

1
10. Number of solutions of the equation {f(x)} = , is
2
[Note : {x} denotes fractional part of x.]
(A) 1 (B) 2 (C) 3 (D) 0
PARAGRAPH FOR QUESTION NO. 11 TO 14
Let f(x) = x – 2x – 1 "x Î R . Let f : (–¥, a] ® [b, ¥) where ‘a’ is the largest real number for which f(x) is
2

bijective
11. The value of (a + b) is equal to
(A) –2 (B) –1 (C) 0 (D) 1
12. Let f : R ® R, g(x) = f(x) + 3x – 1, then the least value of function
y = g(|x|) is
-9 -5
(A) (B) (C) –2 (D) –1
4 4
13. Let f : [a, ¥) ® [b, ¥) then f–1(x) is given by
(A) 1 + x + 2 (B) 1 - x + 3 (C) 1 - x + 2 (D) 1 + x + 3
14. Let f : R ® R, then range of values of k for which equation f(|x|) = k has 4 distinct real roots is
(A) (–2, –1) (B) (–2, 0) (C) (–1, 0) (D) (0, 1)
MULTIPLE CORRECT CHOICE TYPE
æ x 2 + 2x -3 ö
log[2tan p x] ç 2
15. The domain of definition of the function, f (x) = [2 tan px] ç 4x - 4 x -3 ÷÷ where [ ] denotes the greatest
è ø

é 1 1ö
integer function is given by the interval ê n + , n + ÷ where n Î I then n can be equal to
ë 4 2ø
(A) – 5 (B) – l (C) 0 (D) 1

MATHEMATICS /AQ # 1 E-2


TM TARGET: JEE (MAIN + ADVANCED) 2020
ENTHUSIAST & LEADER
Path to success KOTA (RAJASTHAN)
COURSE
16. Let f : A ® B and g : B ® C be two functions and gof : A ® C is defined. Then which of the following
statement(s) is/are incorrect ?
(A) If gof is into then g must be into
(B) If gof is onto then f must be onto
(C) If gof is one-one then f must be one-one
(D) If gof is bijective then both f and g must be bijective
17. Which of the following statement(s) is/are correct ?

( )
(A) The graph of the function f(x) = sgn x + x + 1 is symmetric with respect to origin, (Where sgn x
3 6

denotes signum function of x)

-1 æ 1 ö ép p ö
(B) The range of the function f(x) = cos ç x - x ÷ is ê , ÷ .
èe +e ø ë3 2 ø

(C) The domain of the definition of function f (x) = 1 - x + x - 2 is [1, 2]


(D) The value of 2cos–1(cos 7) – sin–1(sin 11) is equal to 3
18. Consider the function f(x) = sgnx and g(x) = x(1 – x2) then which of the following hold (s) true
(A) (fog)(x) is neither odd nor even
(B) (gof) (x) is even as wellas odd
(C) (fog)(x) is neither continuous nor differentiable for some x in (–¥, ¥)
(D) (gof)(x) is continuous and differential for every x on (–¥, ¥)
19. Let f(x) = [x]2 + [x + 1] – 3, where [x] is greatest integer less than or equal to x, then
(A) f(x) is a many-one (B) f(x) = 0 for infinite values of x
(C) f(x) = 0 for only two real values (D) f(x) is one -one
INTEGER ANSWER TYPE
20. If f : R ® R be an injective mapping and p, q, r are non-zero distinct real quantities satisfying

æpö æ p-qö æqö ærö


fç ÷=fç ÷ and f ç ÷ = f ç ÷ . If the graph of g(x) = px2 + qx + r passes through M(1, 6) then find
èrø è q-r ø èrø èpø
the value of q.

( 2x ) ! e log p ( 2x - 1)
2

21. Let f : X ® Y be an onto function such that y = f(x) = exp { (


ln 3cos
-1
x2
)} +
x +1
-
[ 3x ]
,

Find sum of number of all possible elements of sets X and Y.


where x Î X, y Î Y and [*] denotes the greatest integer function.

22. Find the number of solution of the equation x - 1 = 2 [ x ] - 3{x} for x


(where [x] & {x} denotes integral and fractional part of x)

MATHEMATICS /AQ # 1 E-3


TM TARGET: JEE (MAIN + ADVANCED) 2020
ENTHUSIAST & LEADER
Path to success KOTA (RAJASTHAN)
COURSE
MATCH THE COLUMN :
23. Match the entries of Column-I with one or more than one entries of column-II. Note that [x], {x} and sgn
x denote largest integer less than or equal to x, fractional part of x and signum function of x respectively.
Column-I Column-II
(A) Let f : [–1, 1] ® R be defined by f (x) = 5 x + sin -1 x (p) Odd.
then f(x) is
(B) Let f : R ® {-1,0,1} be defined by (q) Even

æ 1- | x | ö
f (x) = sgn ç ÷ then f(x) is
è 1+ | x | ø

(C) Let f : [–4, 2] ® [0, 3] be defined by f (x) = 8 - 2x - x 2 (r) Onto


then f(x) is
(D) Let f : (-¥,0] ® [0, ¥) be defined by (s) One-one

2-[ x ]
f (x) = - 2|x| then f(x) is ([x] denotes greatest integer (t) Many-One
2{x}
function and {x} denotes fractional part function)

ANSWER KEY
1. (D) 2. (A) 3. (B) 4. (B) 5. (A) 6. (C) 7. (A)
8. (A) 9. (B) 10. (C) 11. (B) 12. (C) 13. (A) 14. (A)
15. (A, C) 16. (A, B, D) 17. (B, D) 18. (B, C, D) 19. (A, B)
20. [0008] 21. [0002] 22. [0002] 23. [A–P, S; B–Q,R,T; C–R,T; D–T]

MATHEMATICS /AQ # 1 E-4


TM TM TARGET:JEE
TARGET: JEE(Main
(MAIN++Advanced)
ADVANCED)2020
2020
ENTHUSIAST & LEADER
Path to success KOTA (RAJASTHAN) ENTHUSIAST & LEADER
COURSE
Path to success KOTA (RAJASTHAN ) COURSE

ADVANCED QUIZ #2 INVERSE TRIGONOMETRY FUNCTIONS MATHEMATICS

SINGLE CORRECT CHOICE TYPE


1. If 0 < cos -1 x < 1 and 1 + sin(cos -1 x) + sin 2 (cos -1 x) + sin 3 (cos -1 x) + ....¥ = 2 , then x equals

1 1 3 1
(A) (B) (C) (D)
2 2 2 2 3

-1 x y
2. If cos - sin -1 = q (a, b ¹ 0) . Then the maximum value of b 2 x 2 + a 2 y 2 + 2ab xy sin q equals
a b

(A) ab (B) (a + b) 2 (C) 2(a + b) 2 (D) a 2 b 2


3. If 3 £ a < 4 then the value of sin–1 (sin[a]) + tan–1(tan[a]) + sec–1(sec[a]), where [x] denotes greatest
integer function less than or equal to x, is equal to
(A) 3 (B) 2p – 9 (C) 2p – 3 (D) 9 – 2p
4. The smallest positive integral value of n for which (n – 2)x2 + 8x + n + 4 > sin–1(sin 12) + cos–1(cos 12)
"x Î R , is
(A) 4 (B) 5 (C) 6 (D) 7

-1 1 1 1 1 p
5. For n Î N , If tan + tan -1 + tan -1 + tan -1 = then n is equal to
3 4 5 n 4
(A) 43 (B) 47 (C) 49 (D) 51

6. ( (
For function f (x) = ln sin log p ( cos x )
-1 -1
))
æ pù
(A) domain is (–1, cos 1] (B) Range is ç -¥, ln ú
è 2û
(C) domain [–1, 0) (D) range is (–¥, 0]
7. The number of solution of the equation mcos (1 – x) = ncos–1x, m > 0, n < 0 is
–1

(A) 0 (B) 1 (C) 2 (D) 3

p
8. Number of integral solutions of cos–1 |x| + tan–1|x| = is /are
2
(A) 0 (B) 1 (C) 2 (D) Infinite

9. The number of real solutions of the equation tan -1 x 2 - 3x + 2 + sin -1 4 - x 2 - 2x = p is


(A) 2 (B) 3 (C) Infinite (D) 0

10. Find the range of f(x) = (sin–1x) sinx is

é pù é p ù é p pù
(A) ê0, ú (B) ê0, sin1ú (C) ê - , ú (D) None of these
ë 2û ë 2 û ë 2 2û

MATHEMATICS /AQ # 2 E-5


TM TARGET: JEE (MAIN + ADVANCED) 2020
ENTHUSIAST & LEADER
Path to success KOTA (RAJASTHAN)
COURSE
PASSAGE TYPE QUESTION
PARAGRAPH FOR QUESTION NO. 11 TO 13

æ pö
For x Î ç 0, ÷ ,
è 4ø
2n
( )
2n
( )
2n
( )
Let Sn = å sin sin -1 x 3r -2 , C n = å cos cos -1 x 3r -1 and Tn = å tan tan -1 x 3r where n Î N and n ³
r =1 r =1
r =1

3.
11. The correct order of Sn , C n and Tn is given by

(A) Sn > Tn > C n (B) Sn > C n > Tn (C) Sn < Tn < C n (D) None of these

12. The value of nlim (Sn + C n + Tn ) is equal to


®¥

1 x 1 x
(A) (B) (C) (D)
1- x 1- x 1- x 1+ x
13. The value of ‘x’ for which Sn = C n + Tn , is

p p p p
(A) sin (B) 2 sin (C) 2 sin (D) sin
5 5 10 10
MULTIPLE CORRECT CHOICE TYPE

-1 æ 5 ö -1
æ 3ö æ
-1 7 2
ö -1 æ 3 ö
14. Let a = 3cos ç ÷ + 3tan çç 2 ÷÷ and b = 4sin çç 10 ÷÷ - 4 tan ç 4 ÷ then which of the following
è 28 ø è ø è ø è ø
hold (s) good ?
(A) a < p but b > p (B) a > p but b < p
(C) Both a and b are equal (D) cos (a + b) = 1
15. Which of the following is (are) correct ?

æ -1 10 1ö
(A) The value of sin çç sec + cot -1 ÷÷ is a rational number
è 3 3ø

-1 -1 p
(B) Number of solution of the equation cos x - sin (- x) = , is one
3

-1 -1 p
(C) Number of solution of the equation cos x - sin (- x) = , is infinite
3
(D) Let g : R ® R be defined as g(x) = sgn(4e–x) then g–1(x) is not defined [ sgn x denote the signum function
of x)

é p 2p ù
16. Let f : ê - , ú ® [0, 4] be a function defined as f(x) = 3 sin x - cos x + 2 then f–1(x) is given by
ë 3 3 û

-1 æ x - 2 ö p -1 æ x - 2 ö p 2p æ x-2ö p -1 æ x - 2 ö
(A) sin ç ÷ - (B) sin ç ÷ + (C) - cos-1 ç ÷ (D) - cos ç ÷
è 2 ø 6 è 2 ø 6 3 è 2 ø 3 è 2 ø
MATHEMATICS /AQ # 2 E-6
TM TARGET: JEE (MAIN + ADVANCED) 2020
ENTHUSIAST & LEADER
Path to success KOTA (RAJASTHAN)
COURSE
INTEGER ANSWER TYPE

-1 3p
17. The number of solutions of the equation 2 sin x 2 + x + 1 + cos -1 x 2 + x = is
2
MATCH THE COLUMN :
18. Number of real solutions of
Column - I Column - II

(A) tan ( p / 4 + (1/ 2 ) cos -1 x ) + tan ( p / 4 - (1/ 2 ) cos -1 x ) = 1 (p) 0

1 1
(B) tan
-1
+ tan -1 = tan -1 ( 2 / x 2 ) (q) 2
2x + 1 4x + 1

(C) tan -1 ( x + ( 2 / x ) ) - tan -1 ( 4 / x ) - tan -1 ( x - ( 2 / x ) ) = 0 (r) 3

(D) tan -1 (1 - x ) + tan -1 (1 + x ) = tan -1 2x (s) 1

19. Column - I Column - II


(A) If x = cosec2 (cot–1 3) – sec2 (tan –1 2), then (p) x = 2
(B) If tan–1 x + tan–1 (1/y) = tan–1 3 and y2 + y – 56 = 0 (q) x = 5
(C) If cos–1 x = tan–1 y and y2 = 3 (r) x = 1/2

(D) If sin -1 ( tan p / 4 ) - sin -1 3 / y - p / 6 = 0 and x2 = y (s) x = – 1/2

20. Column-I contain four functions and Column-II contain their properties. Match every entry of Column-I
with one or more entries of Column-II
Column-I Column-II

(A) f ( x) = sin -1 ( sin x ) + cos -1 ( cos x ) (p) range is [0, p ]

-1 -1
(B) g (x) = sin | x | +2 tan | x | (q) is increasing " x Î (0,1)

-1 æ 2 x ö
(C) h (x) = 2sin ç 2 ÷
, x Î [ 0,1] (r) fundamental period is 2p
è 1+ x ø

(D) k (x) = cot (cot–1 x) (s) is decreasing " x Î (0,1)

MATHEMATICS /AQ # 2 E-7


TM TARGET: JEE (MAIN + ADVANCED) 2020
ENTHUSIAST & LEADER
Path to success KOTA (RAJASTHAN)
COURSE
21. Match the entries of Column-I with one or more than one entries of column-II. Note that [x], {x} and sgn
x denote largest integer less than or equal to x, fractional part of x and signum function of x respectively.
Column-I Column-II

(A) Let f : [–1, 1] ® R be defined by f (x) = 5 x + sin -1 x (p) Odd.

then f(x) is

(B) Let f : R ® {-1,0,1} be defined by (q) Even

æ 1- | x | ö
f (x) = sgn ç ÷ then f(x) is
è 1+ | x | ø

(C) Let f : [–4, 2] ® [0, 3] be defined by f (x) = 8 - 2x - x 2 (r) Onto

then f(x) is

(D) Let f : (-¥,0] ® [0, ¥) be defined by (s) One-one

2-[ x ]
f (x) = {x}
- 2|x| then f(x) is ([x] denotes greatest integer (t) Many-One
2
function and {x} denotes fractional part function)

ANSWER KEY
1. (C) 2. (D) 3. (A) 4. (B) 5. (B) 6. (B) 7. (A)
8. (B) 9. (D) 10. (B) 11. (B) 12. (B) 13. (C) 14. (CD)
15. (AD) 16. (BC) 17. [0002] 18. (A – p; B– s; C– q; D – s)
19. (A –q; B – pq; C – r ; D – p) 20. [A –p, q, r ; B–p, q; C–p, q; D–q]

21. [A –P, S; B–Q,R,T; C–R,T; D–T ]

MATHEMATICS /AQ # 2 E-8


TM TM TARGET:JEE
TARGET: JEE(Main
(MAIN++Advanced)
ADVANCED)2020
2020
ENTHUSIAST & LEADER
Path to success KOTA (RAJASTHAN ) ENTHUSIAST & LEADER
COURSE
Path to success KOTA (RAJASTHAN) COURSE

ADVANCED QUIZ # 3 LIMITS, CONTINUITY, DIFFERENTIABILITY & MOD MATHEMATICS

SINGLE CORRECT CHOICE TYPE

lim
( 3 ( n + 1) )!
1. equals
( n + 1) ( 3n )!
n ®¥ 3

(A) 3 (B) 9 (C) 27 (D) 36

é ( e 2 x + 1) - ( x + 1) ( e x + e - x )
2. Let f (x) = ê x ( e x - 1)
if x ¹ 0 if f (x) is continuous at x = 0 then k is equal to
ê
ê
ëk if x=0

(A) 1/2 (B) 1 (C) 3/2 (D) 2

a n sin bx - b n sin ax
3. Consider the function f (x) = if x ¹ 0, a, b > 0 and a ¹ b then at x = 0
tan bx - tan ax
(A) f has non removable discontinuity of finite type
(B) f has non removable discontinuity of infinite type
(C) f has non removable discontinuity of oscillatory type
(D) f has removable discontinuity

d2y
4. The value of 2 if x 2 - y 4 = 6 , is
dx

3 y y - 3x 2 y 4 - 3x 2
(A) - (B) (C) (D)
16 y 7 3x 2 y4 4 y7

1- x
x 2 x3 æ -7 ö
5. If the function f (x) = -4e 2 +1+ x + + and g(x) = f -1 (x) , then the value of g¢ ç ÷ equals
2 3 è 6 ø

1 1 6 6
(A) (B) - (C) (D) -
5 5 7 7
6. Let g(x) = f (x) sin x, where f(x) is a twice differentiable function on (–¥, ¥) such that f¢ (–p) = 1. The value
of g¢¢(–p) equals
(A) 1 (B) 2 (C) –2 (D) 0
7. Let f (x) be a real valued function such that f (a) = 0. If g(x) = (x – a) f(x) is continuous but not differentiable
at x = a and h (x) = (x – a)2 f(x) is continuous and differentiable at x = a. Then f (x)
(A) must be continuous and differentiable at x = a
(B) must be continuous but not differentiable at x = a
(C) may or may not be continuous at x = a
(D) Must be discontinuous at x = a

MATHEMATICS /AQ # 3 E-9


TM TARGET: JEE (MAIN + ADVANCED) 2020
ENTHUSIAST & LEADER
Path to success KOTA (RAJASTHAN )
COURSE

The derivative of y = ( sin x + cos x )


x
8. =

é log e y ù é log e y ù
(A) y ê + x (sec 2x - tan 2x) ú (B) y ê + x (sec 2x - tan 2x) ú
ë2 x û ë 2x û

é log e y ù é log e y ù
(C) y ê + x (sec 2x + tan 2x) ú (D) y ê + x (tan 2x - sec 2x) ú
ë2 x û ë 2x û

ì -1, - 1 £ x £ 1
9. The function f(x) = g(|x|) + |g(x)| where g(x) = í is
î x - 2, 1 < x £ 3
(A) continuous, but not differentiable at x = 2 (B) is continuous and differentiable at x = 2
(C) is neither continuous nor differentiable at x = 2 (D) has a removable discontinuity at x = 2
1 1
10. If f ( x ) = ( x - 1)( x - 2 ) and g ( x) = 2 , then points of discontinuity of f (g(x)) are
x

ì 1 ü ì -1 1 ü ì 1 ü
(A) í-1, 0,1, ý (B) í , -1, 0,1, ý (C) {0, 1} (D) í0,1, ý
î 2þ î 2 2þ î 2þ

x3
cos 2 x - cos x - e x cos x + e x -
11. The integer n for which the lim 2 is a finite non-zero number is
x® 0 xn
(A) 2 (B) 3 (C) 4 (D) 5

é ù
ê n ú
12. The value of ê lim 2 å 1
ú , where [.] represents greatest integer function, is -
ê x®¥ n r =1 1 + cos æ 2r ö ú
êë ç ÷ú
è n øû

(A) 0 (B) 1 (C) 2 (D) 3


cos 4x cos x - cos 6x cos 9x
13. lim is equal to
x ®0 ln 2 (1 + sin 4x)

25 25 25 25
(A) (B) (C) (D) –
4 8 2 4

x + sin x
14. The value of xlim is equal to
®¥ (x - cos x)1/ 4 (x + sin 3x)1/ 4
(A) 0 (B) 1 (C) –1 (D) –2

1ì 7 ü
15. If f(x) = íf (x + 1) + ý and f(x) > 0 for all x Î R then xLt f (x) is
5î f (x + 2) þ ®¥

7 2
(A) (B) (C) 2 (D) ¥
2 7

MATHEMATICS /AQ # 3 E-10


TM TARGET: JEE (MAIN + ADVANCED) 2020
ENTHUSIAST & LEADER
Path to success KOTA (RAJASTHAN )
COURSE

2 x + 3 3 x + 4 4 x + ........ + n n x
16. lim is equal to
x ®¥ (2x - 3) + 3 (2x - 3) + ......... + n (2x - 3)

(A) 1 (B) ¥ (C) Ö2 (D) None of these

(x + y)sec(x + y) - x sec x
17. lim is equal to
y ®0 y
(A) sec x(x tan x + 1) (B) x tan x + sec x (C) x sec x + tan x (D) None of these

ì n æ 1 ö
ï x sin ç 2 ÷ , x ¹ 0
18. If f(x) = í èx ø , (n Î I), then
ï0, x =0
î

(A) lim
x ®0
f (x) exists for n > 1 (B) lim
x ®0
f (x) exists for n < 0

(C) lim
x ®0
f (x) does not exist for any value of n (D) lim
x ®0
f (x) cannot be determined

19. lim((x + 5) tan -1 (x + 5) - (x + 1) tan -1 (x + 1)) is equal to


x ®¥

(A) p (B) 2p (C) p / 2 (D) None of these

20. Let f : R ® R be given by f(x) = 5x, if x Î Q and f (x) = x 2 + 6 if x Î R - Q , then


(A) f is continuous at x = 2 and x = 3 (B) f is not continuous at x = 2 and x = 3
(C) f is continuous at x = 2 but not at x = 3 (D) f is continuous at x = 3 but not at x = 2

ìmin({x, x 2 }) x ³ 0
21. Let f (x) = í . Then which of the following is not true.
î max{2x, x 2
- 1} x < 0

(A) f(x) is continuous at x = 0 (B) f(x) is differentiable at x = 1


(C) f(x) is not differentiable at exactly three point (D) f(x) is continuous every where

ìg(x), x £ 0
22. Let g(x) be a polynomial of degree one and f(x) be defined by f (x) = í sin x . If f (x) is continuous
î| x | , x > 0
satisfying f '(1) = f ( -1) , then g (x) is
(A) (1 + sin 1) x + 1 (B) (1 – sin 1) x + 1 (C) (1 – sin 1) x – 1 (D) (1 + sin 1) x – 1

1 - cos 2(x - 1)
23. lim
x ®1 x -1
(A) exists and is equal to 2
(B) exists and is equal to - 2
(C) does not exist because left hand limit is not equal to right hand limit
(D) None of these

MATHEMATICS /AQ # 3 E-11


TM TARGET: JEE (MAIN + ADVANCED) 2020
ENTHUSIAST & LEADER
Path to success KOTA (RAJASTHAN )
COURSE
n.3n 1
24. If nlim n +1 n = , then the range of x is (where n Î N)
®¥n(x - 2) + n.3 - 3
n
3
(A) [2, 5) (B) (1, 5) (C) (–1, 5) (D) (– ¥, ¥)
ì 2
ü
25. lim í(1 + x ) x ý (where {.} denotes the fractional part of x) is equal to
x ®0
î þ
(A) e2 –7 (B) e2 –8 (C) e2 –6 (D) None of these

MULTIPLE CORRECT CHOICE TYPE


26. If lim(2 - x + a[x - 1] + b[1 + x]) exists, then a and b can take the values (where [.] denotes the greatest
x ®1
integer function)
(A) a = 1/3, b = 1 (B) a = 1, b = –1 (C) a = 9, b = – 9 (D) a = 2, b = 2/3
-3x -3x
27. If D (Ae cos x + Be sin x) w.r.t. ‘x’ is e cos x then which of the following hold good ?
-3x

(A) A 2 + B2 = 1 (B) B – 3A = 1 (C) A + 3B = 0 (D) A – 2B = 0


é x2
ê- 2 for x£0
28. If the function f(x) defined as f (x) = ê is continuous but not derivable at x =
ê x n sin 1 for x >0
êë x
0, then n can not be
(A) 1/2 (B) 1 (C) 3/2 (D) 2
ì x 2 (sgn[x]) + {x}, 0£x<2
29. If f (x) = í , where [ ] and { } represent the greatest integer and the
îsin x + | x - 3 |, 2£x<4
fractional part function, respectively.
(A) f(x) is differentiable at x = 1 (B) f(x) is continuous but non-differentiable at x = 1
(C) f(x) is non-differentiable at x = 2 (D) f(x) is discontinuous at x = 2
30. A continuous function f(x) on R ® R satisfies the relation f(x) + f(2x + y) + 5xy = f(3x – y) + 2x2 + 1 for
" x, y Î R . Then which of the following hold (s) true
(A) f is many one (B) f has no minima
(C) f is neither odd nor even (D) f is bounded
ì1- | x |
ï x ¹ -1
31. If f (x) = í 1 + x , then f([2x]) is, (where [ ] represents the greatest integer function)
îï 1 x = -1
1
(A) continuous at x = –1 (B) continuous at x = 0 (C) discontinuous at x =(D) discontinuous at x = 1
2
32. In which of the following cases the given equations has atleast one root in the indicated interval ?
(A) x – cos x = 0 in (0, p/2)
(B) x + sin x = 1 in (0, p/6)
a b
(C) + = 0 , a, b > 0 in (1, 3)
x -1 x - 3
(D) f (x) – g (x) = 0 in (a, b) where f and g are continuous on [a, b] and f (a) > g (a) and
f (b) < g(b)
ANSWER KEY
1. (C) 2. (B) 3. (D) 4. (D) 5. (A) 6. (C) 7. (D)
8. (B) 9. (A) 10. (B) 11. (C) 12. (B) 13. (B) 14. (B)
15. (A) 16. (C) 17. (A) 18. (A) 19. (B) 20. (A) 21. (B)
22. (B) 23. (C) 24. (C) 25. (A) 26. (B,C) 27. (B, C) 28. (C,D)
29. (B, C, D) 30. (A, B) 31. (ABCD) 32. (A,B,C,D)
MATHEMATICS /AQ # 3 E-12
TM TM TARGET:JEE
TARGET: JEE(Main
(MAIN++Advanced)
ADVANCED)2020
2020
ENTHUSIAST & LEADER
Path to success KOTA (RAJASTHAN) ENTHUSIAST & LEADER
COURSE
Path to success KOTA (RAJASTHAN ) COURSE

ADVANCED QUIZ # 4 APPLICATION OF DERIVATIVES MATHEMATICS

SINGLE CORRECT CHOICE TYPE


1. The sum of all positive integral values of ‘a’, a Î [1, 500] for which the equation [ x ]3 + x – a = 0
has solution is (where [ ] denote the greatest integer function)
(A) 462 (B) 512 (C) 784 (D) 812

é x{x} + 1 0 £ x <1
2. Consider the function f (x) = ê where {x} denotes the fractional part
ë 2 - {x} 1£ x £ 2
function. Which one of the following statements is NOT correct?

(A) lim
x ®1
f (x) exists
(B) f (0) ¹ f (2)
(C) f (x) is continuous in [0,2]
(D) Rolles theorem is not applicable to f (x) in [0,2]
3. If x = 0 is the critical number of a derivable function f(x) then the function g(x) = f (x – h) + k has its critical
number at
(A) x = –h (B) x = 0 (C) x = k (D) x = h

1
4. What can one say about the local extrema of the function f (x) = x + ?
x
(A) The local maximum of f (x) is greater than the local minimum of f (x).
(B) The local minimum of f (x) greater than the local maximum of f (x).
(C) The function f (x) does not have any local extrema.
(D) f (x) has one asymptote.

5. The equation of the normal line to the curve x 2 y - 2 = y 2 - 3x - 5 at (1, 3) is

(A) 14x + 11y = 41 (B) x + y = 4 (C) 10x – 11y = 41 (D) 11x + 10y = 41

1 1
6. A funcion g (x) is defined as g(x) = f (2 x 2 - 1) + f (1 - x 2 ) and f ¢(x) is an increasing function then g (x)
4 2
is increasing in the interval

æ 2 2ö æ 2 ö æ 2 ö
(A) çç - 3 , 3 ÷÷ (B) çç - 3 , 0 ÷÷ È çç 3 , ¥ ÷÷
è ø è ø è ø
(C) (–1, 1) (D) None of these

x 3 ( x 2 + 1)
7. The length of the sub-tangent to the curve y = 3
at x = 1 is-
5
5-x

81 20 27 20
(A) (B) (C) (D)
20 81 20 27
MATHEMATICS /AQ #4 E-13
TM TARGET: JEE (MAIN + ADVANCED) 2020
ENTHUSIAST & LEADER
Path to success KOTA (RAJASTHAN)
COURSE
PASSAGE TYPE QUESTION
PARAGRAPH FOR QUESTION NO. 8 TO 10
Consider a cubic f(x) = ax3 + bx2 + cx + d that has a local maximum value of 3 at x = –2 and a local minimum
value of 0 at x = 1 (a, b, c, d Î R, a ¹ 0)
8. Coefficient of x2 is equal to
(A) 1/3 (B) 2/3 (C) 2/9 (D) None
9. Inflection point occurs when ‘x’ equal to
(A) –1/4 (B) –1/2 (C) 0 (D) 1/4
10. The area enclosed by the curve, the x-axis and the ordinates at the extremum is
(A) 16/3 (B) 22/9 (C) 38/9 (D) 9/2
PARAGRAPH FOR QUESTION NO. 11 TO 13
Consider f, g and h be three real valued functions defined on R.

ì-1, x < 0
ï
Let f ( x ) = í0, x = 0, g(x) = x(1 - x ) and h(x) is such that h¢¢ (x) = 6x – 4.
2

ï1, x > 0
î

Also h(x) has local minimum value 5 at x = 1.


11. The equation of tangent at M(2, 7) to the curve y = h (x), is
(A) 5x + y = 17 (B) x + 5y = 37 (C) x – 5y + 33 = 0 (D) 5x – y = 3

12. The area bounded by y = h (x), y = g ( f (x) ) between x = 0 and x = 2 equal to

23 20 32 40
(A) (B) (C) (D)
2 3 3 3

13. Range of the function sin -1 ( fog ( x ) ) , is


æ pö ì pü ì p pü ìpü
(A) ç 0, ÷ (B) í0, ý (C) í - , 0, ý (D) í ý
è 2ø î 2þ î 2 2þ î2þ
PARAGRAPH FOR QUESTION NO. 14 TO 16

æ 3 ö æ a2 a ö f '(x)
Let f (x) is a cubic polynomial f (x) = 2x 3 + 3 ç1 - a ÷ x 2 + 6 ç - ÷ x + b and g(x) = then (a Î
è 2 ø è 2 2ø 6

R)
14. Complete set of values of a for which f(x) has negative point of local minimum is

æ2 ö
(A) ( -¥, 0 ) (B) (1, ¥ ) - {2} (C) ç , ¥ ÷ - {2} (D) (1,¥ )
è3 ø
15. Complete set of values of a for which vertex of parabola y = g (x) have negative ordinate

æ1 ö
(A) R (B) R – {2} (C) ç ,1÷ (D) ( 0, ¥ )
è2 ø
MATHEMATICS /AQ #4 E-14
TM TARGET: JEE (MAIN + ADVANCED) 2020
ENTHUSIAST & LEADER
Path to success KOTA (RAJASTHAN)
COURSE
16. If the line y + 1 = 0 touches the parabola y = g (x) then locus of point of interesection of perpendicular tangent
to the parabola is
(A) 4x + 5 = 0 (B) 4y – 15 = 0 (C) 4x – 17 = 0 (D) None of these
MULTIPLE CORRECT CHOICE TYPE
17. Consider the function,

é min .f (t) for1 £ t £ x, 1 £ x £ 6


f(x) = x3 – 9x2 + 15x + 6 for 1 £ x £ 6 and g(x) = ê then which of the
ë x - 18 for x > 6
following hold (s) good ?
(A) g(x) is differentiable at x = 1 (B) g(x) is discontinuous at x = 6
(C) g(x) is continuous and derivable at x = 5 (D) g(x) is monotonic in (1, 5)
18. Which of the following pair(s) of curves touch each other ?
(A) y 2 = 4x and x 2 + y 2 = 6x - 1 (B) xy =4 and x2 + y2 = 8

x+2
(C) y = 2x – 3 and y = x3 – x + 1 (D) y = 6 + x – x2 and y =
x-2
19. Which of the following inequalities always hold good in (0, 1)
x2
(A) x > tan -1 x (B) cos x < 1 -
2

( )
(C) 1 + x ln x + 1 + x 2 > 1 + x 2 (D) x -
x2
2
< ln (1 + x)

20. The function f(x) = (x + 2)1/3 at x = –2


(A) is monotonic
(B) is differentiable
(C) is such that vertical tangent can be drawn at this point.
(D) changes its concavity
21. Which of the following hold(s) good for the function f (x) = 2x – 3x2/3 ?
I The function has an extremum point at x = 0.
II The function has a critical number at x = 1
III The graph is concave down at every point in (-¥, ¥ ).
(A) Statement I (B) Statement II
(C) Statement III (D) All statements I, II and III are true.
22. f : R ® R,f (x2 + x + 3) + 2f(x2 – 3x + 5) = 6x2 – 10x + 17 " x Î R
then
(A) f is strictly decreasing (B) f(x) = 0 has root in (0, 2)
(C) f(x) is an odd function (D) f(x) is invertible
23. Equation of a line which is tangent to both the curves y = x2 + 1 and y = –x2 is
1 1 1 1
(A) y = 2 x + (B) y = 2 x - (C) y = - 2 x + (D) y = - 2 x -
2 2 2 2
MATHEMATICS /AQ #4 E-15
TM TARGET: JEE (MAIN + ADVANCED) 2020
ENTHUSIAST & LEADER
Path to success KOTA (RAJASTHAN)
COURSE
24. Let f(x) = 1 + 22x2 + 32x4 + 42x6 + ......... + n2x2n – 2 then f(x) has
(A) exactly one critical point (B) exactly two critical point
(C) exactly one minimum (D) exactly one maxima
INTEGER ANSWER TYPE
25. Consider a polynomial P (x) of the least degree that has a maximum value is 6 at x = 1, and a minimum value
is 2 at x = 3. Compute the value of P (2) + P ¢ (0).
x 2 y2
26. Let a be the angle in radians between + = 1 and the circle x2 + y2 = 12 at their points of intersection.
36 4

-1 k
If a = tan , then find the value of k2.
2 3
27. The number of real solutions of the equation (9/10)x = –3 + x – x2 is

MATCH THE COLUMN :


28. Column-I Column-II
(A) If 4X – 2 x + 2 + 5 + | | b – 1| –3 | = | sin y |, x, y, b Î R, then possible (p) – 2
values of b is/are
(B) Let f (x) = min {x2, 2}, then possible integral values of ‘k’ for which (q) 2
f (x) < k for atleast one real x is/are

ì| 2x - 1| -2 £ x £ 1
(C) Letf (x) = í 2 ,then possible integers in the domain (r) 4
î x - 4 1< x £ 8
of y = f (f (x)) is/are
(D) Let f(x) = x3 + px2 + qx + 6, where p, q Î R and f' (x) < 0 in largest (s) 6

æ 5 ö
possible interval ç - , - 1÷ , then (p + q) is greater than (t) 0
è 3 ø

29. Column-I Column-II


1
2x x
(A) Lim(e +e + x) x equals (p) e–2
x®¥

(B) Let x > 1, y > 1 and (ln x)2 + (ln y)2 = ln x2 + ln y2, then the (q) e
maximum value of xln y is
ln (1 + x )
1
(C) Lim
x ® 0 sin x ò (1 - tan 2y)1/ y dy equals (r) e2
0

(s) e4

ANSWER KEY
1. (D) 2. (C) 3. (D) 4. (B) 5. (D) 6. (B) 7. (D)
8. (A) 9. (B) 10. (D) 11. (D) 12. (C) 13. (B) 14. (A)
15. (B) 16. (D) 17. (A,B,C,D) 18. (A,B) 19. (A,C,D)
20. (A, C, D) 21. (A, B) 22. (B, D) 23. (A, C) 24. (A,C)
25. [0013] 26. [0016] 27. [0000]
28. [A–p, r; B–q, r, s; C–p, q,t ; D–p, q, r, s,t] 29. [A–r; B–s; C–p]

MATHEMATICS /AQ #4 E-16


TM TM TARGET:JEE
TARGET: JEE(Main
(MAIN++Advanced)
ADVANCED)2020
2020
ENTHUSIAST & LEADER
Path to success KOTA (RAJASTHAN) ENTHUSIAST & LEADER
COURSE
Path to success KOTA (RAJASTHAN ) COURSE

ADVANCED QUIZ # 5 INTEGERAL CALCULUAS PART -1 MATHEMATICS

SINGLE CORRECT CHOICE TYPE


dy
1. If solution of differential equation - y = 1 - e - x and y ( 0 ) = y0 has a finite value. When x ® ¥ , then
dx
y0 is equal to
(A) –1/2 (B) 0 (C) 1 (D) –1

2. The area enclosed by the curve y = x & x = - y , the circle x 2 + y 2 = 2 above the x-axis, is

p 3p p
(A) (B) (C) p (D)
4 2 2

p/3

3. The value of the definite integral ò tan x × ln(cos x)dx equals


0

ln 2 2 ln 2 2
(A) -ln 2 (B) (C) - (D) -ln 2 2
2 2

òx ln (ex) dx is equal to
x
4.

(A) x x + C (B) x . ln x + C (C) (ln x)x + C (D) None

6n

5. If the value of Lim(n


-3 / 2
). å j is equal to N then N equals
n ®¥
j=1

(A) 124 (B) 196 (C) 106 (D) 96

t2
x
1+ u4
6. If F(x) = ò f (t) dt where f (t) = ò du then the value of F''(2) equals
1
u
1

7 15 15 17
(A) (B) (C) 257 (D)
4 17 17 68
p/ 2
1 - sin 2x a
7. If ò
0
(1 + sin 2x) 2
dx = where a, b are relatively prime, then the value of (a + b + ab), is
b

(A) 4 (B) 5 (C) 7 (D) 9


2p

ò ( sin 2 x - sin 4 2 x ) dx , is
2
8. Value of the definite integral
0

p p p p
(A) (B) (C) (D)
16 4 8 2

MATHEMATICS /AQ # 5 E-17


TM TARGET: JEE (MAIN + ADVANCED) 2020
ENTHUSIAST & LEADER
Path to success KOTA (RAJASTHAN)
COURSE
3/ 2
dx
9. ò
1/ 2 x (2 - x) + 1
equals

p p p p p p p p
(A) - 2 tan (B) - tan (C) - 2 tan (D) + 2 tan
3 12 3 12 6 12 3 12

10. 1 æ 1 1 1 ö equals
lim ç1 + + + ......... + ÷
n ®¥ nè 2 3 2 nø

(A) 4 (B) 2 2 (C) 2 (D) 2


p/ 4
1 + cot x
11. If the value of the definite integral ò
p/ 6
e x sin x
dx is equal to ae -p / 6 + be -p / 4 , then (a + b) equals

(A) 2 - 2 (B) 2 + 2 (C) 2 2 - 2 (D) 2 3 - 2


x 8
x2
12. If f (x) be a continuous function for all real values of x and satisfies ò f (t) dt = + ò t 2 f (t) dt , then
3
2 x

ò
-1/ 2
f (x) dx is equal to

æ5ö 1 æ8ö æ8ö 1 æ5ö


(A) ln ç ÷ ln (C) 2ln ç ÷ ln
2 çè 5 ÷ø 2 çè 8 ÷ø
(B) (D)
è8ø è5ø

æ 1 æ 1 + n2 ö 2 æ 4 + n 2 ö 3 æ 9 + n2 ö 2 ö
13. lim
Let n ®¥ ç n 2 sin ç 2 ÷ + 2
sin ç 2 ÷+ 2 ç 2 ÷ + .... + × sin ( 5 ) ÷ = y , then the value of y is
è è n ø n è n ø n è n ø n ø
(A) cos 2. sin 3 (B) sin 2. cos 3 (C) sin 2. sin 3 (D) cos 2. cos 3

(e 2x
- 1) dx
14. The value of the intergal ò is
e2x 2e4x - 2e2x + 1

1
(A) 2 - 2e -2 x + e -4 x + C (B) 2 - 2e-2x + e -4x + C
2

1
(C) +C (D) e2x + e4x - 2 + 2
-2 x
2 - 2e + e -4 x

( x sin x + cos x ) æ x 4 cos3 x - x sin x + cos x ö


15. The value of ò e ×ç ÷ dx is equal to
è x 2 cos 2 x ø

x sin x + cos x æ 1ö
(A) e x sin x +cos x ( x + xcosec x ) + c (B) e ç x cos x + x ÷ + c
è ø

æ 1 ö x sin x + cos x æ 1 ö
(C) e x sin x + cos x × ç x - +c (D) e ×ç x + +c
è x cos x ÷ø è x cos x ÷ø
MATHEMATICS /AQ # 5 E-18
TM TARGET: JEE (MAIN + ADVANCED) 2020
ENTHUSIAST & LEADER
Path to success KOTA (RAJASTHAN)
COURSE

3 p
4 + cos -1 x
2
16. ò1 2sin -1 x + 3cos -1 x + cos -1 (1 - x) dx =
4

1 1 1
(A) 1 (B) (C) (D)
2 3 4
17. Area of the region given by x 2 + y 2 - 6y £ 0 and 3y £ x 2 is

9p 9p 9p
(A) - 12 (B) -6 (C) 9p - 24 (D) +6
2 4 2
18. Let C be the curve representing the locus of the centre of the circle passing through the point
(a, b) and orthogonal to the circle x2 + y2 = k2. The area of the region bounded by C and the coordinate
axes is
a 2 + b2 - k 2 (a 2 + b 2 + k 2 ) 2 (a 2 + b 2 - k 2 ) a 2 + b 2 - ab
(A) (B) (C) (D)
4ab 8ab 2ab 2k 2
2
æ dy ö 2 dy 1
19. Solution of the equation 2x ç ÷ + (y + 6x ) + 3xy = 0 , given that y = when x = 1 is
è dx ø dx 2
(A) 3x2 + 4y – 5 = 0 (B) 4xy2 – 1 = 0 (C) 3x + 2y – 4 = 0 (D) 4x2y2 – 1 = 0
1
20. ò dx, a ¹ np, n Î Z is equal to
sin x sin(x + a)
3

(A) – 2 cosec a(cos a – tan x sin a)1/2 + C (B) – 2(cos a + cot x sin a)1/2 + C
(C) – 2 cosec a(cos a + cot x sin a)1/2 + C (D) – 2 cosec a(sin a + cot x cos a)1/2 + C
PASSAGE TYPE QUESTION
Paragraph for question No. 21to 22
Let a + b = 1, 2a 2 + 2b2 = 1 and f (x) be a continuous function such that f (x + 2) + f (x) = 2 for all
4
a
x Î[0, 2] and p = ò f (x) dx - 4 , q =
0
b
21. The value of q is
(A) 1 (B) 2 (C) –1 (D) –2

22. The value of p is


(A) –1 (B) 1 (C) 2 (D) 0
MULTIPLE CORRECT CHOICE TYPE
23. Which of the following definite integral vanishes ?
2 4
xn -1 é (log x 2) 2 ù
(A) ò n + 2 dx (n Î N) (B) ò ê x
log 2 - ú dx
1/ 2
x +1 2ë
ln 2 û

2 p
1 æ 1ö
(C) ò x
sin ç x - ÷ dx
è xø
(D) ò cos mx.sin nx dx, where (m, n Î I) and (m – n) is an even integer
1/ 2 0

MATHEMATICS /AQ # 5 E-19


TM TARGET: JEE (MAIN + ADVANCED) 2020
ENTHUSIAST & LEADER
Path to success KOTA (RAJASTHAN)
COURSE

24. If f(x) = ò | t - 1| dt , where 0 £ x £ 2 , then


0

(A) f '(x) = 1 - x 0 £ x £ 1 (B) f(x) is differentiable at x = 1

(C) f '(x) = x - 1 1 < x £ 2 (D) f ¢(1/2) = 1/2

et x

25. If f (x)= ò dt , " x > 0 , then


1 1+ t
2

(A) g(x) = tan–1x – f(x) is a decreasing function for x > 0

p
(B) f(x) ³ tan–1x – , "x ³1
4

-1 p
(C) f (x) £ tan x - , " x £ 1
4

(D) g(x) = tan -1 x - f (x) is an increasing function for x < 1

INTEGER ANSWER TYPE

26. If the value of the definite integral ò


207
C7 x 200 .(1 - x) 7dx is equal to 1 where k Î N. Find k.
0 k

1
(x - x )
3 1/ 3

27. Evaluate ò
1/ 3
x4
dx

p /2
p
28. If ò ( 2ln sin x - ln sin 2 x ) dx = - k l n k . Find minimum value of k. (k Î I)
0

29. The number of integral values of x satisfying - x 2 + 10x - 16 < x - 2 is

30. Let C be the curve passing through the point (1, 1) has the property that the perpendicular distance of the
origin from the normal at any point P of the curve is equal to the distance of P from the x-axis. If the area
kp
bounded by the curve C and x-axis in the first quadrant is square units, then find the value of k.
2

Aa 2
31. If the area of the loop of the curve, ay2 = x2 (a – x) is then the value of (A + B) is (where A and B are
B
relatively prime numbers and a >0)
4 4
-1
32. If ò f (x) dx = 4 and ò (3 - f (x)) dx = 7 , then the value of ò2
f (x) dx is
-1 2

MATHEMATICS /AQ # 5 E-20


TM TARGET: JEE (MAIN + ADVANCED) 2020
ENTHUSIAST & LEADER
Path to success KOTA (RAJASTHAN)
COURSE
MATCH THE COLUMN :
33. Column-I Column-II

p æpö
(A) ò xln sin xdx (p) ç ÷ ln2
0 è8ø

¥ dx -p2
(B) ò 0
ln(x + x -1 )
1+ x2
(q)
2
ln2

p/4
(C) ò ln(1 + tan x) dx (r) – p ln 2
0

p
(D) ò ln(1 - cos x) dx (s) p ln 2
0

ANSWER KEY
1. (A) 2. (D) 3. (C) 4. (A) 5. (D) 6. (C) 7. (C)
8. (B) 9. (A) 10. (A) 11. (A) 12. (B) 13. (C) 14. (A)
15. (C) 16. (D) 17. (A) 18. (B) 19. (B) 20. (C) 21. (A)
22. (D) 23. (A,D) 24. (A, B, C, D) 25. (A, B) 26. [0208]
27. [0006] 28. [0002] 29. [0003] 30. [0001]
31. [0023] 32. [0005] 33. [A – q, B – s, C – p, D – r]
MATHEMATICS /AQ # 5 E-21
TM TM TARGET:JEE
TARGET: JEE(Main
(MAIN++Advanced)
ADVANCED)2020
2020
ENTHUSIAST & LEADER
Path to success KOTA (RAJASTHAN) ENTHUSIAST & LEADER
COURSE
Path to success KOTA (RAJASTHAN ) COURSE

ADVANCED QUIZ # 6 INTEGERAL CALCULUAS PART -2 MATHEMATICS


SINGLE CORRECT CHOICE TYPE
1. If ò x5 (1 + x 3 ) 2 / 3 dx = A(1 + x3)8/3 + B(1+ x3)5/3 + C, then

1 1 1 1 1 1
(A) A= ,B= (B) A= , B= – (C) A= – ,B= (D) None of these
4 5 8 5 8 5
(1 - cos q) 2/ 7
2. The value of the integral ò d q is
(1 + cos q)9 / 7
11 11 11
7æ qö7 7æ qö7 7 æ qö7
(A) ç tan ÷ + C (B) ç cos ÷ + C (C) ç sin ÷ + C (D) None of these
11 è 2ø 11 è 2ø 11 è 2 ø

x 9 dx
3. ò (x 2 + 4)6 is equal to
-5 -5
1 æ 1 ö 1æ 1 ö 1
(1 + 4x 2 ) + c
-5
(A) ç 4+ 2 ÷ +c (B) ç 4 + 2 ÷ + c (C) (D) None of these
5x è x ø 5è x ø 10x
tan -1 x
4. òe (1 + x + x 2 ) d(cot -1 x) is equal to

(A) -etan -1 x + c (B) e tan -1 x + c (C) - xetan -1 x + c (D) xetan -1 x + c

3 + 2 cos x
5. ò dx is equal to
( 2 + 3cos x )
2

æ sin x ö æ 2cos x ö æ 2 cos x ö æ 2sin x ö


(A) ç ÷+c (B) ç ÷+c (C) ç ÷+c (D) ç ÷+c
è 3cos x + 2 ø è 3sin x + 2 ø è 3cos x + 2 ø è 3sin x + 2 ø
0
é
-1 æ ö -1 æ 2 ö 1 ù
6. ò êcot ç 2cos x - 1 ÷ + cot ç cos x - 2 ÷ ú dx is equal to
-p / 3 ë è ø è øû

p2 p2 p2 3p2
(A) (B) (C) (D)
6 3 8 8

-1
sin t t
sin z2 dy æ æ p öö
7. If x = ò sin z dz, y = ò dz , then is equal to ç t Î ç 0, ÷ ÷
c k z dx è è 2 øø

tan t tan t tan t tan t 2


(A) (B) 2 (C) (D)
2t t 2t 2 2t 2
x
8. If f (x) = cos x - ò (x - t)f (t)dt , then f ''(x) + f (x) is equal to
0

(A) –cos x (B) –sin x (C) ò (x - t)f (t)dt (D) zero


0

MATHEMATICS /AQ #6 E-22


TM TARGET: JEE (MAIN + ADVANCED) 2020
ENTHUSIAST & LEADER
Path to success KOTA (RAJASTHAN)
COURSE

p p
9. Let f(x) = sin3x + l sin2x for - < x < and l > 0. The complete interval in which l should lie in order
2 2
that f(x) has one critical point
æ 1 ö æ 3 ö æ 3ö æ3 ö
(A) (0, 1) (B) ç 0 , ÷ (C) ç 0 , ÷ (D) ç 0, ÷ È ç , ¥ ÷
è 2 ø è 2 ø è 2ø è2 ø
1

10. ò [f (x)g"(x) - f "(x)g(x)]dx is equal to (where f(0) = g(0) = 0)


0

(A) f '(1)g(1) - f (1)g '(1) (B) f (1)g '(1) + f '(1)g(1)


(C) f (1)g '(1) - f '(1)g(1) (D) None of the above
20

ò [cot
-1
11. The value of integral x]dx [where, [ ] denotes the greatest integer function] is
-10

(A) 30 + cot 1 (B) 30 + cot 1 + cot 2


(C) 30 + cot 1 + cot 2 + cot 3 (D) None of these
1
12. Area bounded by y = and x - axis is
x - 2x + 2
2

p
(A) 2p sq. units (B) sq. units (C) 2 sq. units (D) p sq. units
2
p
13. If I n = ò x n sin xdx , tehn value of I5 + 20I3 is
0

(A) 0 (B) p5 (C) p5 – p3 (D) None of these


14. Let f(x) = x , g(x) = cosx and a, b, (a < b) the roots of the equation 18x2 – 9px + p2 = 0. The area
2

bounded by the curve y = fog(x), the ordinates x = a, x = b and the x-axis is


1 1 p p
(A) (p - 3) (B) (p - 3) (C) (D)
6 12 12 6

é x2 ù
15. Area bounded by the curves y = ê + 2 ú , y = x – 1 and x = 0 above x-axis is (where [ ] denotes the
ë 64 û
greatest integer function)
(A) 2 (B) 3 (C) 4 (D) None of these

16. The orthogonal trajectories of the family of curves an –1y = xn are given by (n Î N, n ³ 2)
(A) xn + n2y = constant (B) ny2 + x2 = constant
(C) n2x + yn = constant (D) n2x – yn = constant
1- t2 2t
17. Area enclosed by the curve y = f(x) defined parametrically as x = ,
2 y= is equal to
1+ t 1+ t2

3p 3p
(A) psq. units (B) p/2 sq. units (C) sq. units (D) sq. units
4 2
MATHEMATICS /AQ #6 E-23
TM TARGET: JEE (MAIN + ADVANCED) 2020
ENTHUSIAST & LEADER
Path to success KOTA (RAJASTHAN)
COURSE
PASSAGE TYPE QUESTION
PARAGRAPH FOR QUESTION NO. 18 TO 20
2
y = f (x) satisfies the relation ò f (t) dt = x + ò t 2f (t) dt - 2
x 2

2 2 x
18. The range of y = f (x) is

é 1 1ù
(A) [0, ¥) (B) R (C) (-¥, 0] (D) ê - , ú
ë 2 2û

19. The value of ò f (x) dx is


-2

(A) 0 (B) –2 (C) 2 log e 2 (D) None of these


20. The value of x for which f(x) is increasing is
(A) (-¥,1] (B) [-1, ¥ ) (C) [–1, 1] (D) None of these
PARAGRAPH FOR QUESTION NO. 21 TO 23
p/2
f (x) = sin x + ò (sin x + t cos x) f (t) dt
-p / 2

21. The range of f (x) is

é 3 3ù é 5 5ù é 5 5ù
(A) ê - 2 , 2 ú (B) ê - 3 , 3 ú (C) ê - 2 , 2 ú (D) None of these
ë û ë û ë û
22. f(x) is not invertible for

é p -1 p -1 ù é -1 1 1ù
(A) x Î ê - - tan 2, - tan 2 ú (B) x Î ê tan , p + tan -1 ú
ë 2 2 û ë 2 2û

(C) x Î éë p + cot -1 2, 2p + cot -1 2 ùû (D) None of these

p/ 2
23. The value of ò f (x) dx is
0

(A) 1 (B) –2 (C) – 1 (D) 2


PARAGRAPH FOR QUESTION NO. 24 TO 26
p/ 2

f(x) satisfies the relation f (x) - l ò sin x cos tf (t) dt = sin x


0

24. If l > 2, then f(x) decreases in which of the following interval


(A) (0, p) (B) (p/2, 3p/2) (C) (–p/2, p/2) (D) None of these
25. If f(x) = 2 has at least one real root, then complete l is
(A) [1, 4] (B) [-1, 2] (C) [0,1] (D) [1,3] – {2}

MATHEMATICS /AQ #6 E-24


TM TARGET: JEE (MAIN + ADVANCED) 2020
ENTHUSIAST & LEADER
Path to success KOTA (RAJASTHAN)
COURSE
p/ 2

26. If ò f (x)dx = 3 , then the value of l is


0

(A) 1 (B) 3/2 (C) 4/3 (D) None of these


MULTIPLE CORRECT CHOICE TYPE
p/2
sin(2n - 1)x p/2 2
27. If A n = ò dx ; Bn = ò æç sin nx ö÷ dx , for n Î N, then
0 sin x 0 è sin x ø

(A) A n +1 = A n (B) Bn +1 = B n (C) A n +1 - A n = Bn +1 (D) B n +1 - Bn = A n +1

p/ 2
sin 2 nx
28. If I n = ò sin 2 x
dx , then n Î N
0

np
(A) I n = (n Î N) (B) In = np (n Î N)
2
(C) I1, I2, I3, .......... In, .............. is a A.P. (D) sin(I16) = 0
ln 4
39
29. If f (x) = ae 2x
+ be + cx satisfies the conditions f(0) = –1, f '(l n 2) = 31 ,
x
ò (f (x) - cx)dx =
2 , then
0

(A) a = 5 (B) b = –6 (C) c = 2 (D) a = 3

xdx + ydy a 2 - x 2 - y2
30. The solution of = is
xdy - ydx x 2 + y2

é æ yö ù é æ yö ù
(A) x 2 + y 2 = a êsin ç tan -1 ÷ + c ú (B) x 2 + y 2 = a êcos ç tan -1 ÷ + c ú
ë è xø û ë è xø û

é æ yö ù æ -1 1 ö
(C) x 2 + y 2 = a ê tan ç sin -1 ÷ + c ú (D) y = x tan ç c + sin x 2 + y2 ÷
ë è xø û è a ø
INTEGER ANSWER TYPE

ex
f (a ) f (a ) I2
31. If f (x) =
1 + ex
, I1 = ò
f (- a )
xg(x(1 - x)) dx and I 2 = ò g(x(1 - x)) dx , then the value of I1 is
f (- a )

éa f (x) dx 2a
f (x) dx
32. f (x) > 0 " x Î R and is bounded. If n ®¥ ê ò
lim + a ò +
ë 0 f (x) + f (a - x) a f (x) + f (3a - x)

3a
f (x) dx n -1
na
f (x) dx ù
a2 ò + .......+ a ò ú = 7 / 5 (where a Î(0, 1)) and a is
2a f (x) + f (5a - x) (n -1)a f (x) + f[(2n - 1)a - x û

b
equal to find the value of b (b Î I+) .
19

MATHEMATICS /AQ #6 E-25


TM TARGET: JEE (MAIN + ADVANCED) 2020
ENTHUSIAST & LEADER
Path to success KOTA (RAJASTHAN)
COURSE

Let F(x) be a non-negative continuous function defined on R such that F(x) + F æç x + ö÷ = 3 . Find the value
1
33.
è 2ø
1500

of ò
0
F(x) dx .

p/4
p3 l n 2
34. Let I = ò ( px - 4x ) l n (1 + tan x ) dx . If the value of I = k where k Î N , find k.
2

35. If the area of the region bounded by the curves, y = x2, y = | 2 – x2| and y = 2 which lies to the right of the

K - 12 2
line x = 1 is Sq. units then find the value of K where (K Î I).
3

1
æ 1 ö
36. If ò t 2 (f (t))dt = (1 - sin x) then f ç
è 3ø
÷ is
sin x

1/ n
é p 2p np ù
37. The value of lim
n ®¥ ê
tan tan .....tan ú is
ë 2n 2n 2n û

38. The area bounded by the curves y = x , 2y + 3 = x and x-axis in the 1st quadrant is
MATCH THE COLUMN :
39. Column-I Column-II

10
(A) The area bounded by the curve y = x|x|, x-axis and the (p) sq. units
3
ordinates x = 1, x = –1

64
(B) The area of the region bounded the line x – y + 2 = 0, (q) sq. units
3
x = 0 and the curve x = Öy

2
(C) The area enclosed between the curves y2 = x and y = |x| (r) sq. units
3

1
(D) The area bounded by parabola y2 = x, straight line y = 4 (s) sq. units
6
and y- axis

ANSWER KEY
1. (B) 2. (A) 3. (D) 4. (C) 5. (A) 6. (A) 7. (C)
8. (A) 9. (D) 10. (C) 11. (C) 12. (D) 13. (B) 14. (C)
15. (C) 16. (B) 17. (A) 18. (D) 19. (A) 20. (C) 21. (B)
22. (D) 23 (C) 24. (C) 25. (D) 26. (C) 27. (A, D)
28. (A, C, D) 29. (A, B) 30. (AD) 31. [0002] 32. [0014] 33. [2250]
34. [0192] 35. [0020 ] 36. [0003] 37. [0001] 38. [0009] 39. [A-r; B-p; C-s; D-q]
MATHEMATICS /AQ #6 E-26
TM TM TARGET:JEE
TARGET: JEE(Main
(MAIN++Advanced)
ADVANCED)2020
2020
ENTHUSIAST & LEADER
Path to success KOTA (RAJASTHAN) ENTHUSIAST & LEADER
COURSE
Path to success KOTA (RAJASTHAN ) COURSE

ADVANCED QUIZ # 7 STRAIGHT LINE & CIRCLE PART -1 MATHEMATICS


SINGLE CORRECT CHOICE TYPE
1. The vertices of a triangle ABC are A(p2, – p), B(q2, q), C(r2, – r). The area of the triangle ABC is
(p, q, r > 0 and r – p < 0)
1 1
(A) (p + q) (q + r) (r + p) (B) (p – q) (q + r) (r + p)
2 2

1 1
(C) (p + q) (q – r) (r – p) (D) (p + q) (q + r) (p – r)
2 2
2. One side of a rectangle lies along the line 4x + 7y + 5 = 0, two of its vertices are (–3, 1) and (1, 1) Which
of the following may be an equation of one of the other three straight lines ?
(A) 7x – 4y = 3 (B) 7x – 4y + 3 = 0 (C) y + 1 = 0 (D) 4x + 7y = 3
3. Let A(5, 12), B(–13 cos q, 13 sin q) and C(13 sin q, –13 cos q) are angular points of DABC where qÎ R .
The locus of orthocentre of DABC is
(A) x – y + 7 = 0 (B) x – y – 7 = 0 (C) x + y – 7 = 0 (D) x + y + 7 = 0
4. Minimum distance between the circles x 2 + y 2 = 144 and x 2 + y 2 - 6x - 8y = 0 , is
(A) 0 (B) 2 (C) 7 (D) 17
5. Four unit circles pass through the origin and have their centres on the coordinate axes. The area of the
quadrilateral whose vertices are the points of intersection (in pairs) of the circles, is
(A) 1 sq. unit (B) 2 2 sq. units
(C) 4 sq. units (D) can not be uniquely determined, insufficient data
6. Equation of the circle which cuts the circle x 2 + y 2 + 2x + 4y – 4 = 0 and the lines
xy – 2x – y + 2 = 0 othogonally, is
(A) x 2 + y 2 - 2x - 4y - 6 = 0 (B) x 2 + y 2 - 2x - 4y + 6 = 0

(C) x 2 + y 2 - 2x - 4y + 12 = 0 (D) Not possible to determine.

7. Let C be a circle x 2 + y 2 = 1. The line l intersects C at the point (–1, 0) and the point P. Suppose that the
slope of the line l is a rational number m. Number of choices for m for which both the coordinates of P are
rational, is
(A) 3 (B) 4 (C) 5 (D) infinitely many
8. From a point R(5,8) two tangents RP and RQ are drawn to a given circle S = 0 whose radius is 5. If
circumcentre of the triangle PQR is(2,3) then the equation of circle S = 0 is
(A) x2 + y2 + 2x + 4y – 20 = 0 (B) x2 + y2 + x + 2y – 10 = 0
(C) x2 + y2 – x – 2y – 20 = 0 (D) x2 + y2 – 4x – 6y – 12 = 0
9. A line with positive rational slope, passes through the point M(6, 0) and is at a distance of 5 from N(1, 3).
The slope of line equals

15 8 5 8
(A) (B) (C) (D)
8 15 8 5
MATHEMATICS /AQ # 07 E-27
TM TARGET: JEE (MAIN + ADVANCED) 2020
ENTHUSIAST & LEADER
Path to success KOTA (RAJASTHAN)
COURSE
10. The area bounded by the circles x 2 + y 2 = r 2 , r = 1, 2 and the rays given by 2x 2 - 3xy - 2y 2 = 0, y > 0 is

p p 3p
(A) sq unit (B) sq unit (C) sq unit (D) p sq unit
4 2 4
11. The equations of perpendicular bisectors of two sides AB and AC of a triangle ABC are x + y + 1 = 0 and
x – y + 1 = 0 respectively. If circumradius of DABC is 2 units, then the locus of vertex A is
(A) x2 + y2 + 2x – 3 = 0 (B) x2 + y2 + 2x + 3 = 0
(C) x2 + y2 – 2x + 3 = 0 (D) x2 + y2 – 2x – 3 = 0
12. If the points of intersection of lines L1 : y – m1x – k = 0 and L2 : y – m2x – k = 0 (m1 ¹ m2) lies inside a
triangle formed by the lines 2x + 3y = 1, x + 2y = 3 and 5x – 6y – 1 = 0, then complete set of values of k are

æ1 3ö æ1 ö æ 3ö æ -3 ö
(A) ç , ÷ (B) ç , 1 ÷ (C) ç 0, ÷ (D) ç , 0 ÷
è3 2ø è2 ø è 2ø è 2 ø
13. Let all the points on the curve x2 + y2 – 10x = 0 are reflected about the line y = x + 3. The locus of the
reflected points is in the form x2 + y2 + gx + fy + c = 0. The value of (g + f + c) is equal to
(A) 28 (B) –28 (C) 38 (D) –38

3p
14. Locus of midpoint of all chords subtending an angle of at the minor arc of the circle x2 + y2 = 2 is
4
(A) director circle of x2 + y2 = 2 (B) director circle of x2 + y2 = 1
(C) circle, whose director circle is x2 + y2 = 2 (D) circle whose director circle is x2 + y2 = 1
15. Let a, b, c Î R and satisfying (a + c)2 + 4b2 – 4ab – 4bc = 0 then the variable line ax + by + c = 0 passes
through a fixed point whose co-ordinates are
(A) (1, –2) (B) (–1, 2) (C) (1, 2) (D) (–1, –2)
16. The point P (2, 1) is shiffed through a distance 3 2 units measured parallel to the line x + y = 1 in the
direction of decreasing ordinates, to reach at Q. The image of Q with respect to given line is
(A) (3, –4) (B) (–3, 2) (C) (0, –1) (D) None of these
17. If p and q be the longest and the shortest distances respectively of the point (–7, 2) from any point (a, b) on
the curve whose equation is x 2 + y 2 - 10 x - 14 y - 51 = 0 then geometric mean of p and q is

(A) 2 11 (B) 5 5 (C) 13 (D) None of these


18. Two circles of radii r1 and r2 are both touching the coordinate axes and intersecting each other orthogonally.
r1
The value of
r2 (Where r1 > r2) equals

(A) 2 + 3 (B) 3 +1 (C) 2 – 3 (D) 2 + 5


19. A system of lines is given as y = mix + ci, where mi can take any value out of 0,1, – 1 and when mi is positive
then ci can be 1 or – 1 when mi equal 0, ci can be 0 or 1 and when mi equal – 1, ci can take 0 or 2. Then the
area enclosed by all these straight lines is

3 3 3
(A) ( 2 - 1) (B) (C) (D) 2
2 2 2

MATHEMATICS /AQ # 07 E-28


TM TARGET: JEE (MAIN + ADVANCED) 2020
ENTHUSIAST & LEADER
Path to success KOTA (RAJASTHAN)
COURSE
20. Number of integral values of ‘b’ for which the origin and the point (1,1) lie on the same side of the straight
line a2x + aby + 1 = 0, for all a Î R - {0} is
(A) 4 (B) 3 (C) 5 (D) None of these
Match the Column :
21. Consider two circle C1 of radius ‘a’ and C2 of radius ‘b’ (b>a) both lying in the first quadrant and touching
b
the coordinate axes. In each of the conditions listed in column-I, the ratio of is given in column-II
a
Column-I Column-II
(A) C1 and C2 touch each other (p) 2 + Ö2
(B) C1 and C2 are orthogonal (q) 3
(C) C1 and C2 intersect so that the common chord is longest (r) 2 + Ö3
(D) C2 passes through the center of C1 (s) 3 + 2Ö2
(t) 3 – 2Ö2
22. Column - I Column - II
(A) The four lines 3x – 4y + 11 = 0 ; 3x – 4y – 9 = 0 ; (p) a quadrilateral which
4x + 3y + 3 = 0 and 4x + 3y – 17 = 0 enclose a is neither a parallelogram
figure which is nor a trapezium nor a kite.
(B) The lines 2x + y = 1, x + 2y = 1, 2x + y = 3 and (q) a parallelogram which is
x + 2y = 3 form a figure which is neither a rectangle nor a
rhombus
(C) If ‘O’ is the origin, P is the intersection of the lines (r) a rhombus which is .
2x 2 - 7xy + 3y 2 + 5x + 10y - 25 = 0 , A and B are not a square
the points in which these lines are cut by the line
x + 2y – 5 = 0, then the points O, A, P, B (in some (s) a square
order) are the vertices of
23. Column - I Column - II
(A) If the straight line y = kx " k ÎI touches or passes outside (p) 1
the circle x 2 + y 2 - 20y + 90 = 0 then |k| can have the value
(B) Two circles x 2 + y 2 + px + py - 7 = 0 (q) 2
and x 2 + y 2 - 10x + 2py + 1 = 0 intersect each other orthogonally
then the value of p can be
(C) If the equation x 2 + y 2 + 2lx + 4 = 0 and x 2 + y 2 - 4ly + 8 = 0 (r) 3
represent real circles then the value of l can be
(D) Each side of a square is of length 4. The centre of the square (s) 5
is (3, 7). One diagonal of the square is parallel to y = x.
The possible abscissae of the vertices of the square can be

ANSWER KEY
1. (D) 2. (A) 3. (A) 4. (B) 5. (C) 6. (A) 7. (D)
8. (A) 9. (B) 10. (C) 11. (A) 12. (A) 13. (C) 14. (C)
15. (A) 16. (A) 17. (A) 18. (A) 19. (C) 20. (B)
21. (A- s; B-r; C-q, D-p) 22. [A–S ; B–R ; C–Q] 23. [A–pqr; B–qr; C–qrs; D–ps]
MATHEMATICS /AQ # 07 E-29
TM TM TARGET:JEE
TARGET: JEE(Main
(MAIN++Advanced)
ADVANCED)2020
2020
ENTHUSIAST & LEADER
Path to success KOTA (RAJASTHAN) ENTHUSIAST & LEADER
COURSE
Path to success KOTA (RAJASTHAN ) COURSE

ADVANCED QUIZ # 08 STRAIGHT LINE & CIRCLE PART-2 MATHEMATICS


PASSAGE TYPE QUESTION
PARAGRAPH FOR QUESTION NO. 1 TO 3
Consider a triangle PQR with coordinates of its vertices as P(–8, 5) ; Q(–15, –19) and R (1, –7). The
bisector of the interior angle of P has the equation which can be written in the form ax + 2y + c = 0.
1. The distance between the orthocentre and the circumcentre of the triangle PQR is
1 1 1 3
(A) 12 (B) 15 (C) 10 (D) 11
2 2 4 4
2. Radius of the incircle of the triangle PQR is
(A) 4 (B) 5 (C) 6 (D) 8
3. The sum of the coefficients (a + c)
(A) 129 (B) 78 (C) 89 (D) 99
PARAGRAPH FOR QUESTION NO. 4 TO 6
The equation of perpendicular bisectors of the sides AB and AC of triangle ABC are x – y – 4 = 0 and
2x – y – 5 = 0 respectively. The coordinates of the vertex A is (–2, 3).
4. The coordinates of the circumcentre is
(A) (1, –3) (B) (–1, 3) (C) (–1, –3) (D) (1,3)
5. Equation of the side BC is
(A) 7x + y + 55 = 0 (B) x – 7y + 43 = 0 (C) 7x – y = 55 (D) 7x – y = 43
6. Area of the triangle ABC, is
98 108 119
(A) (B) (C) (D) none
5 5 5
PARAGRAPH FOR QUESTION NO. 7 TO 9
Consider the circles S1 : x 2 + y 2 - 6y + 5 = 0;S2 : x 2 + y 2 - 12x + 35 = 0
and a variable circle S : x 2 + y 2 + 2gx + 2fy + c = 0
7. Number of common tangents to S1 and S2 is
(A) 1 (B) 2 (C) 3 (D) 4
8. Length of a transverse common tangent to S1 and S2 is
(A) 6 (B) 2 11 (C) 35 (D) 11 2
9. If the variable circle S = 0 with centre C moves in such a way that it is always touching externally the circles
S1=0 and S2=0 then the locus of the centre C of the variable circle is
(A) a circle (B) a parabola (C) an ellipse (D) a hyperbola
MULTIPLE CORRECT CHOICE TYPE
10. The origin, the intersection of the lines 2x2 + 5xy – 3y2 + 3x – 5y – 2 = 0 and the points in which these lines
are cut by the line 3x – 5y = 2, are the vertices of a
(A) parallelogram (B) rectangle (C) rhombus (D)square
11. Let L1 : 3x + 4y = 1& L 2 : 5x - 12y + 2 = 0 be two given lines. Let image of every point on L1 with respect to
a line L lies on L2, then possible equations of L can be
(A) 14x + 112y – 23 = 0 (B) 64x – 8y – 3 = 0 (C) 11x – 4y = 0 (D) 52y – 45x = 7
12. The sides of a right triangle T1 are 20 ; x and hypotenuse y. The sides of another right triangle T2 are 30,
x – 5 and hypotenuse y + 5. If P1 and P2 are the radii of the circles inscribed and D1 and D2 are the areas of
the triangles T1 and T2 respectively then which of the following hold good ?
(A) 6D1 = 5D2 (B) 8D1 = 7D2 (C) P1 = P2 (D) 2P1 = P2
MATHEMATICS /AQ # 08 E-30
TM TARGET: JEE (MAIN + ADVANCED) 2020
ENTHUSIAST & LEADER
Path to success KOTA (RAJASTHAN)
COURSE
13. In the xy plane, the line ‘ l1 ’ passes through the point (1, 1) and the line ‘ l 2 ’ passes through the point (–1, 1). If
the difference of the slopes of the lines is 2, the locus of the point of intersection of the lines l1 and l 2 can be
(A) y = x2 (B) y = 2 – x2 (C) y2 = x (D) y2 = x – 2
14. The line 3x + 6y = k intersect the curve 2x 2 + 2xy + 3y 2 = 1 at points A and B. The circle on AB as
diameter passes through origin. The possible value of k is
(A) 3 (B) 4 (C) –4 (D) –3
15. A circle x + y = 1 cuts the line x + y = k and makes the chord with length 1. The value of ‘k’ is
2 2

3 6 6 6
(A) (B) (C) - (D)
2 2 2 2
16. Let DPQR be formed by the common tengents to the circles x + y + 6x = 0 and x + y2 – 2x = 0 then
2 2 2

which of the following is(are) true ?


(A) Centroid of DPQR is (1, 0)
(B) Radius of circle inscribed in DPQR is 1
(C) Area of DPQR is 3 3
(D) The y-intercept of common tangent having negative slope is 3
17. Let A º (1, 2) ; B º (3, 4) and C º (x, y) be any point satisfying
(x – 1)(x –3) + (y – 2)(y – 4) = 0 then which of the following hold good ?
(A) Maximum possible area of the triangle ABC is 2 square units
(B) Maximum number of positions of C in the XY plane for the area of the triangle ABC to be unity, is 4
(C) Least radius of the circle passing through A and B is 2
(D) If ‘O’ is the origin then the orthocentre as well as circumcentre of the triangle OAB lies outside this
triangle
INTEGER ANSWER TYPE
18. Consider 3 lines L1 : 5x – y + 4 = 0
L2 : 3x – y + 5 = 0
L3 : x + y + 8 = 0
If these lines enclose a triangle ABC and sum of the squares of the tangent of the interior angles can be
expressed in the form p/q where p and q are relatively prime numbers, compute the value of (p + q).
19. Circle S1 is centered at (0, 3) with radius 1. Circles S 2 & S1 are externally tangent to each other & S2 is
x2
tangent to the x-axis. If an equation for the locus of the centres (x, y) of circle S2 is y = 1 + .
a
(a Î I+) Find a.
20. A circle with center in the first quadrant is tangent to y = x + 10 ; y = x – 6, and the y-axis. Let (h,k) be the
centre of the circle. If the value of (h + k) = a + b a where a is a surd, find the value of a + b.
21. Consider a family of circles passing through two fixed points A (3, 7) & B (6, 5). The chords in which the
circle x 2 + y 2 - 4 x - 6 y - 3 = 0 cuts the members of the family are concurrent at a point (a, b/3), find
a + b. (a, b Î I+)
Answer Key
1. (A) 2. (B) 3. (C) 4. (A) 5. (C) 6. (B) 7. (D)
8. (A) 9. (D) 10. (A) 11. (A, B) 12. (B,C) 13. (A,B) 14. (A,D)
15. (B,C) 16. (A, B, C) 17. (A, B, C, D) 18. [0465] 19. [0008]
20. [0010] 21. [0025]
MATHEMATICS /AQ # 08 E-31
TM TM TARGET:JEE
TARGET: JEE(Main
(MAIN++Advanced)
ADVANCED)2020
2020
ENTHUSIAST & LEADER
Path to success KOTA (RAJASTHAN) ENTHUSIAST & LEADER
COURSE
Path to success KOTA (RAJASTHAN ) COURSE

ADVANCED QUIZ # 09 CONIC SECTION MATHEMATICS


SINGLE CORRECT CHOICE TYPE

x 2 y2
1. The magnitude of the gradient of the tangent at any extremity of latera recta of the hyperbola - = 1 is
a 2 b2
equal to (where e is the eccentricity of the hyperbola)
(A) be (B) e (C) ab (D) ae

x 2 y2
2. Any ordinate MP of an ellipse + = 1 meets the auxillary circle in Q, then locus of point of intersection
25 9
of normals at P and Q is
(A) x2 + y2 = 8 (B) x2 + y2 = 34 (C) x2 + y2 = 64 (D) x2 + y2 = 15
3. If the lines y = x and x + y = 0 are tangents to a variable circle, then locus of centre of the circle, is
(A) x + y = 1 (B) xy = 0 (C) xy = 1 (D) 2x + 3y = 5
4. Tangents to the parabola y = 4ax are drawn at points whose abscissa are in ratio k2 : 1. Then the locus of
2

their point of intersection is (k > 0)


2 2
æ 1 ö æ 1 ö
(A) y = ax ç k + (B) y = ax ç k +
2 2
÷ ÷
è kø è kø

2æ 1 ö æ 1 ö
(C) y = 4ax ç k + (D) y = ax ç k +
2
÷ ÷
è kø è kø
5. If two of the conormal points form any point to the parabola y2 = 8x lie on x + y = 2, then the sum of the
coordinates of the third conormal point is
(A) 2 (B) 4 (C) 16 (D) 0
6. The locus of the middle points of chords of an ellipse the tangents at the ends of which intersect at right
x 2 y2
angles for the ellipse 2 + 2 = 1 is
a b

(A) a 4 b 4 ( x 2 + y 2 ) = ( b 2 x 2 + a 2 y 2 ) (B) a 4 b 4 ( x 2 + y 2 ) = ( a 2 + b 2 )( b 2 x 2 + a 2 y 2 )
2 2

(C) a 2 b 2 ( x 2 + y 2 ) = b 2 x 2 + a 2 y 2 (D) x 2 + y 2 = a 2 b 2 ( b 2 x 2 + a 2 y 2 )
2

7. If f(x) = x3 + ax2 + bx + g, where a,b,g are rational numbers and two roots of f(x) = 0 are eccentricities
of a parabola and a rectangular hyperbola, then a + b + g is equal to
(A) – 1 (B) 0 (C) 1 (D) 2

x 2 y2
8. If a tangent of slope 2 of the ellipse + = 1 is normal to the circle x 2 + y 2 + 4x + 1 = 0 , then the
a 2 b2
maximum value of ab is (a, b > 0)
(A) 4 (B) 2 (C) 1 (D) None of these

MATHEMATICS /AQ # 09 E-32


TM TARGET: JEE (MAIN + ADVANCED) 2020
ENTHUSIAST & LEADER
Path to success KOTA (RAJASTHAN)
COURSE
9. If values of m for which the line y = mx + 2 5 touches the hyperbola 16x 2 - 9y 2 = 144 are the roots of

the equation x 2 - ( a + b ) x - 4 = 0 , then value of (a + b) is equal to

(A) 2 (B) 4 (C) zero (D) None of these

x 2 y2
10. From a point on the line y = x + c, (c is parameter), tangents are drawn to the hyperbola - = 1 such
2 1

x1
that chords of contact pass through a fixed point ( x1 , y1 ) . Then y is equal to
1

(A) 2 (B) 3 (C) 4 (D) None of these


11. The straight line y = mx + c (m > 0) touches the parabolas y2 = 8(x + 2) then the minimum value taken by c is
(A) 4 (B) 8 (C) 12 (D) 6

x 2 y2
12. If the normal at ‘q’ on the hyperbola 2 - 2 = 1 meets the transverse axis at G, then AG × A 'G is
a b
(Where A and A’ are the vertices of the hyperbola).
(A) a2 sec q (B) a2(e4 sec2 q + 1) (C) a2(e4 sec2 q – 1) (D) None of these

x2 y2
+ y 2 = 1 meets the ellipse x + 2 = 1 in four distinct points and a = b2 – 5b + 7, then b
2
13. If the ellipse
4 a
does not lie in

(A) [4,5] (B) ( -¥, 2 ) È ( 3, ¥ ) (C) ( -¥, 2 ) (D) [2,3]

14. From the point (–1, 2) tangent lines are drawn to the parabola y 2 = 4x. The area of the triangle formed by
the chord of contact and the tangent is
(A) 4 2 (B) 4 (C) 8 (D) 8 2
PASSAGE TYPE QUESTION
PARAGRAPH FOR QUESTION NOS. 15 TO 17
A line drawn from point P(–1,2) meet the hyperbola xy = c2 at A and B (points A & B are on the same side of
P)
15. A Point Q on this line in such a way that PA, PQ and PB are in A.P. Then locus of Q
(A) x = y (1 + 2x) (B) x = y (1 + x) (C) 2x = y(1 + 2x) (D) none of these
16. If PA, PQ and PB are in G.P. then locus of Q
(A) xy – y + 2x – c2 = 0 (B) xy + y – 2x + c2 = 0
(C) xy + y + 2x + c2 = 0 (D) xy – y – 2x – c2 = 0
17. If PA, PQ and PB are in H.P. then locus of Q
(A) 2x – y = 2c2 (B) x – 2y = 2c2 (C) 2x + y = 2c2 (D) x + 2y = 2c2

MATHEMATICS /AQ # 09 E-33


TM TARGET: JEE (MAIN + ADVANCED) 2020
ENTHUSIAST & LEADER
Path to success KOTA (RAJASTHAN)
COURSE
MULTIPLE CORRECT CHOICE TYPE
18. The tangent at any point ‘P’ on the standard ellipse with focii as S & S’ meets the tangents at the vertices
A & A’ in the points V & V’ then

(A) (AV)(A 'V ') = b 2 (B) (AV)(A 'V ') = a 2

(C) ÐV 'SV = 90° (D) V 'S'SV is a cyclic quadrilateral


19. Which of the following statement(s) is/are correct ?
(A) Focus of the parabola x2 – 2x – 4y + 9 = 0 is (1, 3)

x 2 y2
(B) Sum of the focal radii of any point P on the ellipse + = 1 , is 6
25 9
(C) Locus of the point P which moves such that distance of P from (1, 2) is equal to twice the distance of
P from the line 2x + y – 4 = 0, is hyperbola.

1
(D) Eccentricity of the ellipse 2x2 + 3y2 = 6 is
3

20. Let PSQ be a focal chord of parabola y 2 = 8x . If focal distance of one end P of this focal chord is 10 units
and coordinates of other end Q are (a, b) then (a, b) can be.
(A) a = 1/2, b = –2 (B) a = 2, b = –1/2 (C) a = 1/2, b = 2 (D) a = –2, b = 1/2
21. The equation y2 + 3 = 2(2x + y) represents a parabola with the vertex at :

æ1 ö æ 1ö
(A) ç ,1÷ and axis parallel to x-axis (B) ç1, ÷ and axis parallel to x-axis
è2 ø è 2ø

æ1 ö æ3 ö æ1 ö
(C) ç ,1÷ and focus at ç 2 ,1÷ (D) ç ,1÷ and axis parallel to y-axis
è2 ø è ø è2 ø
22. The point of contact of 5x + 12y = 9 and x2 – 9y2 = 9 will lie on
(A) 4x + 15y = 0 (B) 7x + 12y = 19 (C) 4x + 15y + 1 = 0 (D) 7x – 12y = 19
23. If a variable tangent of the circle x2 + y2 = 1 intersects the ellipse x2 + 2y2 = 4 at P and Q, then the locus of
the points of intersection of tangents of P and Q is
(A) a circle of radius 2 units (B) a parabola with focus as (2, 3)

3
(C) an ellipse with eccentricity (D) an ellipse with length of latus rectum is 2 units
2
INTEGER ANSWER TYPE

(x - 3) 2 (y + 4) 2
24. Given the equation of the ellipse + = 1, a parabola is such that its vertex is the lowest point
16 49
of the ellipes and it passes through the ends of the minor axis of the ellipse. the equation of the parabola is in
the form 16y = a (x –h)2 – k. Determine the value of (a + h + k).

MATHEMATICS /AQ # 09 E-34


TM TARGET: JEE (MAIN + ADVANCED) 2020
ENTHUSIAST & LEADER
Path to success KOTA (RAJASTHAN)
COURSE
MATCH THE COLUMN :

(12x - 5y + 3) 2
25. Consider the parabola (x – 1)2 + (y – 2)2 =
169
Column-I Column-II
(A) Locus of point of intersection of perpendicular tangent (p) 12x – 5y – 2 = 0
(B) Locus of foot of perpendicular from focus upon any tangent (q) 5x + 12y – 29 = 0
(C) Line along which minimum length of focal chord occurs (r) 12x – 5y + 3 = 0
(D) Line about which parabola is symmetrical (s) 24x – 10y + 1 = 0
26. Column-I Column-II
(A) Distance between the points on the curve 4x 2 + 9y 2 = 1 , where (p) 1
tangent is parallel to the line 8x = 9y, is less than
(B) The distance between the foci of the curve (q) 4
25(x + 1) 2 + 9(y + 2) 2 = 225 is more than
(C) Sum of distances from the x-axis of the points on the ellipse (r) 7

x 2 y2
+ =1, where the normal is parallel to the line 2x + y = 1
9 4
is less than
(D) Tangents are drawn from points on the line x – y + 2 = 0 to the (s) 5
ellipse x 2 + 2y 2 = 2 , then all the chords of contact pass through
the point whose distance from (2, 1/2) is more than

ANSWER KEY
1. (B) 2. (C) 3. (B) 4. (B) 5. (C) 6. (B) 7. (A)
8. (A) 9. (C) 10. (A) 11. (A) 12. (C) 13. (D) 14. (D)
15 (C) 16 (B) 17 (A) 18. (A,C,D) 19. (A,D) 20. (A, C)
21. (A,C) 22. (A,B) 23. (C,D) 24. [0186] 25. (A – r, B – S. C – p, D–q)
26. (A – p, q, r, s; B – p, q, r, s; C – q, r, s; D – p)
MATHEMATICS /AQ # 09 E-35
TM TM TARGET:JEE
TARGET: JEE(Main
(MAIN++Advanced)
ADVANCED)2020
2020
ENTHUSIAST & LEADER
Path to success KOTA (RAJASTHAN) ENTHUSIAST & LEADER
COURSE
Path to success KOTA (RAJASTHAN ) COURSE

ADVANCED QUIZ # 10 LOG AND QUADRATIC EQUATION MATHEMATICS


SINGLE CORRECT CHOICE TYPE
1. If a, b, c and d are non-zero real numbers such that c and d are the roots of x 2 + ax + b = 0 and a and b
are the roots of x 2 + cx + d = 0 then the value of (a + 2b + 3c + 4d) is
(A) –1 (B) –3/2 (C) –5 (D) –8
2. If min. (2x2 – ax + 2) > max.(b – 1 + 2x – x2) then roots of the equation 2x2 + ax + (2–b) = 0, are
(A) positive and distinct (B) Negative and distinct
(C) opposite in sign (D) Complex roots
3. The number of integral values of a for which the inequality x2 – 2(4a –1)x + 15a2 > 2a + 7 is true for every
x Î R, is
(A) 0 (B) 1 (C) 2 (D) 3
4. For a, b, c Î R and b2 ³ 4ac, if all the roots of the equation ax4 + bx2 + c = 0 are real, then
(A) b > 0, a < 0, c > 0 (B) b < 0, a > 0, c > 0 (C) b > 0, a > 0, c > 0 (D) b > 0, a < 0, c > 0
5. The smallest integral value of p such that px2 + 12x + 6 > 3x2 – p "x Î R , is
(A) 5 (B) 6 (C) 7 (D) 8

1
6. Suppose n be an integer greater than 1, let a n = log 2002 . Suppose b = a 2 + a 3 + a 4 + a 5 and
n

c = a10 + a11 + a12 + a13 + a14 . Then (b – c) equals

1 1
(A) (B) (C) –1 (D) –2
1001 1002
7. Find complete set of all negative values of ‘a’ which makes the quadratic inequality
sin 2 x + a cos x + a 2 ³ 1 + cos x true for every x Î R .
(A) a Î ( -¥, -2] (B) a Î ( -¥, -3) (C) a Î ( -¥, -4) (D) None of these
8. The real roots a, b of the equation px2 + qx + r = 0 are such that –3 < a < 1 and b > 7. Then the real roots
g, d of the equation 4px2 + x (12p + 2q) + 9p + 3q + r = 0 satisfy the condition (d > g)
(A) –3 < g < –1, d > 2 (B) –3 < g < 5, d > 17 (C) 3 < g < 5, d > 7 (D) –3 < g < 7, d >17

9. If the roots a, b of the equation px 2 + qx + r = 0 are real and of opposite sign (where p,q,r are real

coefficient), then the roots of the equation a ( x - b ) + b ( x - a ) = 0 are


2 2

(A) positive (B) negative


(C) real and of opposite sign (D) imaginary

B
10. If log4A = log6 B = log9 (A + B) then equals
A
(A) 2 sin 54° (B) 2 cos 72° (C) 2 sin 15° (D) 2 cos 15°

MATHEMATICS /AQ # 10 E-36


TM TARGET: JEE (MAIN + ADVANCED) 2020
ENTHUSIAST & LEADER
Path to success KOTA (RAJASTHAN)
COURSE
1- a -b
11. If 60a = 3 and 60b = 5 then the value of equals
12 2(1-b )
(A) 2 (B) 3 (C) 3 (D) 12
12. The polynomial p(x) = x3 + ax2 + bx + c has the property that the mean of its zeros, the product of its zeroes,
and the sum if its coefficients are all equal. If the y-intercept of the graph of y = p(x) is 2, then the value of b
is
(A) –11 (B) –9 (B) –7 (C) 5
13. Number of integral value (s) of x satisfying the equation log 2(3 – x) +

æ 9p ö
ç sin 4 ÷ 11p
log 1 ç ÷ = cos - log 1 (x + 7)
is
2 ç 5-x ÷ 3 2
è ø

(A) One (B) two (C) three (D) Four

14. If log24, log 2 8 and log 3 9k -1 are consecutive terms of a geometric sequence, then the number of integers
that satisfy the system of inequalities x2 – x > 6 and |x| < k2 is
(A) 193 (B) 194 (C) 195 (D) 196
15. If a, b be the roots of 4x – 16x + l = 0, l Î R . Such that 1 < a < 2 and 2 < b < 3, then the number of
2

integral value of l is
(A) 5 (B) 6 (C) 2 (D) 3
16. If a and b are the distinct real roots of the equation ax + bx + c = 0 and a and b are also the distinct real
2 4 4

roots of the equation lx 2 + mx + n = 0, then the roots of the equation a 2 l x 2 - 4ac lx + 2c 2l + a 2 m = 0


are (all coefficients are real)
(A) always positive (B) always non - real (C) opposite in sign (D) negative
PASSAGE TYPE QUESTION
PARAGRAPH FOR QUESTION NO. 17 TO 19
Let f(x) = x2 + bx + c "x Î R(b, c Î R) attains its least value at x = –1 and the graph of f(x) cuts y-axis at y = 2
17. The least value of f(x) "x Î R is
(A) –1 (B) 0 (C) 1 (D) 3/2
18. The value of f(–2) + f(0) + f(1) =
(A) 3 (B) 5 (C) 7 (D) 9
19. If f(x) = a has two distinct real roots,then complete set of values of a is
(A) (1, ¥ ) (B) (-2, -1) (C) (0,1) (D) (1, 2)
MULTIPLE CORRECT CHOICE TYPE

20. If the quadratic equation ( ab - bc ) x 2 + ( bc - ca ) x + ca - ab = 0 where a, b, c Î R has both the roots equal
then
(A) both roots are equal to 0 (B) both roots are equal to 1
(C) a, c, b are in harmonic progression (D) ab 2 c2 , b 2 a 2 c, a 2 c2 b are in arithmetic progression

MATHEMATICS /AQ # 10 E-37


TM TARGET: JEE (MAIN + ADVANCED) 2020
ENTHUSIAST & LEADER
Path to success KOTA (RAJASTHAN)
COURSE

1
If x + = 14 ( x > 0) then
2
21.
x2
(A) x3 + x–3 = 62 (B) x3 + x–3 = 52 (C) x5 + x–5 = 624 (D) x5 + x–5 = 724
INTEGER ANSWER TYPE

Find the sum of all integral values of x in interval [–4, 100] satisfying 2x - (2x - 1) = 1.
2
22.

7
æ log |x - 2|
æ 1ö
ç ÷ log7 3(
|x - 2|-9

23. Find the sum of possible real values of x for which the sixth term of çç 3
3 9
+7 è 5ø
÷ equals 567
÷
è ø
24. If 5x2 – 2kx + 1 < 0 has exactly one integral solution then find the sum of all positive integral values of k.
25. Find the number of integral values of a so that the inequation x2 – 2(a + 1)x + 3(a – 3) (a + 1) < 0 is satisfied
by atleast one x Î R+
26. If x is real, then the maximum value of (3x 2 + 9x + 17) /(3x 2 + 9x + 7) is
MATCH THE COLUMN :

æ 4-x ö
27. For the equation 1 + log x ç ÷ = (log10 (log10 p) - 1) log x 10 , match the value of p given in column-II
è 10 ø
corresponding to the number of roots given in clumn-I.
Column-I Column-II
(A) for no root (p) p Î {103}

(B) for exactly one real root (q) p Î {104 }

(C) for two distinct real roots (r) p Î (1,104 ) - {103}

(s) p Î (104 , ¥ )

ANSWER KEY
1. (D) 2. (D) 3. (B) 4. (B) 5. (C) 6. (C) 7. (A)
8. (A) 9. (C) 10. (A) 11. (A) 12. (A) 13. (A) 14. (A)
15. (D) 16. (C) 17. (C) 118. (D) 19. (A) 20. (BCD) 21. (BD)
22. [5050] 23. [0004] 24. [0009] 25. [0005] 26. [0041] 27. [A–s, B–p,q, C–r]
MATHEMATICS /AQ # 10 E-38
TM TM TARGET:JEE
TARGET: JEE(Main
(MAIN++Advanced)
ADVANCED)2020
2020
ENTHUSIAST & LEADER
Path to success KOTA (RAJASTHAN) ENTHUSIAST & LEADER
COURSE
Path to success KOTA (RAJASTHAN ) COURSE

ADVANCED QUIZ # 11 PROGRESSION & BINOMIAL THEOREM MATHEMATICS


SINGLE CORRECT CHOICE TYPE
¥
1 a+ b
1. If the sum å = where a, b, c Î N and lie in [1, 15] then a + b + c equals
( k + 2) k =1 k +k k+2 c

(A) 6 (B) 8 (C) 10 (D) 11


2. Consider the sequence 8A + 2B, 6A + B,4A, 2A – B, ..... Which term of this sequence will have a
coefficient of A which is twice the coefficient of B ?
(A) 10th (B) 14th (C) 17th (D) none
n =¥
ænö
The value of å (-1)
n +1
3. ç 5n ÷ equal
n =1 è ø

5 5 5 5
(A) (B) (C) (D)
12 24 36 16
4. The 5th and 8th terms of a geometric sequence of real number are 7! and 8! respectively. If the sum of first
n terms of the geometric progression is 2205, then n =
(A) 3 (B) 4 (C) 5 (D) 6
5. If the roots of the equation x – 12x + bx + cx + 81 = 0 are positive then values of b and c respectively are
4 3 2

(A) 54, 108 (B) – 54, 108 (C) 54, – 108 (D) None of these
6. In the expansion of (1 + x + x 2 + ... + x 27 ) (1 + x + x 2 + .... + x14 ) 2 , the coefficient of x 28 is
(A) 195 (B) 224 (C) 378 (D) 405

32 20 33 2a - 1 æbö
If 3 . C 0 + . C1 + . 20 C2 + ........ upto 21 terms =
1 20
7. , then the value of ç ÷ is equal to
2 3 b èaø

1 1
(A) (B) 2 (C) (D) 3
2 3

Let f (n) = å k ( Ck ) then the value of f(5) equals


n 2
2 n
8.
k =1

(A) 1000 (B) 1250 (C) 1750 (D) 2500


9. The last digit in the number 843843 + 492295 is
(A) 0 (B) 1 (C) 3 (D) 5
10. What is unit digit for the sum (346)44 + (344)45
(A) 0 (B) 1 (C) 2 (D) 3
11. If (1 + ax + bx2) (1 – 2x)18 be expressed in ascending powers of x such that co-efficients of x3, x4 are zero,
then (a, b) is equal to

æ 272 ö æ 44 ö æ 1 88 ö
(A) ç16, (B) ç 4, ÷ (C) ç , ÷
3 ÷ø
(D) None of these
è è 3 ø è 16 3 ø

MATHEMATICS /AQ # 11 E-39


TM TARGET: JEE (MAIN + ADVANCED) 2020
ENTHUSIAST & LEADER
Path to success KOTA (RAJASTHAN)
COURSE

100 C
100 r ×
å r
12. 100 equals
r =1 C
r -1

(A) 100 (B) 4950 (C) 5050 (D) 5151


13. The coefficient of a2 b3 c4 in (3a + b – c)9 is
(A) 324 (B) 756 (C) 1260 (D) 11340
PASSAGE TYPE QUESTION
PARAGRAPH FOR QUESTION NOS. 14 TO 16

Consider the binomial expansion expansion R = (1 + 2 x )n = I + f , where I is the integral part of R and ‘f’ is
the fractional part of R, n Î N. Also the sum of the coefficients of R is 6561.
1
14. The value of (n + R – Rf) for x = equals
2
(A) 7 (B) 8 (C) 9 (D) 10
1
15. If ith terms is the greatest term for x = , then ‘i’ equals
2
(A) 4 (B) 5 (C) 6 (D) 7
16. If k th terms is having greatest coefficient then sum of all possible value (s) of k is
(A) 6 (B) 7 (C) 11 (D) 13
MULTIPLE CORRECT CHOICE TYPE
17. If the sum of first 100 terms of an arithmetic progression is –1 and the sum of the even terms lying in first 100
terms is 1, then which of the following statement (s) is (are) correct ?
-3
(A) Common difference of the arithmetic progression is
50

149
(B) First term of the arithmetic progression is
50

74
(C) 100th term of the arithmetic progression is
25

47
(D) Sum of an infinite geometric progression whose first term is and common ratio is common differ-
25
ence of arithmetic sequence, equals 2
18. If 9th, 13th and 15th terms of arithmetical progression are the first three terms of a geometric series whose
sum of infinite terms is 80, then which of the following hold(s) good ?
(A) Sum of the first 16 terms of the geometric progression is 860
(B) First term of the arithmetic progression is 80
(C) First term of the geometric progression is 40
(D) If d is the common difference of arithmetic progression and r is the common ratio of geometric progres-
-5
sion then dr = .
2
MATHEMATICS /AQ # 11 E-40
TM TARGET: JEE (MAIN + ADVANCED) 2020
ENTHUSIAST & LEADER
Path to success KOTA (RAJASTHAN)
COURSE
19. Difference between the sum of the squares of the first fifty even natural numbers and the sum of the square
of the first fifty odd natural numbers is equal to

x
(A) The value of the expression y = 1 + x 1 + ( x + 1) 1 + ( x + 2)( x + 4) where x = 100
2
(B) f (100) where f (1) = 1 and f (x) = x + f (x – 1)
(C) Sum of all possible integers between 50 and 350 whose digit at the unit place is 1.
(D) Sum of the reciprocals of all the 100 harmonic means if these are inserted between 1 and
1/100
n n n
æ n ö æ n öæ n ö
ænö
20. å å å ç i ÷ ç j ÷ç k ÷ , ç ÷ = n Cr
i =0 j=0 k =0 è ø è øè ø èr ø
(A) is less than 500 if n =3 (B) is greater than 600 if n = 3
(C) is less than 5000 if n = 4 (D) is greater than 4000 if n = 4
n
21. Consider the binomial expansion of æç x + 1 ö÷ , n Î N where the terms of the expansion are written in
è 2. 4 x ø
decreasing powers of x. If the coefficients of the first three terms form an arithmetic progression then the
statement(s) which hold good is/are
(A) total number of terms in the expansion of the binomial is 8
(B) number of terms in the expansion with integral power of x is 3
(C) there is no term in the expansion which is independent of x
(D) fourth and fifth are the midle terms of the expansion
22. Ratio of the product of first n odd natural numbers to the product of first n even numbers can be written as

n 2

n
æ 2r - 1 ö n
æn+r ö n
æ n Cr ö
2
å( n
Cr )
(A) Õ çè 2r ø
÷ (B) Õ ç 4r ÷
r =1 è ø
(C) åç n ÷ (D)
r =0
2
r =1 r =0 è 2 ø æ n n ö
ç å Cr ÷
è r =0 ø

23. Let a and b be the coefficient of x3 in (1 + x + 2x2 +3x3)3 & (1 + x + 2x2 +3x3 +4x4)3 , respectively then
(A) a = b (B) a > b (C) a < b (D) a + b = 44

10
æ 2 1 ö
24. The value of the constant term in the trinomial ç x + 2 - 2 ÷ is equal to
è x ø

(11)(12)(13)..........(20)
(A) (10 C 0 ) 2 + (10 C1 ) 2 + (10 C 2 ) 2 + ....... + (10 C10 ) 2 (B)
(1)(2)(3)......(10)

(2)(6)(10)........(38) 1 20
(C) (D) P10
(1)(2)(3).......(10) 10!

MATHEMATICS /AQ # 11 E-41


TM TARGET: JEE (MAIN + ADVANCED) 2020
ENTHUSIAST & LEADER
Path to success KOTA (RAJASTHAN)
COURSE
25. If 100 C 6 + 4100 C7 + 6100 C8 + 4100 C9 +100 C10 has the value equal to xCy, then the value (x + y) can be
(A) 114 (B) 115 (C) 198 (D) 199
26. If (1 + x) = C0 + C1x + ........ + Cnx , where n is positive number, then
n n

æ np ö æ np ö
(A) C0 – C2 + C4 ...... = 2n/2 cos ç ÷ (B) C1 – C3 + C5 ...... = 2n/2 sin ç ÷
è 4 ø è 4 ø

(C) C0 + C4 + C8 +...... = 2n–2 + 2(n – 2)/2 cos æç np ö÷ (D) None of these


è 4 ø
27. In the expansion of (x + y + z)20
(A) coefficient of x7x8z7 is zero
(B) total number of distinct terms is 231

20!
(C) every term is of the form x 20- r y r - k z k
(20 - r)!(r - k)!k!
(D) None of the above
28. If n is a positive integer and (5 5 + 11) 2n +1 = I + f where I is integer and 0 < f < 1 then
(A) I is an even integer (B) (I + f)2 is divisible by 22n + 1
(C) I is divisible by 22 (D) None of these
INTEGER ANSWER TYPE
100
æ k ö 100 a(2100 ) + b
29. If å ç ÷ C k = where a, b, c Î N then find the least value of (a + b + c).
k =0 è k + 1 ø c

15
æ 1 1 ö
The number of distinct terms in the expansion of ç x + + x + 2 ÷ is/are
2
30.
è x x ø
31. The absolute coefficient of x5 in the expansion of (x2 – x – 2)5 is
32. Find the coefficient of x2009 in the expansion of (1 – x)2008 (1+ x + x2)2007
MATCH THE COLUMN :
33. Column I Column II
(A) Number of distinct terms in the expansion of (x + y – z)20 is (p) 5
(B) Number of terms in the expansion of (q) 2 15

(x + ) ( )
8 8
x2 -1 + x - x2 -1 is

( )
100
(C) The number of irrational terms in 8
5+6 2 is (r) 231

(D) The sum of numerical coefficients in the (s) 98


15
æ x 2y ö
expansion of ç1 + + ÷ is (t) 97
è 3 3 ø

MATHEMATICS /AQ # 11 E-42


TM TARGET: JEE (MAIN + ADVANCED) 2020
ENTHUSIAST & LEADER
Path to success KOTA (RAJASTHAN)
COURSE
34. Column - I Column - II
(A) The harmonic mean of the roots of the equation (p) 2

(5 + 2)x 2 - (4 + 5)x + 8 + 2 5 = 0 is

(B) Let a1 ,a 2 ,....,a10 , be in A.P. and h1 , h 2 ,...., h10 be in H.P.. (q) 3

If a1 = h1 = 2& a10 = h10 = 3 then a 4 h 7 is


(C) The number of integeral values of m, for which the x coordinate (r) 4
of the point of intersection of the lines 3x + 4y = 9 and y = mx+1
is also an integer, is
(D) Between 2 and 5 six geometric means are inserted. If their product (s) 6
can be expressed as (10)n then the value of n equals

ANSWER KEY
1. (D) 2. (D) 3. (C) 4. (A) 5. (C) 6. (B) 7. (A)
8. (C) 9. (D) 10. (A) 11. (A) 12. (C) 13. (D) 14. (C)
15. (B) 16. (D) 17. (CD) 18. (BCD) 19. (ABD) 20. (CD) 21. (BC)
22. (ABCD) 23. (A D) 24. (A BCD) 25. (AC)
26. (ABC) 27. (ABC) 28. (ABC) 29. [0201] 30. [0061] 31. [0081] 32. [0000]
33. (A – r ; B – p; C – t; D – q) 34. [A – r ; B– s ; C–p ; D–q ]

MATHEMATICS /AQ # 11 E-43


TM TM TARGET:JEE
TARGET: JEE(Main
(MAIN++Advanced)
ADVANCED)2020
2020
ENTHUSIAST & LEADER
Path to success KOTA (RAJASTHAN) ENTHUSIAST & LEADER
COURSE
Path to success KOTA (RAJASTHAN ) COURSE

ADVANCED QUIZ # 12 COMP LEX NUMBER MATHEMATICS

SINGLE CORRECT CHOICE TYPE


1. Modulus of non zero complex number z, satisfying z + z = 0 and | z |2 -4zi = z 2 is
(A) 1 (B) 2 (C) 3 (D) 4
2. Number of complex numbers satisfying the relation | z + z | + | z - z | = 2 and |z + i| +|z – i| = 2, is
(A) 1 (B) 2 (C) 3 (D) 4

æ1ö 1
3. Let z = x + iy, where x, y Î R and i = -1 . If locus of P(z) satisfying Re ç ÷ = represents a circle then
èzø 2
maximum distance of a point on the circle from M(–2, 4), is equal to
[Note : Re(z) denotes the real part of z]
(A) 4 (B) 5 (C) 6 (D) 8
4. Let a is a complex number, satisfying ia 3 + a 2 - a + i = 0 then maximum value of | a – 3 – 4i| is
(A) 2 (B) 4 (C) 5 (D) None of these
30 30
1
If a1, a2, a3,..........., a30 are the roots of the equation å x = 0 , the value of å
k
5. is
k =0 i =1 (a i - 1)

(A) 30 (B) –30 (C) 15 (D) –15

æ 2z 2 , 5z ∗ 3 ö÷ ο
ç ÷ = is
In the Argand plane the locus of z ¹ 1 such that arg çç 2
è 3z , z , 2 ø÷÷ 2
6.

,3 ,2
(A) the straight line joining the points z = and z =
2 3

3 2
(B) the straight line joining the points z = and z =
2 3

,3 2
(C) A segment of a circle passing through z = and z =
2 3

3 -2
(D) Locus of z is a semicircle described on the line segment joining z = and z =
2 3

p p
7. If a = cos + i sin , then |1 + a + a2 + .................+ an –1| is ( n Î N )
2n 2n

p p p p p p p p
(A) sin .sin , (B) sin .cos , (C) cos sec (D) cos .cosec
4 4 4 4n 4 4n 4 4n

MATHEMATICS /AQ # 12 E-44


TM TARGET: JEE (MAIN + ADVANCED) 2020
ENTHUSIAST & LEADER
Path to success KOTA (RAJASTHAN)
COURSE
8. The smallest value of |z|2 + |z – 6|2 + |z – 9i|2, where z is a complex number is
117
(A) 117 (B) 78 (C) 39 (D)
2
9. All the four real complex roots of the equation x – 2x + 4 = 0 are the vertices of an inscribed
4 2

(A) square in a circle of radius 2 (B) rectangle in a circle of radius 2


(C) Square in a circle of radius 2 (D) rectangle in a circle of radius 2
n -1
1
10. If 1, a, a2, ........., an – 1 are the nth roots of units, then å k is equal to
k =1 2 - a

(n - 2)2n -1 + 1 (n - 2)2 n -1
(A) (n – 2)2n (B) (C) (D) None of these
2n - 1 2n - 1

éwr w r +1 w r+ 2 ù
ê ú
11. If A(r) = ê w r -1 wr w r +1 ú where w is complex cube root of unity, then
ê w 2r w 2r + 2 w 2r + 4 úû
ë
(A) A (r) is singular only if r is even (B) A(r) is singular only if r is odd
(C) A(r) is singular (D) A(r) is non-singular
12. Let w, w are complex cube root of unity and P (z) is any point on the circle | z | = 4 such that
| z – 1| is maximum and centroid of triangle formed by z, – w and – w is a, then Re ( a ) is

2
(A) –2 (B) 2 (C) –1 (D) -
3
13. The reflection of the complex number 2 – i in the straight line iz = z is
(A) 4 – 3i (B) 3 + 4i (C) 2 + i (D) 1– 2i
14. If (z + i) – (z – i) = 0, then which of the following is correct?
3 3

(A) all roots are real (B) all roots are complex
(C) exactly one roots is real (D) exactly one root is complex

( -1 + i 3 ) + ( -1 - i 3 )
30 30

15. The value of is


(1 - i) 20 (1 - i) 20
(A) 221 (B) – 221 (C) 1 (D) 1 - 3
PASSAGE TYPE QUESTION
PARAGRAPH FOR QUESTION NOS. 16 TO 18
Consider a conic in the Argand plane which has A(z1), B(z2) as its foci. Also, the origin is a point on the
conic.
16. If the conic is a hyperbola and z1 = 3 + 4i and z2 = 5 + 12i, then its eccentricity is

17 17 9 16
(A) (B) (C) (D)
4 3 17 15
17. If the conic is an ellipse and z1 = 3 + 4i and z2 = 5 + 12i then its eccentricity is
17 4 3 15
(A) (B) (C) (D)
9 17 17 16
MATHEMATICS /AQ # 12 E-45
TM TARGET: JEE (MAIN + ADVANCED) 2020
ENTHUSIAST & LEADER
Path to success KOTA (RAJASTHAN)
COURSE
18. If the origin is outside the ellipse given in above question then

17 4 17 17 4
(A) 0 < e < (B) < e <1 (C) < e <1 (D) <e<
9 17 9 9 17
MULTIPLE CORRECT CHOICE TYPE
(1 - z) 2
19. If | z | = 1 and let w = , then locus of w is equivalent to
1 - z2
(A) | z – 2 – 4i| = | z – 2 + 4i| (B) | z – 3 + 4i| = | z + 3 + 4i|
(C) | z – 2| = | z + 2| (D) | |z – i| – |z + i || = 2
m
é ì æ 2pk ö
4n +1 m -1
æ 2 pk ö üù
20. The sum å êå ísin ç ÷ - i cos ç ÷ ýú is
m =1 ë k =1 î è m ø è m ø þû
(A) independent of n (B) purely imaginary (C) purely real (D) a root of x 4n + 1 + 1 = 0
21. If iz3 + 3z 2 - z + 3i = 0, then | z | can be
(A) 1 (B) 2 (C) 3 (D) 4
INTEGER ANSWER TYPE
22. ABCD is a rhombus. Its diagonals AC and BD intersect at the point M and satisfying BD = 2AC. If the
points D and M represent the complex numbers 1 + i and 2 - i respectively and if (x1 + iy1) and (x2 + iy2) are
x1x 2
the complex number of the points A and C, then find the value of y y
l 2

z +1
23. Let | z | = 2 and w = where z, w Î C (where C is the set of complex numbers). If M and m
z -1
respectively be the greatest and least modulus of w, then find the value of (2010m + M).
24. If z1 , z 2 Î C, z12 + z 22 Î R, z1 (z12 - 3z 22 ) = 2 and z 2 (3z12 - z 22 ) = 11 , then the value of z12 + z 22 is
MATCH THE COLUMN :
25. Match the equation in z, in Column - I with the corresponding values of arg (z) in Column - II.
Column - I Column - II
2p
(A) z 2 - z + 1 = 0 (p) –
3
p
(B) z 2 + z + 1 = 0 (q) -
3
p
(C) 2 z 2 + 1 + i 3 = 0 (r)
3
2p
(D) 2 z 2 + 1 - i 3 = 0 (s)
3

ANSWER KEY
1. (B) 2. (B) 3. (C) 4. (D) 5. (D) 6. (D) 7. (D)
8. (B) 9. (D) 10. (B) 11. (C) 12. (C) 13. (D) 14. (A)
15. (B) 16. (A) 17. (A) 18. (C) 19. (B, C) 20. (A, B) 21. (A, C)
22. [0004] 23. [0673] 24. [0005] 25. [A–q,r ; B–p,s ; C–q,s ; D–p,r]
MATHEMATICS /AQ # 12 E-46
TM TM TARGET:JEE
TARGET: JEE(Main
(MAIN++Advanced)
ADVANCED)2020
2020
ENTHUSIAST & LEADER
Path to success KOTA (RAJASTHAN) ENTHUSIAST & LEADER
COURSE
Path to success KOTA (RAJASTHAN ) COURSE

ADVANCED QUIZ # 13 PERMUATION, COMBINATION & PROBABILITY PART -1 MATHEMATICS

SINGLE CORRECT CHOICE TYPE


1. A fair coin is flipped n times. Let E be the event “a head is obtained on the first flip”, and let F k be the event
“exactly k heads are obtained” . For which one of the following pairs (n, k) such that E and Fk independent?
(A) (12, 4) (B) (20,10) (C) (40, 10) (D) (100, 51)
2. An urn contains3 red balls and n white balls. Mr. A draws two balls together from the urn. The probability
that they have the same colour is1/2. Mr. B draws one balls from the urn, notes its colour and replaces it. He
then draws a second ball from the urn and finds that both balls have the same colour is, 5/8. The possible
value of n is
(A) 9 (B) 6 (C) 5 (D) None of these
3. If three fair dice are rolled, the probability that none will show six and all will shows different numbers, is

5 125 5 1
(A) (B) (C) (D)
18 216 9 8
4. Number of ways in which 15 indistinguishable oranges can be distributed in 3 different boxes so that every
box has atmost 8 oranges, is
(A) 52 (B) 108 (C) 76 (D) 28
5. There are 7 persons which include a group of 3 friends F1, F2, F3. Numberof ways they can be seated on
a round table if no two out of F1, F2, F3 are seated next to each other is
(A) 144 (B) 576 (C) 36 (D) None
6. Number of 7 digit numbers the sum of whose digits is 61 is :
(A) 12 (B) 24 (C) 28 (D) none
7. A team of four students is to be selected from a total of 12 students. Total number of ways in which team
can be selected such that two particular students refuse to be together and other two particular students
wish to be together only, is equal to
(A) 220 (B) 182 (C) 226 (D) None
8. Among the 8! permutations of the digit 1, 2, 3,.....,8, consider those arrangements which have the following
property. If we take any five consecutive positions, the product of the corresponding digit is divisible by 5.
The number of such arrangements will be equal to
(A) 7! (B) 2 . 7! (C) 7 C4 (D) 5 . 7!
9. In a birthday party, each man shook hands with everyone except his spouse, and no handshakes took place
between women. If 13 married couples attended, how many handshakes were there among these 26
people?
(A) 185 (B) 234 (C) 312 (D) 325
10. If all the letters of the word “HARSHITA” are arranged alphabetically as they are in a dictionary, then
(5050) th word will be
(A) IHAAHSTR (B) IAAHRTHS (C) IAAHRTSH (D) IAAHRSTH
11. A man has nine friends, four boys and five girls. In how many ways can he invite them, if there have to be
exactly three girls in the invitees ?
(A) 80 (B) 200 (C) 320 (D) 160
MATHEMATICS /AQ # 13 E-47
TM TARGET: JEE (MAIN + ADVANCED) 2020
ENTHUSIAST & LEADER
Path to success KOTA (RAJASTHAN)
COURSE
12. The product of all odd positive integers less than 10000, is

(9999)! (10000)! (10000)! (10000)!


(A) (B) (C) (D)
25000 2 × (5000)!
5000
(5000!) 2 25000
PASSAGE TYPE QUESTION
PARAGRAPH FOR QUESTION NO. 13 TO 15
Consider the set S = {0, 1, 2, ........ 9}
13. Number of all five digit numbers that can be formed using the digits from if their digits are in due order is
(A) 126 (B) 252 (C) 310 (D) 378
14. Number of all five digit numbers that can be formed using the digits from S containing 2 alike and 3 other
alike digit, is
(A) 810 (B) 750 (C) 720 (D) None
15. Number of 10-digit prime numbers that can be formed using each and every digit of S, is
(A) 0 (B) 1 (C) 10 (C) 100
PARAGRAPH FOR QUESTION NOS. 16 TO 18
Let S be the set {0, 1, 2, 3, 4 , 5}
16. The number of 3 digit numbers formed using the digits in S so that the digits either increase or decrease is
(A) 24 (B) 30 (C) 45 (D) 56
17. The number of 5 digit numbers divisible by 3 that can be formed using the digits in S without repetition is
(A) 216 (B) 224 (C) 236 (D) 244
18. The number of 6 digit numbers formed using the digits in S without repetition is
(A) 540 (B) 640 (C) 720 (D) 600
INTEGER ANSWER TYPE
19. Sum of all the 3 digit palindromes which are odd, is of the form 10 K where K Î N , then the value of K
equals.
20. Let ‘A’ denotes the number of ways in which 5 people in a lift at ground level can leave the lift of a 6 floor
building at different floors. (Assume that each way only differs by the number of people leaving at different
floors and any number of people can get down at any floor) ‘B’ denotes the number of different terms in the
expantion of (x + 2y + 3z)20.
Find (A + B)
21. In an examination, 5 children were found to have their mobiles in their pocket. The Invigilator fired them and
took their mobiles in his possession. Towards the end of the test, Invigilator randomly returned their mo-
biles. Find the number of ways in which at most two children did not get their own mobiles.
n n
æk ö
22. Let f (n) = åå ç r ÷ . Find the total number of divisors of f (9).
r=0 k = r è ø

23. Let f : A ® B be any function where A is a set containing the positive integral solution of the inequality
cos ec -1 (cos ec2) > x 2 - 3x and B is the set of all divisors of the natural number 2010. If f (i) £ f ( j) " i < j ,
then find the total number of mapping from A to B.
24. Find the number of integral solution of the equation a1 + a2 + a3 + a4 =15,
where a1 ³ 0,a 2 > 5, a 3 ³ 2 and a 4 ³ 1 .
MATHEMATICS /AQ # 13 E-48
TM TARGET: JEE (MAIN + ADVANCED) 2020
ENTHUSIAST & LEADER
Path to success KOTA (RAJASTHAN)
COURSE
25. Find the number of 8 digit numbers a1a2a3a4a5a6a7a8 where all a i 's Î{0, 1} with a1 = 1 and have the prop-
erty a1 + a3 + a5 + a7 = a2 + a4 + a6 + a8
26. Seven different lecturers are to deliver lectures in seven periods of a class on a particular day. A, B and C
are three of the lecturers. The number of ways in which a routine for the day can be made such that A
delivers his lecture before B and B before C, is
MATCH THE COLUMN :
27. Column - I Column - II
(A) Number of all six digit natural numbers such that the sum of their (p) 350
digits is 10 and each of the digits 0,1, 2, 3 occurs atleast once
in them is
(B) A question paper consists of 2 parts A and B. Part-A has 4 (q) 405
questions with 1 alternative each and part-B has 3 question without
any alternative. Number of ways in which one can select the
question when atleast one question must be attempted from (r) 490
each part, is
(C) Number of ordered pairs of positive integers (a, b) such that (s) 560
the least common multiple of ‘a’ and ‘b’ is 22.54.114

ANSWER KEY
1. (B) 2. (D) 3. (A) 4. (A) 5. (A) 6. (C) 7. (C)
8. (B) 9. (B) 10. (D) 11. (D) 12. (B) 13. (D) 14. (A)
15. (A) 16. (B) 17. (A) 18. (D) 19. [2750] 20. [0483] 21. [0011]
22. [0008] 23. [0816] 24. [0084] 25. [0035] 26. [0840] 27. [A–R ; B–S ; C–Q]

MATHEMATICS /AQ # 13 E-49


TM TM TARGET:JEE
TARGET: JEE(Main
(MAIN++Advanced)
ADVANCED)2020
2020
ENTHUSIAST & LEADER
Path to success KOTA (RAJASTHAN) ENTHUSIAST & LEADER
COURSE
Path to success KOTA (RAJASTHAN ) COURSE

ADVANCED QUIZ # 14 PERMUATION, COMBINATION & PROBABILITY PART -2 MATHEMATICS

SINGLE CORRECT CHOICE TYPE


1. The number of ways in which 7 distinct objects can be distributed among 3 persons so that each get at least
2 objects is
(A) 3780 (B) 7860 (C) 630 (D) 360
2. The number of three digit numbers, whose middle digit is a prime number and unit digit is from the set {0, 3,
6, 9}, is
(A) 81 (B) 100 (C) 121 (D) 144
3. The number of numbers greater than a million that can be formed with the digits 4,3,2,2,0,3,3 is
(A) 2160 (B) 360 (C) 300 (D) 420
5. If a,b,c,d are four odd natural numbers such that a + b + c + d = 40, then the number of values of (a,b,c,d) is
(A) 655 (B) 969 (C) 12341 (D) 1330
6. Three boxes are labelled A, B and C and each box contains four balls numbered 1, 2, 3 and 4. The balls in
each box are well mixed. A child chooses one ball at random from each of the three boxes. If a, b, and c are
the numbers on the balls chosen from the boxes A, B and C respectively, the child wins a toy helicopter
when a = b + c. The odds in favour of the child to receive the toy helicopter are
(A) 3 : 32 (B) 3 : 29 (C) 1 : 15 (D) 5 : 59
7. The number of numbers of 9 different non zero digits such that all the digits in the first four places are less
than the digit in the middle and all the digits in the last four places are greater than that in the middle is
(A) 2(4!) (B) (4!)2 (C) 8! (D) None of these
8. All the possible words obtained by rearranging the letters of the word RACE are arranged as in a dictio-
nary. The rank of the word CARE is
(A) 6 (B) 7 (C) 8 (D) 24
9. A forecast is to be made of the result of five cricket matches, each of which can be a win or a draw or a loss
for Indian team.
Let p = number of forecasts with exactly 1 error, q = number of forecasts with exactly 3 errors and r =
number of forecasts with all five errors
then the incorrect statement is :
(a) 8p = 5r (b) 2q = 5r (c) 8p = q (d) 2 (p + r) > q
10. A machine has two connected parts A and B that it uses to function. From previous experience we know
that P(A fails) = 0.2, P(B fails alone) = 0.15, and P(A and B fails) = 0.15. P(neither A nor B fails) is
(A) 0.75 (B) 0.65 (C) 0.55 (D) 0.45
11. There are two groups one of which contains 5 science and 3 engineering subjects and other group contains
3 science and 5 engineering subjects. An unbiased die is thrown. If the number 3 or 5 turns a subject is
chosen from first group otherwise a subject is chosen from second group. The probability of choosing a
science subject is

1 13 11
(A) (B) (C) (D) None of these
2 24 24

MATHEMATICS /AQ # 14 E-50


TM TARGET: JEE (MAIN + ADVANCED) 2020
ENTHUSIAST & LEADER
Path to success KOTA (RAJASTHAN)
COURSE
12. A bag contains p white and q black balls. Two players A and B alternately draw a ball from the bag,
replacing the balls each time after the draw till one of them draws a white ball and wins the game. If A begins
the game and the probability of A winning the game is three times that of B, then the ratio p : q is
(A) 3 : 4 (B) 4 : 3 (C) 2 : 1 (D) 1 : 2
13. Consider the following two statements :
(1) The total number of ways of arranging 10 identical objects into 4 groups, in which there is no blank= x.
(2) The number of selections that can be made from 9 distinct objects by taking ‘3’ at a time without
repetition = y.
(A) x > y (B) x < y (C) x = y (D) x = 2y
14. A bag contains four tickets numbered as 223, 231, 322 and 333. One ticket is drawn at random from the
bag. If E i is the event that the i th digit on the ticket is 3, then

(A) E1 and E 2 are independent (B) E1 and E 2 are mutually exclusive

(C) E 2 and E 3 are mutually exclusive (D) E1 ,E 2 , E 3 are independent


15. Let n Î {1, 2, 3, ......., 1000}. The probability that the remainder when 3 n + 1 is divided by 4 is 1 or 3 is

1 1 2
(A) (B) (C) (D) 0
2 3 3
16. Find the number of non negative solution of 2x + y + z = 21
(A) 42 (B) 123 (C) 132 (D) 86

PASSAGE TYPE QUESTION


PARAGRAPH FOR QUESTION NOS. 17 TO 19
An objective test contains two sections : A and B, each consisting of 10 questions. In section A, only one
choice out of 4 choices is correct and student is awarded 1 mark for every correct answer. In section B,
one or more than one choice is (are) correct out of 4 choices and the student is awarded 3 marks if he (she)
ticks only the correct choices and all the correct choices. There is no negative marking.
17. In how many ways can a student answer to any question of section B
(A) 11 (B) 9 (C) 4 (D) 15
18. If a student attempts 3 particular questions- one from section A and two from section B, the probability that
he will get marks in only two questions is (assuming all ways to answer a question to be equally likely)
31 23 23
(A) (B) (C) (D) None of these
900 900 484
19. The probability that a student gets 10 marks if he attempts only 4 questions is
3 3 3
1æ 1 ö 4æ 1 ö 1æ1ö
(A) ç ÷ (B) ç ÷ (C) ç ÷ (D) None of these
5 è 15 ø 5 è 11 ø 4è9ø

MATHEMATICS /AQ # 14 E-51


TM TARGET: JEE (MAIN + ADVANCED) 2020
ENTHUSIAST & LEADER
Path to success KOTA (RAJASTHAN)
COURSE
MULTIPLE CORRECT CHOICE TYPE
20. Number of ways in which n distinct things can be distributed to 3 children if each receiving none, one or
more number of things, is NOT equal to
(A) the number of ways of all possible selections of one or more questions from n given questions, each
question having an alternative.

(B) the sum of all coefficients in the expansion of the binomial (2p + q) n .

(C) number of n digit numbers (containing at leastone odd digit) that can be written, if each digit of the
number is selected from the set {1, 2,3, 4, 5, 6}.
(D) number of different signals that can be transmitted by making use of 3 different coloured flags keeping
one above the other, if n different flags are available.
21. There are 10 questions, each question is either True or False. Number of different sequences of incorrect
answers is also equal to
(A) Number of ways in which a normal coin tossed 10 times would fall in a definite order if both Heads and
Tails are present.
(B) Numbers of ways in which a multiple choice question containing 10 atternatives with one or more than
one correct alternatives, can be answered.
(C) Number of ways in which it is possible to draw a sum of money with 10 coins of different denominations
taken some or all at a time.
(D) Number of different selections of 10 indistinguishable thing taken some or all at a time.

22. The combinatorial coefficients n-1 C p denotes

(A) the number of ways in which n things of which p are alike and rest are different can be arranged in a
circle.
(B) the number of ways in which p different things can be selected out of n different thing if a particular thing
is always excluded.
(C) number of ways in which n alike balls can be distributed in p different boxes so that no box remains
empty and each box can hold any number of balls.
(D) the number of ways in which (n – 2) white balls and p black balls can be arranged in a line if black balls
are separated, balls are all alike except for the colour.
23. Which of the following statement (s) is / are true ?

(A) 100 C50 is not divisible by 10.

(B) n (n – 1)(n – 2) ...... (n – r + 1) is always divisible by r! (n Î N and 0 £ r £ n) .


(C) Morse telegraph has 5 arms and each arm moves on 6 different positions including the position of rest.
Number of different signals that can be transmitted is 56 – 1.
(D) A shopkeeper places before you 5 different books each having 5 copies. Number of different selections
is 65 – 1.

MATHEMATICS /AQ # 14 E-52


TM TARGET: JEE (MAIN + ADVANCED) 2020
ENTHUSIAST & LEADER
Path to success KOTA (RAJASTHAN)
COURSE
INTEGER ANSWER TYPE
24. The maximum number of points into which 6 circles and 4 straight lines intersect, is
25. The total numbers of six-digit natural numbers that can be made with the digits 1, 2, 3, 4, if all digit are to
appear in the same number at least once is
26. The letters of word ‘ZENITH’ are written in all possible ways. If all these words are written in the order of
a dictionary, then the rank of the word ‘ZENITH’ is
27. A team of four students is to be selected from a total of 12 students. The total number of ways in which the
team can be selected such that two particular students refuse to be together and other two particular
students wish to be together only is equal to
28. Find the number of ways in which the letters of the word ‘KUTKUT’ can be arranged so that no two alike
letters are together.
29. The number of even divisors of the numbers N = 12600 = 23 32 52 7 is

MATCH THE COLUMN :


30. Column-I Column-II
(A)Number of increasing permutations of m symbols are there from the (p) nm

n set numbers {a1 , a2 ,....an } where the order among the numbers is

given by a1 < a2 < a3 < ....an -1 < an is

(B) There are m men and n monkeys. Number of ways in which every monkey (q) m Cn
has a master, if a man can have any number of monkeys

(C)Number of ways in which n red balls and (m – 1) green (r) n Cm


balls can be arranged in a line, so that no two red balls
are together is, (balls of the same colour are alike)
(D)Number of ways in which ‘m’ different toys can be distributed (s) mn
in ‘n’ children if every child may receive any number of toys, is

ANSWER KEY
1. (C) 2. (D) 3. (B) 4. (D) 5. (B) 6. (B) 7. (C)
8. (A) 9. (B) 10. (C) 11. (C) 12. (C) 13. (A) 14. (D)
15. (C) 16. (D) 17. (A) 18. (D) 19. (*) 20. (A,C,D)
21. (B,C) 22. (B, D) 23. (A, B, D) 24. [0084] 25. [1560]
26. [0616] 27. [0226] 28. [0030] 29. [0054]
30. [A - r ; B - s ; C - q ; D - p]
MATHEMATICS /AQ # 14 E-53
TM TM TARGET:JEE
TARGET: JEE(Main
(MAIN++Advanced)
ADVANCED)2020
2020
ENTHUSIAST & LEADER
Path to success KOTA (RAJASTHAN) ENTHUSIAST & LEADER
COURSE
Path to success KOTA (RAJASTHAN ) COURSE

ADVANCED QUIZ # 15 DETERMINANT & MATRICES MATHEMATICS

SINGLE CORRECT CHOICE TYPE

é -1 18 ù
é7 -10 17 ù ê ú
1. If 3A + 4B = ê T
ú , 2B - 3A = ê 4 -6 ú , then B =
T

ë 0 6 31 û
ëê -5 -7 úû

é 1 -3 ù é 1 3ù é 1 -4 ù
ê -1 0 ú ê -1 0 ú é 1 -2 3 ù ê -2 2 ú
(A) ê ú (B) ê ú (C) ê ú (D) ê ú
êë 2 4 úû êë 2 4 úû ë -4 2 5 û êë 3 5 úû

If A is an idempotent matrix then ( I + A ) is equal to


10
2.
(A) I + A (B) I + 10A (C) I + 1023A (D) I + 1024A
3. Consider the matrix A, B, C, D with the order 2 × 3, 3 × 4, 4 × 4, 4 × 2 respectively. Let x = (a A B g C2
D)3 where a and g are scalars. Let | x | = K | ABC2D|3, then K is
(A) ag (B) a2 g2 (C) a4 g4 (D) a6 g6
r - 1ù
If the matrix Mr is given by M r = éê
r
4. ú , r = 1, 2, 3, ......, then the value of det(M1) + det (M2)+...........
ë r - 1 r û
+ det(M2008) is
(A) 2007 (B) 2008 (C) (2008)2 (D) (2007)2

1 a a2
cos(p - d)x cos px cos(p + d)x
5. The value of the determinant does not depend upon
sin(p - d)x sin px sin(p + d)x
(A) p (B) d (C) x (D) a
é3 -4 ù
6. If X = ê ú , the value of Xn is
ë1 -1û

é3n -4n ù é2 + n 5 - n ù é3n ( -4) n ù


(A) ê ú (B) ê (C) ê n ú
-n úû
(D) None of these
ë n -n û ë n ë1 ( -1) n û

1 1 1
7. If a b c = (a - b)(b - c)(c - a)(a + b + c) , where a, b, c are all different, then the determinant
a3 b 3 c3

1 1 1
(x - a)2 (x - b) 2 (x - c) 2
vanishes when
(x - b)(x - c) (x - c)(x - a) (x - a)(x - b)

1 1
(A) a + b + c = 0 (B) x = (a + b + c) (C) x = (a + b + c) (D) x = a + b + c
3 2
MATHEMATICS /AQ # 15 E-54
TM TARGET: JEE (MAIN + ADVANCED) 2020
ENTHUSIAST & LEADER
Path to success KOTA (RAJASTHAN)
COURSE

1 + sin 2 x cos 2 x sin 2x


If maximum and minimum values of the determinant sin x 1 + cos x
2 2
8. sin 2x are a and b, then
2
sin x cos x 1 + sin 2x
2

(A) a + b99 = 4
(B) a3 – b17 = 25
(C) ( a 2 n - b 2n ) is always an or integer for n Î N
(D) a triangle can be constructed having it’s sides as a, b, and a – b

ab bg ga
9. If a, b, g are the roots of px3 + qx2 + r = 0 then the value of the determinant bg ga ab is
ga ab bg

(A) p (B) q (C) 0 (D) r]

é a1 a 2 a3 ù
10.
ê
If ê b1 b 2 b3 úú is a singular matrix and P1 º a1x + b1 y + c1z = 0 ; P2 º a 2 x + b 2 y + c 2 z = 0 ;
êë c1 c 2 c3 úû

P3 º a 3 x + b 3 y + c3 z = 0 are three distinct planes then which of the following statement is true.
(A) P1, P2, P3 are concurrent at only one point
(B) P1, P2, P3 are non concurrent
(C) P1, P2, P3 having three distinct line of intersection
(D) P1, P2, P3 having only one line of intersection
PASSAGE TYPE QUESTION
PARAGRAPH FOR QUESTION NOS. 11 TO 12

8 ( x3 + x )
Consider f ( x)=
( 2 x - 1)
3

Let f (x) increases in the interval (a, b) and decreases in ( -¥, a ) È ( b, c ) È ( c, ¥ ) . Let A be the matrix such

é -a c ù
that A = ê ú & let g (x) = f (| x |).
ë f (0) -3b û
On the basis of above information, answer the following questions :

é -1 æ 1 ö ù
11. êsin sin ç abc ÷ ú is equal to, where [.] is greatest integer function
ë è øû
(A) 0 (B) –1 (C) 1 (D) –2
12. The matrix A10 is given by

é -a -15b ù é -a -3b ù é -a -55b ù é -a f (0) ù


(A) ê ú (B) ê ú (C) ê ú (D) ê
ë f (0) -3b û ë f (0) -3b û ë f (0) -3b û ë f (0) -3b úû
MATHEMATICS /AQ # 15 E-55
TM TARGET: JEE (MAIN + ADVANCED) 2020
ENTHUSIAST & LEADER
Path to success KOTA (RAJASTHAN)
COURSE
13. If the set of values of ‘k’ for which the equation g (x) = k has no solution is (p, q], then the value of p + q is
(A) 2 (B) 3 (C) 1 (D) 6
INTEGER ANSWER TYPE

b-c c-a a-b a b c


b '- c ' c '- a ' a '- b ' m a' b' c'
14. If the determinant is expressible as then m is
b"- c" c"- a " a "- b" a " b" c"

1 + sin 2 x cos 2 x 4sin 2x


15. Let f (x) = sin 2 x 1 + cos 2 x 4sin 2x , then the maxmum value of f(x) is
sin 2 x cos 2 x 1 + 4sin 2x

a a+b a+b+c
16. If 2a 3a + 2b 4a + 3b + 2c = 64 , Find the value of (a, b, c Î R) a =
3a 6a + 3b 10a + 6b + 3c

17. If (x1 – x2)2 + (y1 – y2)2 = a2


(x2 – x3)2 + (y2 – y3)2 = b2
(x3 – x1)2 + (y3 – y1)2 = c2

2
x1 y1 1
and k x 2 y 2 1 = (a + b + c) (b + c – a) (c + a – b) × (a + b – c), then the value of k is
x3 y3 1

x3 + 1 x2y x 2z
18. The number of positive integral solutions of the equation xy 2 y3 + 1 y 2 z = 11 is
xz 2 yz 2 z3 + 1

é 2 -1ù é -1 -8 -10 ù
ê1 0ú ê ú
19. If ê ú A = ê 1 -2 -5 ú , then sum of all the elements of matrix A is
êë -3 4 úû êë 9 22 15 úû

20. If A is a square matrix of order n such that |adj (adj A)| = |A|9, find the possible value of n is.

MATHEMATICS /AQ # 15 E-56


TM TARGET: JEE (MAIN + ADVANCED) 2020
ENTHUSIAST & LEADER
Path to success KOTA (RAJASTHAN)
COURSE
MATCH THE COLUMN :
21. Consider a square matrix A of order 2 which has its elements as 0, 1, 2 and 4. Let N denotes the number
of such matrices.
Column -I Column-II
(A) Possible non-negative value of det(A) is (p) 2
(B) Sum of values of determinants corresponding to N matrices is (q) 4
(C) If absolute value of (det (A)) is least, then possible value of (r) –2
|adj (adj (adj A))|
(D) If det (A) is algebraically least, then possible value of det (4A–1) is (s) 0
(t) 8

é3 -4 ù éa b ù
22. Consider the matrices A = ê ú and B = ê0 1 ú and let P be any orthogonal matrix and Q = PAP
APT
ë1 - 1û ë û
and R = PTQKP also S = PBPT and T = PTSKP
Column - I Column - II
(A) If we vary K from 1 to n then the first row (p) G.eometric Progression with
first column elements at R will form common ratio a
(B) If we vary K from 1 to n then the 2 nd row 2nd (q) Arithmetic Progression with
column elements at R will form common difference 2
(C) If we vary K from 1 to n then the first row first (r) Arithmetic Progression with
column elements of T will form common difference 0
(D) If we vary K from 3 to n then the second row 2nd (s) ArithmeticProgression with
column elements of T will form common difference - 2.

ANSWER KEY
1. (B) 2. (C) 3. (D) 4. (C) 5. (A) 6. (D) 7. (B)
8. (A) 9. (C) 10. (D) 11. (B) 12. (A) 13. (C) 14. [0000]
15. [0006] 16. [0004] 17. [0004] 18. [0003] 19. [0001] 20. [0004]
21. (A-p, q, t; B-s; C-p, r; D-r) 22. [A–q ;B–s ; C– p; D–r]

MATHEMATICS /AQ # 15 E-57


TM TM TARGET:JEE
TARGET: JEE(Main
(MAIN++Advanced)
ADVANCED)2020
2020
ENTHUSIAST & LEADER
Path to success KOTA (RAJASTHAN) ENTHUSIAST & LEADER
COURSE
Path to success KOTA (RAJASTHAN ) COURSE

ADVANCED QUIZ # 16 VECTOR & 3D MATHEMATICS


SINGLE CORRECT CHOICE TYPE

1. If aˆ, bˆ and ĉ are unit vectors such that éë aˆ bˆ cˆ ùû = 1 then éë 2aˆ - bˆ 2bˆ - cˆ 2cˆ - aˆ ùû is equal to

(A) 7 (B) 8 (C) 9 (D) 16


2. Which one of the following line is parallel to the line : L : (x, y, z) = (1, 0, –2) + t (–1, 3, 0), t Î R

x +1 z - 3 y y y
(A) = , y = 3 (B) 1 - x = = z + 2 (C) 1 - x = , z = 5 (D) x + 1 = , z = 2
3 2 3 3 3
r r rr r r r r r r r
3. Given | p | = 2; | q | = 3 and p.q = 0. If V = (p ´ (p ´ (p ´ (p ´ q)))) then the vector V is
r r r r r r r
(A) collinear with p (B) V = 16 p (C) V = 48q (D) V = 16q
4. Consider the lines
L1 : x = 3 – t, y = 2 + t, z = 5t, intersecting the plane x – y + 2z = 9 at the point A
L2 : x = 1+ 2t, y = 4t, z = 2 – 3t, intersecting the plane x + 2y – z + 1 = 0 at the point B
and L3 : x = y – 1 = 2z, intersecting the plane 4x – y + 3z = 8 at the point C.
The points A, B, C
(A) constitute a right triangle (B) constitute an acute triangle
(C) constitute an obtuse tringle (D) do not form a tringle
5. Which of the following lines is perpendicular to the plane 2x – y – 5z = 7
(A) x = 3 + 4 t ; y = –2t ; z = –10t (B) x = 5t ; y = 1 + t ; z = –2t
(C) x = 2 + 3t ; y = –1 + t ; z = –5 + t (D) x = 7 + 5t ; y = –t ; z = –10t
6. Let P(2, 1, 4) be a point not lying on the plane passing through the points Q(1, 0, 0), R(0, 2,0) and
S(0, 0, 3). Distance of the point P from the plane containing the points Q,R and S is
(A) 9/7 (B) 11/7 (C) 13/7 (D) 17/7
7. Centroid of the tetrahedron OABC, where A º (a, 2, 3) , B º (1, b, 2) , C º (2,1, C) and O is the origin is
(1, 2, 3) the value of a 2 + b 2 + c 2 is equal to
(A) 75 (B) 80 (C) 121 (D) None of these
x -1 y - 2 z - 3 x -3
8. Equation of plane which passes through the point of intersection of lines = = and
3 1 2 1
y -1 z - 2
= = and at greatest distance from the point (0, 0, 0) is :
2 3
(A) 4x + 3y + 5z = 25 (B) 4x + 3y + 5z = 50 (C) 3x + 4y + 5z = 49 (D) x + 7y – 5z = 2
9. The foot of the perpendicular drawn from a point with position vector ˆi + 4kˆ on the line joining the points
having position vectors as -11jˆ + 3kˆ and 2iˆ - 3jˆ + kˆ has the position vector

(A) 4iˆ + 5ˆj + 5kˆ (B) 4iˆ + 5ˆj - 5kˆ (C) 5iˆ + 4ˆj - 5kˆ (D) None of these

MATHEMATICS /AQ # 16 E-58


TM TARGET: JEE (MAIN + ADVANCED) 2020
ENTHUSIAST & LEADER
Path to success KOTA (RAJASTHAN)
COURSE
10. The equation of a plane through the line of intersection of 2x + 3y + z – 1 = 0 and
x + 5y – 2z +7 =0 and parallel to the line y = 0 = z is
(A) 7x – 5y + 15 = 0 (B) 7y – 5z + 15 = 0
(C) 4x + 7y – 5z + 15 = 0 (D) 13y – 3z + 13 = 0
11. On the sides CA, CB of a triangle ABC, two squares CXYA, CWZB of lengths 1, 2 units are constructed
outwardly. If CA = b, CX = a, CB = x, CW = y,. Then (x + b) 2 + (y + a) 2 = .......
(A) 10 (B) 12 (C) 8 (D) 16
12. Let a, ˆ cˆ be unit vectors such that aˆ ´ bˆ = cˆ and â × bˆ = 0 . Also, x is any vectors such that
ˆ b,

[x bˆ c] ˆ = 2 . Then x is equal to
ˆ = 4 and [x aˆ b]
ˆ = 3,[x cˆ a]

(A) 2aˆ + 3bˆ + cˆ (B) 3aˆ + 4bˆ + 2cˆ (C) aˆ + 2bˆ + 3cˆ (D) None of these
r
13. Equation of the plane containing the lines r = ˆi - ˆj + 3kˆ + l (iˆ - ˆj + 2k)
ˆ
r
r = (2,0, 2) + s(-1,1,0) is
(A) x + 3y + z = 4 (B) x + y – 2 = 0 (C) 5x – 3y – 4z = 2 (D) none of these

14. If a plane meets co-ordinate axes in A, B, C such that the centroid of the triangle is (1, k, k 2 ) then equation
of the plane is
(A) x + ky + k 2 z = 3k 2 (B) k 2 x + ky + z = 3k 2 (C) x + ky + k 2 z = 3 (D) k 2 x + ky + z = 3

15. Image of the point ‘P’ with position vector 7iˆ + ˆj + 2kˆ in the line whose vector equation is
r
( )
r = -3jˆ - 10kˆ + l 4iˆ + 3jˆ + 5kˆ has the position vector

(A) -9iˆ + 5jˆ + 2kˆ (B) 9iˆ + 5ˆj - 2kˆ (C) 9iˆ - 5ˆj - 2kˆ (D) 9iˆ + 5ˆj + 2kˆ
PASSAGE TYPE QUESTION
PARAGRAPH FOR QUESTION NO. 16 TO 18
x -1 y z + 1
Consider the line L : = = and a point A(1, 1, 1). Let P be the foot of the perpendicular from
2 1 -2
A on L and Q be the image of the point A in the line L, ‘O’ being the origin.
16. The distance of the origin from the plane passing the point A and containing the line L, is

1 1 2 1
(A) (B) (C) (D)
3 3 3 2
17. The distance of the point A from the line L, is

4
(A) 1 (B) 2 (C) 3 (D)
3
18. The distance of the origin from the point Q, is

17 17 1
(A) 3 (B) (C) (D)
6 3 3
MATHEMATICS /AQ # 16 E-59
TM TARGET: JEE (MAIN + ADVANCED) 2020
ENTHUSIAST & LEADER
Path to success KOTA (RAJASTHAN)
COURSE
PARAGRAPH FOR QUESTION NOS. 19 TO 21
Points A and B in the Argand plane are represented by the complex numbers z1 = –4i and z2 = 1 + 2i
respectively

z + 4i
19. If P represented by z is any point in the Argand plane such that = 1 , locus of P is the straight line
z - 1 - 2i
(A) 2x + 12y – 11 = 0 (B) 2x + 12y + 11 = 0 (C) 12y + 2x – 10 = 0 (D) 12y + 2x + 10 = 0

p
20. AB is rotated through an angle in anticlock wise w.r.t. point A and the new position of B is denoted by
2
B¢. Then B¢ is represented by
(A) –3 – 6i (B) 3 – 6i (C) –6 + i (D) –6–3i
21. A is translanted by 4 units in the positive direction of the real axis and its new position is A¢. Area of
DOA¢B¢, where O is the point z = 0, is (B' is mentioned in previous question)
9
(A) (B) 9 (C) 18 (D) 36
2
MULTIPLE CORRECT CHOICE TYPE
22. Consider a triangle ABC is xy plane with D, E and F as the middle points of the sides BC, CA and AB
respectively. If the coordinates of the points D, E and F are (3/2, 3/2) ; (7/2, 0)and (0, –1/2) then which of
the following are correct?
(A) circumcentre of the triangle ABC does not lie inside the triangle.
(B) orthocentre, centroid, circumcentre and incentre of triangle DEF are collinear but of triangle ABC are
non collinear.
(C) Equation of a line passes through the orthocentre of triangle ABC and perpendicular to its plane is
r
r = 2(iˆ - ˆj) + l kˆ

5 2
(D) distance between centroid and orthocentre of the triangle ABC is .
3
r r r r
23. Which of the following statement (s) is/are true in respect of the lines rr = ar + lb ; r = c + md where
r r
b´d ¹ 0
r r
æ | b×d | ö
(A) acute angle between the lines is cos ç r r ÷
-1

è | b || d | ø
r rr r r r
(B) The lines would intersect if [c b d] = [a b d]
r r r r
(C) The lines will be skew if [c - a b d] ¹ 0
r r r r r r
(D) If the lines intersect at r = r0 then the equation of the plane containing the lines is [r - r0 b d] = 0 .
24. A line passes through two points A(2, –3, –1) and B(8, –1, 2). The co-ordinates of a poitn on this line at a
distance of 14 units from A are
(A) (14, 1, 5) (B) (–10, –7, –7) (C) (86, 25, 41) (D) None of these
MATHEMATICS /AQ # 16 E-60
TM TARGET: JEE (MAIN + ADVANCED) 2020
ENTHUSIAST & LEADER
Path to success KOTA (RAJASTHAN)
COURSE
r ˆ and rr = ˆi + 2ˆj - kˆ + m(iˆ + ˆj + 3k)
25. Equation of the plane containing the lines r = ˆi + 2ˆj - kˆ + l (iˆ + 2ˆj - k)
is
r
(A) r × (7iˆ - 4ˆj - k)
ˆ =0 (B) 7(x - 1) - 4(y - 2) - (z + 1) = 0
r r
(C) r × (iˆ - 2ˆj - k)
ˆ =0 (D) r × (iˆ + ˆj + 3k)
ˆ =0
INTEGER ANSWER TYPE
26. Given f 2 (x) + g 2 (x) + h 2 (x) £ 9 and U(x) = 3f (x) + 4g(x) + 10h(x), where f(x),g(x) and h (x) are
continuous "x Î R If maximum value of U(x) is N , then find N.
27. Let ABCD is any quadrilateral and P and Q are the midpoints of its diagonal. If
uuur 2 uuur 2 uuur 2 uuur 2 uuur 2 uuur 2 uuur 2
AB + BC + CD + DA - AC - BD = l PQ , then find the value of l.
MATCH THE COLUMN :
28. Column - I Column - II
(A) Let O be an interior point of DABC such that (p) 0
uuur uuur uuur r
OA + 2OB + 3OC = 0, then the ratio of the area
of DABC to the area of DAOC, is
(B) Let ABC be a triangle whose centroid is G, orthocentre is H and (q) 1
circumcentre is the origin ‘O’. If D is any point in the plane of the
triangle such that no three of O, A, B, C and D are collinear
uuur uuur uuur uuuuur uuur
satisfying the relation AD + BD + CH + 3HG = l HD then the (r) 2
value of the scalar ‘l’ is
r r r r
(C) If a, b, c and d are non zero vectors such that no three of them are (s) 3
in the same plane and no two are orthogonal then the value of the
r r r r r r r r
scalar (b ´ c).(ar´ d)r + (c
r r
´ a).(b ´ d) is
(a ´ b).(d ´ c)
29. Column - I Column - II
r r r r r
(A) Given two vectors ar and b such that | a | = | b | = | a + b | = 1 (p) 30°
r r
The angle between the vectors 2ar + b and a is
(B) In a scalene triangle ABC, if a cos A = b cos B (q) 45°
then ÐC can be equal to
(C) In a triangle ABC, BC = 1 and AC = 2. The maximum possible (r) 60°
value which the ÐA can have is
(D) In a D ABC ÐB = 75o and BC = 2AD where AD is the (s) 90°
altitude from A, then ÐC equals to

ANSWER KEY
1. (A) 2. (C) 3. (D) 4. (B) 5. (A) 6. (D) 7. (A)
8. (B) 9. (D) 10. (B) 11. (A) 12. (B) 13. (B) 14. (B)
15. (B) 16. (A) 17. (B) 18. (C) 19. (B) 20. (D) 21. (C)
22. (A, C, D) 23. (A, B, C, D) 24. (A, B) 25. (A,B) 26. [1125]
27. [0004] 28. [A–s ; B–r ; C–q] 29. [A–p; B–s ; C–p; D–p ]
MATHEMATICS /AQ # 16 E-61
TM TM TARGET:JEE
TARGET: JEE(Main
(MAIN+ +Advanced)
ADVANCED)2020
2020
ENTHUSIAST & LEADER
Path to success KOTA (RAJASTHAN) ENTHUSIAST & LEADER
COURSE
Path to success KOTA (RAJASTHAN ) COURSE

ADVANCED QUIZ # 17 TRI, TE & SOLUTION OF TRIANGLE MATHEMATICS


SINGLE CORRECT CHOICE TYPE
n sin(3r q)
1. If P = (tan(3n +1 q) - tan q) and Q = å r +1
, then
r =0 cos(3 q)
(A) P = 2Q (B) P = 3Q (C) 2P = Q (D) 3P = Q
2. The measure of the smallest positive angle x in radians, such that sin(x radians) = sin (x degree), is
p + 180 p 180 180p
(A) (B) (C) (D)
180 180 + p 180 + p 180 + p

1 3 11
3. Let x be real number which satisfy log3x = 1 + sinq, where q Î [0, 2p]. Then x - + - x is equal
2 2 2
to
(A) 7 (B) 2x – 7 (C) 7 – 2x (D) 4
4. Let f (q) = 2 cos q - cos2 q "q Î R then find the range of f(q) is

é1 ù
(A) [-2, 1] (B) ê , 1ú (C) [ -3, 1] (D) [ -3, 0]
ë4 û

p p
+ sin 2
tan 2
5. The value of 5 5 is equal to
p p
tan 2 .sin 2
5 5

11 + 5 11 + 5 13 + 5 13 - 5
(A) (B) (C) (D)
5- 5 3- 5 5+ 5 5+ 5

12p 14p 4p p
6. The exact value of the expression cos + cos + 2 cos cos is
17 17 17 17

1
(A) 0 (B) 1 (C) (D) None of these
2

sin 3 q - cos3 q cos q


7. - - 2 tan q cot q = -1 if
sin q - cos q 1 + cot 2 q

æ pö æp ö æ 3p ö æ 3p ö
(A) qÎ ç 0, ÷ (B) qÎ ç , p ÷ (C) qÎ ç p, ÷ (D) qÎ ç , 2p ÷
è 2ø è2 ø è 2 ø è 2 ø

x
8. If cos 2 x - 2 cos x - 4sin x + sin 2x = 0 , then tan can be equal to
2

1
(A) - (B) 1 (C) 1 + 5 (D) 2 - 5
2
MATHEMATICS /AQ # 17 E-62
TM TARGET: JEE (MAIN + ADVANCED) 2020
ENTHUSIAST & LEADER
Path to success KOTA (RAJASTHAN)
COURSE
9. The minimum value of the expression (3sinx – 4cosx – 10)(3sinx + 4cosx – 10) is

193 - 60 2
(A) 49 (B) (C) 84 (D) 45
2
10. In a triangle ABC the value of angle A is given by 5cosA+ 3 = 0, then the equation whose roots are sinA and
tanA will be
(A) 15x2 – 8x + 16 = 0 (B) 15x2 + 8x – 16 = 0
(C) 15x2 – 8 2 x+ 16 = 0 (D) 15x2 – 8x – 16 = 0
11. cot 50° + 2cot 80° =
(A) cot 30° (B) tan 50° (C) tan 40° (D) tan 60°
12. The number of solutions of the equation sin 3 x cos x + sin 2 x cos 2 x + sin x cos 3 x = 1 , in the interval [0, 2p] ,
is
(A) 4 (B) 2 (C) 1 (D) 0

13. Let f (x) = cos ec 4 x - 2 cos ec 2 x + 1 . The sum of all the solutions of
sin x - cos x
cos ecx(cos ecx - sin x) + + cot x
sin x
f (x) = 0 in [0, 100p] is
(A) 2550p (B) 2500 p (C) 5000 p (D) 5050 p
q 7
14. If cos 2q - cos 3q - cos 4q + cos 5q = l sin sin q cos q is true for all values of q, then l is equal to
2 2
(A) l = -2 (B) l = 2 (C) l = 4 (D) l = -4

cos x + sin x. tan 55°


15. If tan 3x +tan 5x + tan 10x = tan 3x . tan 5x . tan 10x, then is
cos x.tan 55° - sin x
(where x is minimum positive integral value in degree)
(A) 0 (B) 1 (C) –1 (D) does not exist

a b æa+bö
16. If x = cosa + cos b – cos (a + b) and y = 4sin sin cos ç ÷ , then (x – y) equals.
2 2 è 2 ø
(A) 0 (B) 1 (C) –1 (D) –2

17. If x Î (0, 2p) and y Î (0, 2p) , then find the number of distinct ordered pairs (x, y) satisfying the euqation
9cos2x + sec2y – 6cosx – 4secy + 5 = 0
(A) 1 (B) 2 (C) 3 (D) 4
18. Which of the following sets can not be the subsets of the general solution of the equation,
1 + cos3x = 2cos 2x.
p p p
(A) np + (B) np + (C) np - (D) 2np
3 6 6

MATHEMATICS /AQ # 17 E-63


TM TARGET: JEE (MAIN + ADVANCED) 2020
ENTHUSIAST & LEADER
Path to success KOTA (RAJASTHAN)
COURSE

1 1 1 1 a2 + b2 + c2
19. In a DABC if + + + = , then n =
r 2 r12 r22 r32 Dn
(A) 1 (B) 2 (C) 3 (D) 4
sin 3B
20. If in DABC, a2,b2,c2 are in Arithmetic Progression then the value of is
sin B

(a + c2 ) (a - c2 )
2 2
a 2 + c2 a 2 - c2 2 2

(A) (B) (C) (D)


c2 2ab 4a 2 c2 4a 2 c 2
21. In an acute triangle DABC, ÐABC = 45°, AB = 3 and AC = 6 . The angle ÐBAC is
(A) 60° (B) 65° (C) 75° (D) 150° & 75°
22. A triangle has side a = 7 , the opposite angle a = 60° and the sum of the two other side is (b + c) = 5.
The ratio of the longest to the shortest side of the triangle, is
3 7
(A) 2 (B) 2 (C) (D)
2 2
1 1
23. Base angles of triangle are 22 ° and 112 ° , then height of the triangle is
2 2
(A) half the base (B) the base (C) twice the base (D) four times the base
24. In a DABC, the sides a, b and c are such that they are the roots of x 3 - 16x 2 + 34x - 50 = 0. Then
cos A cos B cos C
+ + is equal to
a b c
47 17 32 9
(A) (B) (C) (D) -
25 25 25 25
PASSAGE TYPE QUESTION
PARAGRAPH FOR QUESTION NOS. 25 TO 27
cos A æ cos B cos C ö a b
In a DABC, + 2ç + ÷= +
a è b c ø bc ac
25. B equals
p p p p
(A) (B) (C) (D)
3 6 2 4
26. The radius of the incircle of DABC of above Question is
a +b-c a-b+c b+c-a a+b+c
(A) (B) (C) (D)
2 2 2 2
27. If the angles A, C, B of D ABC of I Qu. of paragraph are in A.P., S sin 2A =
st

(A) 2 (B) 3 (C) 3 (D) 2


MULTIPLE CORRECT CHOICE TYPE
28. If (cos 2 x + sec 2 x)(1 + tan 2 2y)(3 + sin 3z) = 4 , then
(A) x may be a multiple of p (B) x cannot be an even multiple of p
p
(C) z can be a multiple of p (D) y can be a multiple of
2
MATHEMATICS /AQ # 17 E-64
TM TARGET: JEE (MAIN + ADVANCED) 2020
ENTHUSIAST & LEADER
Path to success KOTA (RAJASTHAN)
COURSE
INTEGER ANSWER TYPE
Find number of solutions of the equation ( log 2 cos q ) + log
2
29. 4 (16 cos q) = 2 , in interval [0, 2p).
cos q

3p
30. Find number of integral values of x satisfying log4(3x2 – 8x + 7) – log2(x – 2) ³ –cot .
4

æ 3p ö p
31. For a Î ç , p ÷ and b Î R - ( 2n + 1) , n Î I, find number of ordered pairs (a, b) satisfying the
è 4 ø 2

1 1 + cot a
equation 2 cot a + = .
sin a
2
tan b
32. Find the number of principal solutions of the trigonometric equation

æ cos 3x + sin 3x ö
5 ç sin x + = cos 2x + 3
è 1 + 2sin 2x ÷ø

2s b c a
33. If = + - and orthocentre of DABC is at a distance of 60 units from the mid point of side BC of
r1 r2 r3 r1
the triangle, then find the distance between centroid and circumcentre of DABC.
[Note : All symbols used have their usual meanings in a triangle]]
34. In a triangle ABC; A = p/3, b = 50, c = 30, AD is a median through A, then (AD)2 is equal to
MATCH THE COLUMN :
35. If 0 £ x £ 2p, then number of solutions of
Column - I Column - II
(A) sin 2 x - cos x = 1/ 4 (p) 6
(B) sin 2x = cos 3x (q) 4
(C) tan 2 x + cot 2 x = 2 (r) 2
(D) sin x = 1/ 2, cos x = 3 / 2 (s) 1
36. Column - I Column - II
(A) If the sines of the angles A and B of a triangle ABC satisfy the (p) right angled
equation c2x2 – c(a + b)x + ab = 0 the triangle can be
(B) If one angle of triangle is 30° and the lengths of the sides (q) isosceles
adjacent to it are 40 and 40Ö3, the triangle can be (r) equilateral

(C) In a triangle ABC, cos A cos B + sin A sin B sin C = 1, (s) obtuse angled
then the triangle can be
ANSWER KEY
1. (A) 2. (D) 3. (D) 4. (C) 5. (A) 6. (A) 7. (B)
8. (D) 9. (A) 10. (B) 11. (B) 12. (D) 13. (C) 14. (D)
15. (B) 16. (B) 17. (D) 18. (A) 19. (B) 20. (D) 21. (C)
22. (C) 23. (A) 24. (A) 25 (C) 26. (B) 27. (B) 28. (AD)
29. [0003] 30. [0004] 31. [0000] 32. [0002] 33. [0020] 34. [1225]
35. (A – r; B – p; c – q; D – s) 36. (A-p; B-q, s; C-p, q.)
MATHEMATICS /AQ # 17 E-65
TM TM TARGET:JEE
TARGET: JEE(Main
(MAIN ++Advanced)
ADVANCED) 2020
2020
ENTHUSIAST&
ENTHUSIAST & LEADER
LEADER
Path to success KOTA (RAJASTHAN)
COURSE
Path to success KOTA (RAJASTHAN ) COURSE

ADVANCED QUIZ # 1(SOLUTION) FU NC TI ON S MATHEMATICS


SINGLE CORRECT CHOICE TYPE
1.Ans. (D) h(x) = f(g(x)) + 10 = |x|2 + 4|x| – 1 + 10 = |x|2 + 4|x| + 9 = (|x| + 2)2 + 5
Hence range of h(x) is [9, ¥)
Minimum value occurs when x = 0
2.Ans. (A) Clearly, graph in option (A) best represent the graph of function y = ||x – 2| – 2|
3.Ans. (B) As f is one –one and onto, so f is invertible.
4x
Now y =
3x + 4

4y
Þ 3xy + 4y = 4x Þ (4 - 3y)x = 4y \ g(y) =
4 - 3y

4.Ans. (B) We have f(x)

æ æ x 1 ö2 3 ö
= log (4 x
- 2 x
+ 1) = log 4 çç2 - ÷ + ÷
7p
ç 2 ø 4 ÷ø
3 èè
cos ec2
3
144424443
Increa sin g function

Clearly, range of f(x) = [–1, ¥).


5.Ans. (A) Clearly x2 + 4x ³ 0
2x2 + 3 ³ 0
x2 + 4x ³ 2x2 + 3
and x is an integer
\ x Î {1, 2} \ n = 2
x 2 + 4x
Now maximum value C2x2 +3 = 12

\ Y = | ln12 |

|Y| = 2 ( lne < ln12 < lne )


2 3
\

\ [ n + | Y |] =| 2 + 2 |= 4
6.Ans. (C) y = f(x) = cos2x + sin4x
Þ y = f(x) = cos2x + sin2x (1 – cos2x)
Þ y = cos2x + sin2x – sin2x cos2x
Þ y = 1 – sin2x cos2x

1 2
Þ y = 1 - sin 2x
4

MATHEMATICS /AQ # 1 E-66


TM TARGET: JEE (MAIN + ADVANCED) 2020
ENTHUSIAST & LEADER
Path to success KOTA (RAJASTHAN)
COURSE
3
\ £ f (x) £ 1 (\ 0 £ sin 2 2x £ 1 )
4
Þ f(R) Î [3/4, 1]

é cos(2x + 6) + cos 6 1 + cos(2x + 6) ù


7.Ans. (A) f (x) = ê 2
-
2 ú
ë û

é cos 6 - 1 ù
f (x) = ê ú = ëé - sin 3ûù Þ f (x) = -1
2

ë 2 û
\ -1 = x 3 Þ x = -1 ]
8.Ans. (A) We must have ax3 + (a + b)x2 + (b + c) x + c > 0
Þ ax2(x + 1) + bx(x + 1) + c(x + 1) > 0
Þ (x + 1) (ax2 + bx + c) >0
2
æ b ö
Þ a(x + 1) ç x + ÷ > 0 as b2 = 4ac
è 2a ø
b
Þ x > -1 and ¹ -
2a
Paragraph for question Nos. 9 to 10
Sol. (9. B, 10. C) Drawing the graph of y = f(x)
9. From above graph, clearly range of f(x) is [1, 3]

10. When -2 £ x £ -1 {f (x)} = 0


(As f(x) = 1 "x Î [ -2, -1] )
1 -1
when -1 £ x £ 0 {f (x)} = is possible for one value of x, which is x =
2 2
1 1
And, when 0 < x £ 1, {f (x)} = is possible for one value of x, which is
2 2
1 3
when 1 < x £ 2, {f (x)} = is possible for one value of x, which is
2 2
1 æ -1 1 3 ö
Hence, total number of values of x for which {f (x)} = is three. ç i.e. , , ÷ ]
2 è 2 2 2ø
Sol. [11. B, 12. C. 13. A, 14. A ]
Graph of f(x) is
From graph a must be 1 and b will be –2

(+1, -2)
\ a + b = –1
g(x) = x2 – 2x – 1 + 3x – 1 Þ g(x) = x2 + x – 2
graph of g(x) is (-2, 0) (1, -0)

graph of g(|x|) is (0, -2)

æ 1 5ö
ç - ,- ÷
è 2 4ø

MATHEMATICS /AQ # 1 E-67


TM TARGET: JEE (MAIN + ADVANCED) 2020
ENTHUSIAST & LEADER
Path to success KOTA (RAJASTHAN)
COURSE

(-1, 0) (1, 0)

(0, -2)
Least value of g|x| is –2
Now f : [a, ¥) ® [b, ¥) Þ f : [1, ¥) ® [–2, ¥)
y = x2 – 2x – 1 ® x2 – 2x – 1 – y = 0
2 ± 4 + 4(1 + y)
x= Þ x = 1± 2 + y
2
f–1(x) = 1 ± 2+x
but range of f–1(x) will be [1, ¥) \ f–1(x) will be 1 + 2 + x (0, -1)

graph fo f(|x|) is
f(|x|) = k has four distinct
real root Þ k Î (–2, –1) (-1, -2) (1, -2)

MULTIPLE CORRECT CHOICE TYPE

15. Ans. (A, C) f (x) = a loga N = N Þ 0 < [2 tan px] < 1 or [2 tan px] > 1

x 2 + 2x - 3 (x + 3)(x - 1)
and > 0 i.e. >0
4x - 4x - 3
2 (2x - 3)(2x + 1)

Þ x Î (-¥, -3) È (-1/ 2,1) È (3 / 2, ¥) ....(i)


Now 0 < [2 tan px] < 1 not possible \ [2 tan px] > 1
Þ 2 tan px ³ 2 Þ tan px ³ 1
p p
Þ np + £ px < np + n Î I
4 2

1 1
Þ n+ £ x < n+ ...(ii)
4 2
common solution of (i) and (ii) possible only if n = 0, n ³ 2 or n £ -4

16. Ans. (A, B, D)

(A) (B)

(C) (D)

Note : If f and g are both bijective then gof is also bijective but converse (in general) is not true

17. Ans. (B, D) (A) We have f(x)

(
= sgn x + x + 1 = 1"x Î R
3 6
) (x +3
)
x 6 + 1 > 0 "x Î R
Þ f(x) is an even fucntion

MATHEMATICS /AQ # 1 E-68


TM TARGET: JEE (MAIN + ADVANCED) 2020
ENTHUSIAST & LEADER
Path to success KOTA (RAJASTHAN)
COURSE
1
(B) is even Þ ex + e–x ³ 2
e + e- x
x

1 1 1 ép p ö
Þ Î (0, ] cos -1 x Î , ÷
e x + e- x 2 e + e - x êë 3 2 ø
(C) Clearly, domain of f(x) = f.
(D) cos–1(cos 7) = 7 – 2p and and sin–1(sin 11)
= 11 – 4p.
So,
2 cos -1 (cos 7) - sin -1 (sin11) = 2(7 - 2 p)

-(11 - 4p) = 14 - 4p - 11 + 4p = 3
18. Ans. (B, C, D)

ì1 x>0
ï
x = 0 and g(x) = x(1 – x ) x Î R
2
f (x) = í 0
ï-1 x <0
î

ì1 g(x) > 0
ï ì1 x Î ( -¥, - 1) È (0,1)
f (g(x)) = í 0 g(x) = 0 ï
Þ f (g(x)) = í 0 x = -1, 0,1
ï-1 g(x) < 0
î ï
î -1 x Î (-1, 0) È (1, ¥)
Þ fg(x) is odd and neither continuous nor differentiable for some x in (–¥, ¥)
g(f(x) = f(x) (1–f(x)) (1 + f(x)) f(x) Î R
gf(x) = 0
x Î R ( Q f(x) is 1, 0 or –1)
Þ B, D
19. Ans.(A, B) Given that, f(x) = [x]2 + [x + 1] – 3

Þ f (x) = [x]2 + [x] + 1 - 3 Þ f (x) = [x]2 + [x] - 2


Þ f (x) = ([x] + 2)([x] - 1)
Now, f(x) = 0 Þ ([x] + 2)([x] - 1) = 0
either [x] + 2 = 0 Þ [x] = -2
Þ - 2 £ x < -1
OR
[x] – 1 = 0 Þ [x] = 1
Þ1£ x < 2
i.e. f(x) = 0 for infinite number of values of x.
Also, f(x) will not be one-to-one
INTEGER ANSWER TYPE
20. Ans. [0008] Since f is injective, so

p p -q
= Þ pq - pr = rp - rq Þ 2pr = q(p + r) .....(1)
r q-r

q r
Also, r = p Þ p, r, q are in G.P..

So, Let r = pa, q = pa2, where a is the common ratio of G.P.


Therefore from equation (1), we get
MATHEMATICS /AQ # 1 E-69
TM TARGET: JEE (MAIN + ADVANCED) 2020
ENTHUSIAST & LEADER
Path to success KOTA (RAJASTHAN)
COURSE
2.p.pa = pa2 (p + pa) Þ 2 = a2 + a Þ a2 + a – 2 = 0 Þ (a + 2) (a – 1) = 0 Þ a = –2, 1
So, (p, –2p, 4p) and (p, p, p)
(But common ratio = a = 1, is not possible as p, q, r are non-zero distinct quantities)
Also, p + q + r = 6 [As g(x) = px2 + qx + r passes through M(1, 6)]
Þ p + 4p – 2p = 6 Þ p = 2
Hence, q = 4p = 4(2) = 8

21. Ans. [0002]

)} + ( 2xx +)1! - e log [3x


( 2x - 1)
2

{ (
f ( x ) = exp ln 3cos
-1
x2 p

]
where [.] denotes greatest integer function.

Domain of f (x) = {1} and its range = 1 + 2 { }


Þ X = {1} and Y = { 2 +1 }
22. Ans. [0002] x - 1 = 2 [ x ] - 3{x}

(i) if x ³ 1, then x - 1 = 2 [ x ] - 3{x}

Þ [ x ] + {x} - 1 = 2 [ x ] - 3{x}

Þ 4 {x} = 1 + [ x ]

\ 0 £ 1+ [x] < 4 \ -1 £ [x] < 3

\ possible value of [x] are 1 and 2

1
If [ x ] = 1, then {x} = \ x = 3/ 2
2

11
if [x] = 2, then {x} = 3/4 \ x =
4
(ii) x < 1, then the equation becomes

1 - x = 2 [ x ] - 3{x} Þ 1 - [ x ] - {x} = 2 [ x ] - 3{x}

Þ 2 {x} = 3 [ x ] - 1

\ 0 £ 3[ x ] - 1 < 2 Þ 1 £ 3[ x ] < 3

1
Þ £ [ x ] < 1 which is not possible
3

11
\ x = 3/ 2, are the only solutions
4

MATHEMATICS /AQ # 1 E-70


TM TARGET: JEE (MAIN + ADVANCED) 2020
ENTHUSIAST & LEADER
Path to success KOTA (RAJASTHAN)
COURSE
MATCH THE COLUMN :
23. Ans. [A–P, S; B–Q,R,T; C–R,T; D–T]
(A) We have
f(x) = 5 x + sin -1 x
Clearly, domain of f(x) = [–1, 1]
Also, f(x) is increasing so f(x) is one-one function

æ 1- | x | ö
(B) f (x) = sgn ç ÷
è 1+ | x | ø

Dr = R
Rf = {–1, 0, 1} even function
(C) For domain of f(x), we must have 8 – 2x – x2 ³ 0
Þ x2 + 2x – 8 £ 0

Þ (x + 4) (x – 2) £ 0

Þ x Î [–4, 2]
Rf = [0, 3]

2-[x]
(D) f (x) = - 2|x| = 2- x - 2|x| = 0 "x £ 0
2{x}

MATHEMATICS /AQ # 1 E-71


TM TM TARGET:JEE
TARGET: JEE(Main
(MAIN ++Advanced)
ADVANCED) 2020
2020
ENTHUSIAST&
ENTHUSIAST & LEADER
LEADER
Path to success KOTA (RAJASTHAN)
COURSE
Path to success KOTA (RAJASTHAN ) COURSE

ADVANCED QUIZ #2(SOLUTION) INVERSE TRIGONOMETRY FUNCTIONS MATHEMATICS

SINGLE CORRECT CHOICE TYPE


1.Ans. (C) We have 1 + sin(cos–1 x) + sin2(cos–1 x) + ....... ¥ = 2
1 1
1 - sin ( cos -1 x ) = 2 Þ 2 = 1 - sin ( cos x )
Þ -1

1
Þ sin(cos -1 x) =
2

p 3
Þ cos -1 x = Þx= Ans.]
6 2

2.Ans. (D) We have cos–1 x + cos -1 y = p + q


a b 2

xy x2 y2
Þ - 1 - 2 1 - 2 = - sin q
ab a b

xy x2 y2
Þ + sin q = 1 - 2 1 - 2
ab a b
On squaring both the sides, we get

x 2 y2 2xy x 2 y2 x 2 y2
Þ 2 2
+ sin 2 q + sin q = 1 - 2 - 2 + 2 2
a b ab a b a b
Þ b2x2 + a2y2 + 2ab xysinq = a2b2cos2q £ a2b2
3. Ans. (A) sin–1 (sin 3) + tan–1 (tan 3) + sec–1 (sec 3) = p – 3 + 3 – p + 3 = 3 ]
4.Ans. (B) We have
sin–1(sin 12) + cos–1(cos 12)
= –(4p – 12) + (4p – 12) = 0
\ (n – 2)x2 + 8x + n + 4 > 0 " x Î R
Þ (n – 2) > 0 Þ n ³ 3 and (8)2 – 4(n – 2)(n + 4) < 0
or n2 + 2n – 24 > 0
Þn>4Þn³5
So, nsmallest = 5
5. Ans. (B) We have,

æ1 1ö
1 1 ç + ÷ æ7ö
tan -1 + tan -1 = tan -1 ç 3 4 ÷ = tan -1 ç ÷
3 4 1
çç 1 - ÷÷ è 11 ø
è 12 ø
Again,
æ 7 1ö
7 -1 1
ç +5÷
-1 11
tan–1 + tan = tan ç ÷
11 5 çç 1 - 7 ÷÷
è 35 ø

æ 46 ö 23
= tan -1 ç ÷ = tan -1
è 48 ø 24

MATHEMATICS /AQ # 2 E-72


TM TARGET: JEE (MAIN + ADVANCED) 2020
ENTHUSIAST & LEADER
Path to success KOTA (RAJASTHAN)
COURSE
-1 1 23
\ tan = tan -1 1 - tan -1
n 24

æ 23 ö
ç 1- ÷ æ 1 ö
= tan ç 24 ÷ = tan -1 ç ÷
-1

çç 1 + 23 ÷÷ è 47 ø
è 24 ø
Þ n = 47
6.Ans. (B) sin -1 (log p (cos-1 x)) > 0 ......(i)

0 < log p (cos -1 x) £ 1 .....(ii)


-1
1 < cos x £ p Þ x Î [ -1,cos1)

sin -1 log p cos -1 x Î (0,1]

Þ ln sin -1 log p cos -1 x Î ( -¥, lnp / 2]

æ pù
\ Range is ç -¥, ln 2 ú
è û
7. Ans. (A) -1 £ 1 - x £ 1 ; - 1 £ x £ 1

Þ 0 £ x £ 2 & - 1 £ x £ 1 ® x Î [ 0,1]

Þ 1 - x Î [ 0,1]
LHS ³ 0, RHS £ 0
Possible only if LHS = RHS = 0 (not possible)
no solution.

8.Ans. (B) Let x = cos a & x = tan b

é pù é pö
where a Î ê 0, ú & b Î ê 0, ÷
ë 2û ë 2ø

p p
Þ a+b = Þb = -a
2 2
Þ tan b = cot a Þ sin 2 a tan 2 b = cos 2 a

Þ (1 - x ) x
2 2
= x2 Þ x 2 - x 4 = x 2

Þ x = 0 is the only solution

é pö
9.Ans. (D) \ range of the I term ê0, 2 ÷
ë ø

é pù
Range of the II term ê 0, ú
ë 2û
Their sum is always < p. It can never be equal to p.
\ the number of solutions is zero.
10.Ans. (B) f(–x) = (–sin–1x) (–sinx) = f(x) x Î [-1,1]
Þ f(x) is an even function and increasing in [0, 1]

é p ù
Range Î [f(0) f(1)] Þ ê 0, sin1ú
ë 2 û

MATHEMATICS /AQ # 2 E-73


TM TARGET: JEE (MAIN + ADVANCED) 2020
ENTHUSIAST & LEADER
Path to success KOTA (RAJASTHAN)
COURSE
PASSAGE
Sol. [11. B, 12. B, 13. C] We have
Sn = x + x 4 + x 7 + x10 + ..... (2n terms)

C n = x 2 + x 5 + x 8 + x11 + ..... (2n terms)


Tn = x 3 + x 6 + x 9 + x12 + ..... (2n terms)
Clearly Sn > Cn > Tn as x is a proper fraction
\ x > x2 > x3 & so on
x
Lim ( Sn + Cn + Tn ) = x + x 2 + x 3 + .....¥ =
n ®¥ 1- x

We have Sn = C +T Þ x
(( x ) -1) 3 2n

n n
(x 3
- 1)

=x
(( x ) -1) + x (( x ) -1)
2
3 2n
3
3 2n

(x 3
- 1) (x 3
- 1)

æ pö
But x ¹ 1, as x Î ç 0, ÷ , so we get x = x2 + x3
è 4ø
Þ x 2 + x - 1 = 0 ( x ¹ 0)

5 -1 p æ pö
Þ x= = 2 sin Î ç 0, ÷ ]
2 10 è 4 ø
MULTIPLE CORRECT CHOICE TYPE
-1 -1
æ 3ö æ 3ö
14.Ans.(CD) a = 3 tan çç 5 ÷÷ + 3tan çç 2 ÷÷
è ø è ø

é 3 3ù
ê + ú
= 3 ê tan -1 5 2 ú = 3 tan -1 æ 7 3 ö = 3 p = p
çç ÷÷
ê 3 ú è 7 ø 3
1-
êë 10 úû

é 3ù
b = 4 ê tan -1 7 - tan -1 ú
ë 4û

é 3ù
ê -1 7 - 4 ú æ 25 ö p
= 4 ê tan ú = 4 tan -1 ç ÷ = 4 ´ = p
ê 21 è 25 ø 4
1+ ú
ë 4 û

-1
æ -1
æ 10 ö 1ö
15.Ans.(AD) (A) sin çç sec çç 3 ÷÷ + cot 3 ÷÷
è è ø ø
p
= sin(cot -1 3 + tan -1 3) = sin
=1
2
(B & C) We have cos–1x – sin–1(–x)
p p p p
= Þ but cos -1 x + sin -1 x = Þ =
3 2 3 2
have no solution
(D) As 4e–x > 0 "x Î R , so g(x) = 1 "x Î R Þ g–1(x) is not defined as g is many one.
MATHEMATICS /AQ # 2 E-74
TM TARGET: JEE (MAIN + ADVANCED) 2020
ENTHUSIAST & LEADER
Path to success KOTA (RAJASTHAN)
COURSE

æ 3 cos x ö
16. Ans. (BC) y = 2 çç sin x - ÷+2
è 2 2 ÷ø

æ pö
Þ y = 2sin ç x - ÷ + 2
è 6ø

y-2 æ pö æ y-2 ö
Þ = sin ç x - ÷ ç Î [-1,1] ÷
2 è 6ø è 2 ø

p æ y-2ö p -1 æ x - 2 ö
\ x- = sin -1 ç ÷ Þ y = + sin ç ÷
6 è 2 ø 6 è 2 ø

-1 p æ x -2ö
\ f (x) = + sin -1 ç ÷ ÞB
6 è 2 ø

p p æ x - 2 ö 2p æ x -2ö
= + - cos-1 ç ÷= - cos-1 ç ÷ ÞC
6 2 è 2 ø 3 è 2 ø

INTEGER ANSWER TYPE

17.Ans. [0002] 0 £ x 2 + x + 1 £ 1 and 0 £ x 2 + x £ 1


Þ only possibility is x2 + x = 0
\ x = – 1,0
for x = – 1

3p
L.H.S. = 2 sin–1 1 + cos–1 0 =
2
\ x = – 1 is a solution.

3p
For x = 0, L.H.S. = 2 sin–1 1 + cos–1 0 =
2
\ x = 0 is a solution.
two solution

MATCH THE COLUMN :


18.Ans. (A – p; B– s; C– q; D – s)
Taking (1/ 2 ) cos -1 x = a in (a)

1 + tan a 1 - tan a
+ =1
1 - tan a 1 + tan a

2 (1 + tan 2 a ) 2 2
Þ =1Þ =1Þ =1
1 - tan 2 a cos 2a x
Þ x = 2, But for x = 2, cos–1 x is not defined. So (a)
has no solution.

1 1
+
2x + 1 4x + 1 = 2 Þ 6x + 2 = 2
Next, 1
1- x2 8x 2 + 6x x 2
( 2x + 1)( 4x + 1)
Þ Either x = 0 or 6x2 – 14x – 12 = 0

MATHEMATICS /AQ # 2 E-75


TM TARGET: JEE (MAIN + ADVANCED) 2020
ENTHUSIAST & LEADER
Path to success KOTA (RAJASTHAN)
COURSE
Þ x = 0, 3, – 2/3
So (b) has 3 solutions.
2
But after cross check x = 0, x = - rejected so only one solution.
3
(c) can be written as
4
tan -1 ( x + ( 2 / x ) ) - tan -1 ( x - ( 2 / x ) ) = tan -1
x

x + ( 2 / x ) - ( x - ( 2 / x )) 4
Þ =
(
1+ x - (4 / x
2 2
)) x

Þ 1+ x - (4 / x ) = 1 Þ x = 4 Þ x = ± 2
2 2 4

(c) has only 2 real solutions.

-1 1 - x) + 1 + x
(d) tan = tan -1 2x
1 - (1 - x 2 )

2
= 2x Þ x 3 = 1
x2
x = 1 one solution (x = 1 satistfy the given equation )
19. Ans. (A –q; B – pq; C – r; D – p)

(A) x = 1 + cot 2 ( cot -1 3) - 1 - tan 2 ( tan -1 2 )


=9–4=5
x + 1/ y
(B) tan -1 = tan -1 3 if (take tan on both side)
1- x / y

3y - 1
Þ x + 1/ y = 3 (1 - x / y ) Þ x =
3+ y

y 2 + y - 56 = 0 Þ ( y + 8 )( y - 7 ) = 0
Þ y = – 8 or 7
Þ x = 5 or x = 2
(C) y = 3, - 3

p
Now cos–1x = tan–1 3 =
3

p 1
x = cos =
3 2

-1 -1 -p
Now, cos x = tan ( - 3) =
3

p
cos -1 x = - not possible
3

1
\ x=
2

(D) sin -1 (1) - sin -1 ( )


3/ y - p / 6 = 0

p p
Þ - = sin -1
2 6
( 3/ y )
MATHEMATICS /AQ # 2 E-76
TM TARGET: JEE (MAIN + ADVANCED) 2020
ENTHUSIAST & LEADER
Path to success KOTA (RAJASTHAN)
COURSE

3 3
Þ sin ( p / 3) = 3 / y Þ = Þ y = 4 Þ x = ± 2.
2 y
20. Ans. [A–p, q, r; B–p, q; C–p, q; D–q]
(A) ƒ ( x ) = sin -1 ( sin x ) + cos -1 ( cos x )

= x + x = 2x, x Î [ 0, p / 2]

= p – x + x = p, x Î [ p / 2, p]

= p – x + 2p – x = 3p – 2x, x Î [ p,3p / 2]

= x – 2p + 2p – x = 0, x Î [3p / 2, 2p]
Rf = [0, p]

(B) g ( x ) = sin -1 x + 2 tan -1 x

g is even, Dg Î [ -1, 1]

x Î [0,1]

if x ³ 0,g ( x ) = sin -1 x + 2 tan -1 x

é p æ p öù
R g = ê 0 + 0, + 2 ç ÷ ú = [ 0, p]
ë 2 è 4 øû

g (x) is increasing " x Î( 0,1)

g (x) is decreasing " x Î( -1,0 )

æ 2x ö
(C) h ( x ) = 2sin -1 ç 2 ÷
, x Î[ 0,1]
è1+ x ø

tan -1 x = q, q Î [ 0, p / 4] , 2q Î [ 0, p / 2]

h ( x ) = 2sin -1 ( sin 2q ) = 4q = 4 tan -1 ( x )

(D) cot ( cot -1 x ) = x " x Î R

21. Ans. [A–P, S; B–Q,R,T; C–R,T; D–T]


(A) We have
f(x) = 5 x + sin -1 x
Clearly, domain of f(x) = [–1, 1]
Also, f(x) is increasing so f(x) is one-one function

æ 1- | x | ö
(B) f (x) = sgn ç ÷
è 1+ | x | ø
Dr = R
Rf = {–1, 0, 1} even function

MATHEMATICS /AQ # 2 E-77


TM TM TARGET:JEE
TARGET: JEE(Main
(MAIN ++Advanced)
ADVANCED) 2020
2020
ENTHUSIAST&
ENTHUSIAST & LEADER
LEADER
Path to success KOTA (RAJASTHAN)
COURSE
Path to success KOTA (RAJASTHAN ) COURSE

ADVANCED QUIZ # 3(SOLUTION) LIMITS, CONTINUI TY, DIFFERENTIABILITY & MOD MATHS

SINGLE CORRECT CHOICE TYPE


(3n + 3)(3n + 2)(3n + 1)(3n)!
1.Ans. (C) Lim
n ®¥ (n + 1) 3 (3n)!

æ 1 öæ 2 öæ 1 ö
27n 3 ç 1 + ÷ç 1 + ÷ç 1 + ÷
Lim è n øè 3n øè 3n ø
3
= n ®¥ æ 1ö =27 Ans.
n 3 ç1 + ÷
è nø

-x -x
2.Ans. (B) l = Lim (e + 1) - (e + ex ) - x(e + e )
2x x x

x ®0 æ e -1 ö
x2 ç ÷
è x ø
multiply Nr and Dr by ex
ex (e 2x + 1) - (e2x + 1) - x(e 2x + 1)
Lim
x ®0 x 2 × ex

é (e x - 1) - x ù 1
=
Lim(e 2x + 1) ê 2 ú = 2 × = 1 ans.]
x®0
ë x û 2

ba n - ab n
3.Ans. (D) f(0+) = f(0–) = (b - a) Þ (D)

4.Ans. (D) y4 = x2 – 6
dy dy x
4y3 = 2x Þ y3 =
dx dx 2
2
d2y æ dy ö 1
Þ y3 2
+ 3y 2 ç ÷ =
dx dx
è ø 2
2
d2y æ x ö 1
y3 + 3y 2 ç 3 ÷ =
dx 2 è 2y ø 2
2 2
3 d y 2 x 1 d 2 y 3x 2 1
Þ y dx 2 + 3y 4y6 = 2 Þ y3 + =
dx 2 4y 4 2

d 2 y 1 3x 2 d 2 y 2y 4 - 3x 2 d 2 y 2y 4 - 3x 2
Þ y3 = - Þ y3 = Þ = Ans.]
dx 2 2 4y 4 dx 2 4y 4 dx 2 4y7

æ 1- x ö
dy 4 1-2x ç ÷
5.Ans. (A) = e + 1 + x + x 2 = 2e è 2 ø
+ x2 + x +1
dx 2

dx 1
= ,
g’(y) = dy æ x -1 ö
-ç ÷
2e è 2 ø
+ x + x +1
2

7
when y = - then x = 1
6

dy ù 1 1
ú = = Ans.
dx û x =-7 / 6 2 + 3 5

MATHEMATICS /AQ # 3 E-78


TM TARGET: JEE (MAIN + ADVANCED) 2020
ENTHUSIAST & LEADER
Path to success KOTA (RAJASTHAN)
COURSE
6.Ans. (C) We have g(x) = f(x) sin x .....(1)
On differentiating equation (1) w.r.t.x, we get
g¢(x) = f(x) cos x + f ¢(x) sin x .... (2)
Again differentiating equation (2) w.r.t.x, we get
g¢¢(x) = f(x) (– sin x) +f ¢(x) cos x + f ¢(x) cos x + f ¢¢(x) sin x ......(3)
Þ g¢¢(–p) = 2f ¢(–k), cos(k) = 2 × 1× –1 = – 2
Hence g¢¢ (–p) = – 2 Ans.]
7.Ans. (D) Given : g(x) is not differentiable at x = a
(a - h - a)f (a - h) - 0
\ LHD of g(x) = Lim+
h ®0 -h

= Lim+ f (a - h) = L1
h ®0

Similarly RHD of g(x) = Lim f (a + h) = L 2


+
h ®0

Given L1 ¹ L2
\ f(x) is discontinuous at x = a. ]

8.Ans. (B) y = (sin x + cos x) x

log e y = x log e (sin x + cos x)


Differentiating both sides

1 dy 1
= x ´ (cos x - sin x)
y dx sin x + cos x

1
+ log e (sin x + cos x)
2 x

(cos x - sin x) 2 1
= x + log e (sin x + cos x)
cos x - sin x 2 x
2 2

1
= x.(sec 2x - tan 2x) + log e y
2x

dy é 1 ù
\ dx = y ê x (sec 2x - tan 2x) + 2x log e y ú
ë û
9.Ans. (A) We have the following :
In – 1 £ x £ 1, g(|x|) = g(x) = – 1 and |g(x)| = 1
In 1 < x £ 2, x – 2 £ 0
Þ |g(x)| = 2 – x, g(|x|) = g(x) = x – 2
In 2 < x £ 3, x – 2 > 0 Þ |g(x)| = x – 2,
g(|x|) = g(x) = x – 2

ì 0, -1 £ x £ 2
\ f(x) = g(|x|) + |g(x)| = í2x - 4 2 < x £ 4
î
At x = 2, LHL = 0 = RHL Þ f(x) is continuous at x = 2

ì0, -1 £ x < 2
Now f’(x) = í 2 2 < x £ 4
î
We have f’(2–) = 0, f’(2+) = 2. As f’(2–) ¹ f’(2+), we see that f(x) is not differentiable at x = 2

MATHEMATICS /AQ # 3 E-79


TM TARGET: JEE (MAIN + ADVANCED) 2020
ENTHUSIAST & LEADER
Path to success KOTA (RAJASTHAN)
COURSE

From Fig C we note that f(x) is continuous at x = 2 but not differentiable there

1 x4
10.Ans. (B) f (g (x) ) = =
æ 1 öæ 1 ö (1 - x 2 )(1 - 2x 2 )
ç 2 - 1 ÷ç 2 - 2 ÷
èx øè x ø

1
Þ f (g (x) ) is discontinuous at x = ± 1, x = ±
2
and x = 0
Since g (x) is discontinuous at x = 0
11.Ans. (C) Given that
x3
cos 2 x - cos x - ex cos x + e x -
lim 2
x®0 xn

x3
(cos x - 1)(cos x - e x ) -
= lim 2
x®0 xn
æ x 2 x4 x6 ö éæ x2 x 2 ö æ x2 x3 öù x3
ç1 - + - + ....... - 1 ÷ êç 1 - + - ...... ÷ - ç 1 + x + + ..... ÷ ú -
è 2! 4! 6! ø ëè 2! 4! ø è 2! 3! øû 2
lim
x® 0 xn

æ x 2 x 4 x6 ö éæ x 3 2x 5 öù x3
ç- + - + ..... ÷ ê ç - x - x 2 - - ....... ÷ ú -
è 2! 4! 6! ø ëè 3! 5! øû 2
lim
x®0 xn

MATHEMATICS /AQ # 3 E-80


TM TARGET: JEE (MAIN + ADVANCED) 2020
ENTHUSIAST & LEADER
Path to success KOTA (RAJASTHAN)
COURSE
æ x 3 x 4 x5 x 5 ö x3
ç + + - ...... ÷ -
2 2 12 24 ø 2
lim è n
x®0 x
= non-zero if n = 4.

é ù
ê ú é r ù
ú = ê lim å sec2 æç ö÷ ú
n
12.Ans. (B) ê lim å2 1 1 n
ê æ 2r ö
1 + cos ç ÷ úú ë
n ®¥ n n r =1
r =1
n ®¥
è n øû
ê n
è øû
ë

é1 ù
= ê ò sec2 x dx ú = [ tan x ] = 1
ë0 û

2cos 4x cos x - 2 cos 6x cos 9x


13.Ans. (B) lim
é ln (1 + sin 4x ) ù
x®0 2
2
2ê ú sin 4x
ë sin 4x û

cos 5x + cos 3x - cos15x - cos 3x


= lim
x ®0 2sin 2 4x
2sin10x sin 5x
= lim
x ®0 2sin 2 4x

sin10x sin 5x 10.5 ( 4x )


2
25
= lim . . . =
x ® 0 10x 5x 16 sin 2 4x 8

sin x
1+
x 1
14.Ans. (B) xlim = =1
®¥ 1/ 4
æ cos x ö æ sin 3x ö
1/ 4
(1)(1)
ç 1 - ÷ ç 1 + ÷
è x ø è x ø

15.Ans. (A) Let xLt f (x) = l


®¥

Þ xLt f (x + 1) = l and Lt f (x + 2) = l
®¥ x ®¥

1 é 7ù 7 7
l= l+ Þ l2 = , l =
5 êë l úû 4 2

16.Ans. (C) Since highest degree of x is 1/2, divide numerator and


2
denominator by x , then we have limit or 2.
2

ì x{sec(x + y) - sec x ü
17.Ans. (A) lim í + sec(x + y) ý
y ®0
î y þ

é x ì cos x - cos(x + y) üù
lim ê í ýú + lim sec(x + y)
ë î cos(x + y) cos x þû y ® 0
y ®0 y

é æ y ö æ y öù
ê x2sin ç x + 2 ÷ sin ç 2 ÷ ú
lim ê è ø è ø ú + sec x
y ®0
ê y cos(x + y) cos x ú
ê ú
ë û
MATHEMATICS /AQ # 3 E-81
TM TARGET: JEE (MAIN + ADVANCED) 2020
ENTHUSIAST & LEADER
Path to success KOTA (RAJASTHAN)
COURSE

é æ yö æ y öù
ê x sin ç x + 2 ÷ sin ç 2 ÷ ú
lim ê è ø ´ è ø ú + sec x
y ® 0 cos(x + y) cos x
ê y ú
ê 2 úû
ë
= x tan x sec x + sec x
= sec x (x tanx + 1)
18.Ans. (A) For n > 1,
lim xn sin(1/x2) = 0 × (any value between – 1 to 1) = 0
x ®0

For n < 0,
lim xn sin(1/x2) = ¥ × (any value between – 1 to 1)
x ®0

19.Ans. (B) Given limit is xlim(x + 1)[tan -1 (x + 5) - tan -1 (x + 1)] + 4 tan -1 (x + 5)


®¥

é 4 ù
= xlim (x + 1) tan -1 + 4 tan -1 (x + 5) ú
®¥ ê 1 + (x + 1)(x + 5)
ë û

é æ 4 ö ù
ê tan -1 ç 2 ÷ ú
= lim ê(x + 1) è x + 6x + 6 ø ´ 4
+ 4 tan -1 (x + 5) ú
x ®¥
ê æ 4 ö x + 6x + 6
2
ú
ê ç 2 ÷ ú
ë è x + 6x + 6 ø û

p
=0+4× = 2p
2
20.Ans. (A) f(x) is continuous when 5x = x2 + 6 Þ x = 2, 3.

21.Ans. (B)

From the graph it is cleat that f(x) is every where


continuous but not differentiable at x = 1 – 2 ,0,1.
sin x lim sin x ln|x|
22.Ans. (B) f(0+) = xlim
®0+
x =e x ®0

lnx
lim
= e x®0 cos ec x = e 0 = 1 (Using L’ Hopital’s Rule)
f(0–) = g(0) = 1
Let g(x) = ax + b
Þ b = 1 Þ g(x) = ax + 1

é sin x ù
For x > 0, f¢(x) = esin x In (|x|) ê cos x In(| x |) +
ë x úû
f¢(1) = 1 [0 + sin 1] = sin 1
f(–1) = – a + 1 Þ a = 1 – sin 1
g(x) = (1 – sin 1) x + 1
MATHEMATICS /AQ # 3 E-82
TM TARGET: JEE (MAIN + ADVANCED) 2020
ENTHUSIAST & LEADER
Path to success KOTA (RAJASTHAN)
COURSE
1 - cos 2(x - 1)
23.Ans. (C) Let E = Lim
x ®1 x -1

2sin 2 (x - 1)
Þ E = Lim
x ®1 x -1

2 | sin(x - 1) |
Þ E = Lim
x ®1 x -1

ì- sin(x - 1) x < 1
| sin(x - 1) |= í
î sin(x - 1) x ³ 1

2|sin(x -1)|
Now, LHL = Lim
x ®1- x -1

- 2 | sin(x - 1) |
Þ LHL = Lim =- 2
x ®1- x -1

2 | sin(x - 1) |
and RHL = Lim
x ®1+ x -1

2 | sin(x - 1) |
RHL = Lim = 2
x ®1+ x -1
Since LHL ¹ RHL

1 - cos 2 x - 1)
Lim
x ®1 x -1

n.3n 1
24.Ans. (C) lim n +1
=
n ®¥ n(x - 2) + n.3 - 3
n n
3

1
Þ lim
n ®¥ (x - 2) 1n

n
+3-
3 n
(Dividing Nr and Dr by n × 3n)

For nlim
®¥
to be equal to 1/3

1
lim ® 0 (which is true)
n ®¥ n
n
æx -2ö
and lim ç 3 ÷ ®0
n ®¥
è ø

x-2
Þ -1 < < 1 Þ -1 < x < 5
3
25.Ans. (A) (1 + x)2/x = (1 + x)2/x – [(1 + x)2/x]

Now, lim(1 + x)2 / x = e 2


x ®0

Þ lim(1 + x) 2 / x = e 2 - [e 2 ] = e 2 - 7
x®0

MATHEMATICS /AQ # 3 E-83


TM TARGET: JEE (MAIN + ADVANCED) 2020
ENTHUSIAST & LEADER
Path to success KOTA (RAJASTHAN)
COURSE
MULTIPLE CORRECT CHOICE TYPE
26.Ans.(B,C) Since the greatest integer function is discontinuous (sensitive) at integral values of x, then for a given limit to exist
both left- and right-hand limit must be equal.

L.H.L. = xlim (2 - x + a[x - 1] + b[1 + x])


®1-

= 2 – 1 + a(–1) + b(1) = 1 – a + b
R.H.L. = xlim(2 - x + a[x - 1] + b[1 + x])
®1+

= 2 – 1 + a(0) + b(2) = 1 + 2b
On comparing we have – a = b
27.Ans.(B, C) Let y = Ae–3x cos x + Be–3x sin x
= e–3x (A cos x + Bsin x)
dy
= e -3x (B cos x – A sin x) – 3 (A cos x + B sin x) e–3x .....(1)
dx
hence e–3x [(B – 3A) cos x – (A + 3B) sin x] = e– 3x cos x
\ B – 3A = 1 .....(2) and A + 3B = 0 ......(3)
squaring and adding (2) and (3)
1
10 (B2 + A2) = 1 Þ A2 + B2 = ]
10
28.Ans.(C,D) for continuity at x = 0
f ( 0 ) = 0; f ( 0- ) = 0; f ( 0+ )

1
= Lim h n sin =0 Þ n > 0
h ®0 h
for derivability at x = 0

1
h n sin
f '(0 -
) = 0; f ' ( 0 ) = Lim
+ h for this limit not to exist n £ 1
h ®0 h
hence 0 < n £ 1
Þ n can not be 3/2 or 2 Þ C, D ]
29.Ans.(B, C, D) For continuity at x = 1
lim f (x) = lim(x 2
sgn[x] + {x}) = 1 + 0 = 1
x ®1+ +
x ®1

lim f (x) = lim(x 2


sgn[x] +{x}) = 1sgn(0) + 1 = 1
x ®1- -
x ®1

Also, f(1) = 1
\ L.H.L. = R.H.L. = f(1).
Hence, f(x) is continuous at x = 1
Now for differentiability
f (1 + h) - f (1)
f¢(1+) = lim
h ®0 h
(1 + h) 2 sgn[1 + h] + {1 + h} - 1
= lim
h ®0 h
(1 + h) 2 + h - 1 h 2 + 3h
= lim = lim
h ®0 h h ®0 h
=3
f (1 - h) - f (1)
and f¢(1–) = lim
h ®0 -h

MATHEMATICS /AQ # 3 E-84


TM TARGET: JEE (MAIN + ADVANCED) 2020
ENTHUSIAST & LEADER
Path to success KOTA (RAJASTHAN)
COURSE

(1 - h) 2 sgn[1 - h] + [1 - h] - 1
= lim
h ®0 -h
0 + 1- h -1
= lim =1
h ®0 -h
f ¢ (1 +) ¹ f ¢ (1–)
Hence, f (x) is non differentiable at x = 1.
Now at x = 2,
lim f (x) = lim- (x 2 sgn[x] + {x}) = 4 ´ 0 + 1
h ® 2- h ®2

lim f (x) = lim+ (sin x + 1| x - 3 |) = 1 + sin 2


h ® 2+ h®2

Hence L.H.L. ¹ R.H.L


Hence, f(x) is discontinuous at x = 2 and then f(x) is also
non-differentiable at x = 2
30. Ans. (A, B) Put x = y = 0, f(0) + f(0) + 0 = f(0) + 0 + 1
Þ f(0) = 1
put x = 2y, f(2y) + f(5y) + 10y2 = f(5y) + 8y2 + 1

x2
f(2y) = 1 – 2y2 Þ f(x) = 1 -
2
graph of f(x)

Þ f is many one, no minima, even and not bounded

ì1- | x |
ï x ¹ -1
31. Ans. (ABCD) Given that, f (x) = í 1 + x
îï 1 x = -1

ì 1 x<0 ì- x x<0
ï Q | x |= í
\ f (x) = í1 - x x³0
ïî1 + x x³0 îx

ì 1 [2x] < 0
Now, f ([2x]) = ïí1 - [2x]
ï1 + [2x] [2x] ³ 0
î

ì 1 x < 0)
ï
ï 1 ([2x] = 0) 0 £ x < 1
ï 2
ï
Þ f ([2x]) = í 1
ï 0 ([2x] = 1) 2 £ x < 1
ï
ï- 1 ([2x] = 2) 1 £ x < 3
ïî 3 2

1 ænö
Clearly, f(x) is continuous for all x < and discontinuous at x = ç ÷ where n Î N
2 è2ø

MATHEMATICS /AQ # 3 E-85


TM TARGET: JEE (MAIN + ADVANCED) 2020
ENTHUSIAST & LEADER
Path to success KOTA (RAJASTHAN)
COURSE
32. Ans. (A,B,C,D) (A) f(x) = x – cos x ; f(0) < 0, f(p/2) > 0
(B) f(x) = x + sin x – 1

p 1
f(0) = – 1 < 0; f(p/6) = + -1 > 0
6 2

(C) f(x) = a (x –3) + b(x – 1) in [1,3]

f(1) = – 2a < 0; f(3) = 2b > 0

Þ f(x) = 0 in (1,3)

(D) h(x) = f(x) – g(x)


h(A) = f(A) – g(A) > 0

h(B) = f(B) – g(B) < 0


hence using IVT all the four have at least one root in indicated interval.]

MATHEMATICS /AQ # 3 E-86


TM TM TARGET:JEE
TARGET: JEE(Main
(MAIN ++Advanced)
ADVANCED) 2020
2020
ENTHUSIAST&
ENTHUSIAST & LEADER
LEADER
Path to success KOTA (RAJASTHAN)
COURSE
Path to success KOTA (RAJASTHAN ) COURSE

ADVANCED QUIZ # 4(SOLUTION) APPLICATION OF DERIVATIVES MATHEMATICS


SINGLE CORRECT CHOICE TYPE
1.Ans. (D) x = a - [ x ]3 Þ x Î I
\ a = x3 + x
2
7 7
æ 7´8 ö 7´8
å a = å r + å r = çè
r =1
3

r =1 2 ÷ø
+
2
= 784 + 28 = 812 Ans. ]
2.Ans. (C) f (1+) = f (1–) = f (1) = 2
f (0) = 1, f (2) = 2
f (2–) = 1; f (2) = 2
Þ f is not continuous at x = 2
3.Ans. (D) f¢ (0) = 0 (given)
g¢ (x) = f¢ (x – h) = 0
hence x – h = 0 Þ x = h
\ g¢ (h) = 0
Þ g has its critical number at x = h Þ (D)
1 1
4.Ans. (B) f ( x ) = x + ; f ' ( x ) = 1 - = 0 Þ x = ±1
x x2

2
f "( x ) = at x = 1 + ve minima.
x3
x = – 1 – ve maxima.

Þ local minimum of f (x) is greater than local maximum


also f (x) has two asymptoes one is y-axis and other is y = x ]

dy 10 11
5.Ans. (D) dx = 11 ; \ mn = -
10
]
(1,3)

6.Ans. (B) g ' (x) = xf ' (2x2 – 1) – xf ' (1 – x2)


if x > 0 : 2x2 – 1 > 1 – x2
æ 2ö æ 2 ö æ 2 ö
Þ 3x2 > 2 Þ x Î çç -¥, - 3 ÷÷ È çç 3 , ¥ ÷÷ Þ x Î çç 3 , ¥ ÷÷
è ø è ø è ø
If x < 0 : 2x2 – 1 < 1 – x2
æ 2 2ö æ 2 ö
3x2 < 2 Þ x Î çç - 3 , 3 ÷÷ Þ x Î çç - 3 ,0 ÷÷
è ø è ø

7.Ans. (D) lny = 1 ìí3lnx + ln ( x 2 + 1) - 1 ln (5 - x ) üý


3î 5 þ

1 1 ìï 3 2x 1 üï
.y ' = í + 2 + ý
y 3 îï x x + 1 5 ( 5 - x ) þï

y' 1 ì 1ü
= í3 + 1 + ý = length of sub-tangent
y 3î 20 þ

MATHEMATICS /AQ # 4 E-87


TM TARGET: JEE (MAIN + ADVANCED) 2020
ENTHUSIAST & LEADER
Path to success KOTA (RAJASTHAN)
COURSE
Paragraph for question Nos. 8 to 10
Sol. [8. A, 9. B, 10. D]
f ( x ) = ax 3 + bx 2 + cx + d

Let f ' ( x ) = k ( x + 2 )( x - 1) = k ( x + x - 2 )
2

1 1
Þ f '' ( x ) = k ( 2x + 1) = 0 when x = - . Hence inflection at x = - Ans. (B)
2 2
integrating
æ x3 x 2 ö
f (x) = k ç + - 2x ÷ + C
è 3 2 ø
f (–2) = 3 and f (1) = 0 gives
k = 2/3 and C = 7/9
hence
2 æ x3 x2 ö 7 2x 3 x 2 4x 7
f (x) = ç + - 2x ÷ + = + - +
3è 3 2 ø 9 9 3 3 9

1
1
é 2 x 4 x 3 4x 2 7x ù
Area = ò f ( x ) dx = ê . + - + ú
-2 ë9 4 9 6 9 û -2

é 1 1 2 7 ù é 8 8 8 14 ù
= ê + - + ú-ê - - - ú
ë18 9 3 9 û ë 9 9 3 9 û
5 38 5 + 76 81 9
= + = = = Ans. (iii) ]
18 9 18 18 2
Paragraph for question Nos. 11 to 13
Sol. [11. D, 12. C, 13. B]
We have h ''(x) = 6x - 4 Þ h '(x) = 3x 2 - 4x + C

As h '(1) = 0 Þ C = 1
So, h '(x) = 3x 2 - 4x + 1 Þ h(x) = x3 – 2x2 + x + k
Also h(1) = 5 Þ k = 5
\ h(x) = x3 – 2x2 + x + 5
Now h¢ (2) = 5
\ The equation of tangent at M(2, 7) to y = h(x), is
(y – 7) = 5(x – 2) Þ 5x – y = 3
Also g(f(x)) = 0 " x Î R
\ Required Area
2
2
é x4 2 x2 ù
= ò h(x) dx = ê - x 3 + + 5x ú
0 ë 4 3 2 û0

16 16 32
= 4- + 2 + 10 = 16 - = Ans
3 3 3
Also range of f(g(x)) = {–1, 0, 1}

MATHEMATICS /AQ # 4 E-88


TM TARGET: JEE (MAIN + ADVANCED) 2020
ENTHUSIAST & LEADER
Path to success KOTA (RAJASTHAN)
COURSE
\ f (g(x)) Î {0,1}

-1 ì pü
Hence range of sin (fog(x)) = í0, ý
î 2þ
Paragraph for question Nos. 14 to 16
Sol. [14. A, 15. B, 16. D]

æ 3 ö æ a2 - a ö
14. (A) f ' ( x ) = 6x 2
+ 6 ç 1 - a ÷ x + 6 ç ÷
è 2 ø è 2 ø
for f (x) to have negative point of local minima
Ģ (x) must have two distinct negative roots
ÞD>0
éæ 3 ö 2 ù
Þ 36 êç 1 - a ÷ - 2 ( a 2 - a ) ú > 0
êëè 2 ø úû
2
æa ö
ç 2 - 1÷ > 0 Þ a ¹ 2 ........(i)
è ø
sum of roots < 0
æ 3 ö
- ç1 - a ÷ < 0
è 2 ø

æ 3 ö 2
ç1 - a ÷ > 0 Þ > a ........(ii)
è 2 ø 3
Product of roots > 0
a2 - a
Þ >0
2
a < 0 or a > 1 ........(iii)
Þ a Î (– ¥, 0)

é 2 æ 3 ö a2 - a ù
15. (B) g ( x ) = ê x + ç 1 - 2 a ÷ x + 2 ú
ë è ø û
2
æa ö
as discriminant of g ( x ) = ç - 1÷ > 0 "
è2 ø
a Î R – {2} Þ y – co-ordinate of parabola y = g (x)
lie below x–axis " a Î R - {2}
16. (D) As y + 1 = 0 touches the parabola y = g (x)
Þ y–co-ordinate of vertex is – 1.
-D
Þ = -1 ÞD=4
4
2
æa ö
ç 2 - 1 ÷ Þ a = -2, 6
è ø
Equation of parabola is
(y + a)2 = (x + 2)2 or (y + 1)2 = (x – 2)2
in both cases equation of locus of point of
intersection of perpendicular tangent
(i.e. directrix) is 4y – 15 = 0

MATHEMATICS /AQ # 4 E-89


TM TARGET: JEE (MAIN + ADVANCED) 2020
ENTHUSIAST & LEADER
Path to success KOTA (RAJASTHAN)
COURSE
MULTIPLE CORRECT CHOICE TYPE
17. Ans. (A,B,C,D) f (x) = x3 – 9x2 + 15x + 6
f (t) = t3 – 9t2 + 15t + 6
f ¢ (t) = 3t2 – 18t + 15 = 3 [t2 – 6t + 5] = 3(t – 5) (t – 1)
hence f is increasing in (5, 6) and f is decreasing in (1, 5)
now

é f ( x ) = x 3 - 9x 2 + 15x + 6 1£ x < 5
ê
g ( x ) = ê f ( 5 ) = -19 5£x£6
ê x - 18 if x >6
ë

é x 3 - 9x 2 + 15x + 6 if 1£ x < 5
ê
\ g ( x ) = ê-19 if 5£x£6
ê x - 18 if x<6
ë
Hence
g is continuous and differentiable at x = 1
g is continuous and differentiable at x = 5
g is neither continuous nor derivable at x = 6 ]
18. Ans. (A,B) y2 = 4x; x2 + y2 = 6x – 1
solving, x2 + 4x = 6x – 1
x2 – 2x + 1 = 0 Þ (x – 1)2 = 0 Þ x = 1
touch at (1,2) and (1,–2) Þ (A) is correct
(B) xy = 4 ; x2 + y2 = 8
x2y2 = 16 Þ x2(8 – x2) = 16; put x2 = t > 0
t2 – 8t + 16 = 0 Þ (t – 4)2 = 0
t = 4 Þ x2 = 4 Þ x = 2 or –2 Þ (2,2) or (–2,–2)
dy
(C) solving the points of intersection are (1,1); (–2,–5); also is same at (1,1).
dx
hence they touch at(1,1)]
19.Ans. (A,C,D) f(x) = x – tan–1 x
1 x2
f¢(x) = 1– = > 0 Þ f is increasing in (0,1)
1 + x2 1 + x2
f(x) > f(0) but f(0) = 0
f(x) > 0 Þ x > tan–1 x in (0,1)
x2
(B) f(x) = cos x – 1 +
2
f¢(x) = – sin x + x = x – sin x> 0 in (0,1)
Þ (B) is not correct

(
(C) f(x) = 1 + x ln x + 1 + x - 1 + x
2 2
)
æ 1 2x ö
ç1+ 2 × ÷
ç 1 + x2 ÷
f¢(x) = x ç x + 1 + x 2 ÷
ç ÷
è ø

(
+ ln x + 1 + x -
2
) x
1+ x2

MATHEMATICS /AQ # 4 E-90


TM TARGET: JEE (MAIN + ADVANCED) 2020
ENTHUSIAST & LEADER
Path to success KOTA (RAJASTHAN)
COURSE

=
1+ x
x
2
(
+ ln x + 1 + x 2 - ) x
1 + x2
> 0"x Î R

Þ (C) is true
x2
(D) f(x) = x – - ln(1 + x)
2
1 (1 - x 2 ) - 1
f¢(x) = (1 – x) – =
1+ x 1+ x
x2
=– < 0 Þ (D) is correct
1+ x
hence f(x) is decreasing in (0,1)
\ f(x) < f(0)
x2
f(x) < 0 Þ x – < ln(1 + x)]
2
20. Ans. (A, C, D) y = (x + 2)1/3
differentiate w.r.t. y
1 1 dx
1= ×
3 (x + 3) dy
2/3

dx dy
dy = 3(x + 2)2/3 = 0 Þ ®¥
x =-2 dx

Also f(–2) = Lim


x ®-2
f(x) = 0
Þ continuous but not differentiable
however a vertical tangent can always be drawn.]

æ 1 ö
21. Ans. (A, B) f¢(x) = 2 – 2x–1/3 = 2 ç1 - x1/ 3 ÷
è ø

Clearly f¢(x) < 0 in (0 + h, 0)


and f¢(x) > 0 in (0 – h, 0)
f¢(x) = 0 Þ x = 1 Þ II

2 1
f¢¢(x) = × Þ concave up
3 x4/ 3
f¢¢(1) > 0 Þ x = 1 is minimum
Hence I and II are correct.]
22. Ans. (B, D) From the given equation f(x) must be linear
\ Let f(x) = ax + b
Then a(x2 + x + 3) + b + 2(x2 – 3x + 5)a + 2b = 6x2 – 10x + 17
compare the coefficient of x2, x and constant terms
a + 2a = 6, a – 6a = –10, 3a + b + 10a + 2b = 17
Þ a = 2, b = –3
Þ f(x) = 2x – 3 Þ B, D
23. Ans. (A, C) Let the tangent line be y = ax + b
The equation for its intersection with the upper parabola is
x2 + 1 = ax + b
x2 – ax + (1 – b) = 0
This has a double root when a2 – 4(1–b) = 0 or a2 + 4b = 4

MATHEMATICS /AQ # 4 E-91


TM TARGET: JEE (MAIN + ADVANCED) 2020
ENTHUSIAST & LEADER
Path to success KOTA (RAJASTHAN)
COURSE
For the lower parabola
ax + b = – x2
x2 + ax + b = 0
This has a double root when a2 – 4b = 0
subtract these two equations to get 8b = 4 or b = 1/4
add them to get 2a2 = 4 or a = ± 2

1 1
The tangent lines are y = 2x + and y = – 2x +
2 2

24.Ans.(A,C) f '(x) = 22 ·2x + 32 ·4x 3 + 6x 5

+...... + n 2 ·(2n - 2)x 2n -3

Þ f '(x) = x[8 + 36x 2 + 96x 4 + n 2 (2n - 2)x 2n -4 ]


Þ f '(x) = x (a positive number)
Sign scheme for f '(x) is

-¥ ¥

Þ f '(x) = 0 at x = 0 only
\ f(x) has one critical point i.e. x = 0 and f(x) has exactly one minimum at x = 0
INTEGER ANSWER TYPE
25. Ans.[0013] The polynomial is an every where differentiable function. Therefore, the points of extermum can only be roots of
the derivative. Furthemore, the derivative of a polynomial is a polynomial. The polynomial of the least degree with roots
x1 = 1 and x2 = 3 has the from a (x – 1) (x – 3).

Hence P ' ( x ) = a ( x - 1)( x - 3) = a ( x - 4x + 3 )


2

since at the point x = 1, there must be P (1) = 6, and P(3) = 2

æ x3 x2 ö
P(x) = ò P '(x) dx = a ç - 4 + 3x ÷ + b
è 3 2 ø

4a
P(1) = 6 Þ +b
3
and P(3) = 2 Þ 0 + b = 2 Þ b = 2
\ a= 3

Hence P ( x ) = x 3 - 6x 2 + 9x + 2.

Now P ( 2 ) = 8 - 24 + 18 + 2 = 28 - 24 = 4

Also P ' ( x ) = 3 ( x - 4x + 3) Þ P ' ( 0 ) = 9


2

\ P ( 2 ) + P' ( 0 ) = 4 + 9 = 13

26.Ans. [0016] For the points of intersection, we have


12 - y 2 y 2
+ = 1 Þ y = ± 3 and x = 3
36 4

Let Consider the point P 3, 3 . ( )


MATHEMATICS /AQ # 4 E-92
TM TARGET: JEE (MAIN + ADVANCED) 2020
ENTHUSIAST & LEADER
Path to success KOTA (RAJASTHAN)
COURSE
Equation of the tangent at P to the circle is

3x + 3 y = 12 \ slope of this tangent is - 3


Equation of the tangent at P to the ellipse is

x 3
+ y =1
12 4

\ slope of this tangent is - 1


3 3
If a is angle between these tangents,

1
- 3+
3 3 = -8 = 2
Þ tanx =
1 4 3 3
1+ 3 ´
3 3

2 2
then tan a = \ a = tan -1
3 3

\ k = 4 and hence k2 = 16

27.Ans. [0000] Let f(x) = – 3 + x – x2. Then f(x) < 0 for all x, because coefficientof x2 is
less than 0 and D < 0. Thus, L.H.S. of the given equation is always positive
whereas the R.H.S. is always less than zero. Hence, there is no solution.

MATCH THE COLUMN :


28. Ans.[A–p, r; B–q, r, s; C–p, q,t ; D–p, q, r, s,t]
(A) (2 2x
- 4.2 x + 4 ) + 1 + b - 1 - 3 = | sin y

(2 - 2 ) + 1 + b - 1 - 3 = sin y
x 2

LHS ³ 1 & RHS £ 1

\ 2x = 2, b - 1 - 3 = 0 Þ ( b - 1) = ±3
x= 1 b = 4, – 2 Þ P, R

ìx 2 x Î éë - 2, 2 ùû
ï
(B) f (x) = í
ïî 2 x Î ( -¥, - 2] È [ 2, ¥)

k Î ( 0, ¥ )
Hence, Q, R, S

(C) Df 0f ( x ) = [ -2,1] È éë 2, 2 3 ùû

Hence P, Q,T.

MATHEMATICS /AQ # 4 E-93


TM TARGET: JEE (MAIN + ADVANCED) 2020
ENTHUSIAST & LEADER
Path to success KOTA (RAJASTHAN)
COURSE

æ 5 ö
(D) f ' ( x ) = 3x 2 + 2px + q < 0 x Î ç - , - 1÷
è 3 ø

Þ 3x 2 + 2px + q = ( 3x + 5 )( x + 1)
Þ 2p = 8 ; p=4 &q =5
Þ p+ q=9 Þ P, Q, R, S ]
29. Ans.[A–R; B–S; C–P]
(
ln e2 x + ex + x )
(A) Lim
l =e x ®¥ x

using L¢Hospital¢s rule


2e 2 x + e x +1 2 + e- x + e-2 x
Lim Lim
l=e x ®¥ e 2 x + e x +1
=e x ®¥ 1+ e - x + e -2 x
= e 2 Ans. Þ (R)

Given ( ln x ) + ( ln y ) = 2 ln x + 2 ln y
2 2
(B)

= ( ln x - 1) + ( ln y - 1) = 2
2 2

let ln x - 1 = 2 cos q Þ ln x = 1 + 2 cos q

and ln y - 1 = 2 sin q Þ ln y = 1 + 2 sin q

\ now x ln y = eln y. ln x
now consider
z = ln x . ln y

(
z = 1 + 2 cos q 1 + 2 sin q )( )
= 1 + 2 ( sin q + cos q ) + sin 2q

æp ö
= 1 + 2 sin ç + q ÷ + sin 2q which has the maximum value when q = p
è4 ø 4
ln x. ln y max = 4

ln y
\ x = e4 Ans. Þ (S)
max

ln (1+ x )

ò (1 - tan 2y )
1/ y
dy
(C) l = Lim 0
x®0 sin x
.x
x
Using L¢Hosptial¢s Rule

1
éë1 - tan 2 ( ln (1 + x ) ) ùû
ln (1 + x )
l = Lim
x®0 (1 + x )
(1 )
¥

1 tan ( 2 ln (1+ x ) )
- Lim tan 2 ( ln (1+ x ) ) - Lim .2
x ®0 l n (1+ x ) 2 ln (1+ x )
=e =e
x ®0

æ tan q ö
= e -2 ç using Lim = 1÷ Ans. Þ (P)]
è x ®0 q ø

MATHEMATICS /AQ # 4 E-94


TM TM TARGET:JEE
TARGET: JEE(Main
(MAIN ++Advanced)
ADVANCED) 2020
2020
ENTHUSIAST&
ENTHUSIAST & LEADER
LEADER
Path to success KOTA (RAJASTHAN)
COURSE
Path to success KOTA (RAJASTHAN ) COURSE

ADVANCED QUIZ # 5 (SOLUTION) INTEGERAL CALCULUAS PART -1 MATHEMATICS


SINGLE CORRECT CHOICE TYPE
dy
1. Ans. (A) - y = 1 - e- x , I. F. = e–x
dx

\ y.e = ò ( e - e ) dx
-x -x -2 x

1
y.e - x = -e - x + e-2x + C
2
if x = 0, y = y 0

1 1
y0 = -1 + + C Þ C = y 0 +
2 2
1 1
\ y.e - x = -e- x + e-2x + y0 +
2 2
1
if x ® ¥, then y0 = - Ans. ]
2
2.Ans. (D)
0 1

A = ò é 2 - x 2 - x 2 ù dx + ò é 2 - x 2 - x ù dx
-1
ë û 0
ë û

p
=
2

note that the area is equal to the sector AOB with central angle 90°

Þ 1/4 (the area of the circle)


p p
required area p - =
2 2
p/3

3. Ans. (C) I = ò tan


{ x . ln ( cos x ) dx
0
14 4244 3
II I

p/3
p/3
= -ln ( cos x ) .ln ( cos x ) ùû 0 - ò0 tan x.ln ( cos x ) dx
1444 24443
I

0
2I = é ln ( cos x ) ù
2
ë ûp / 3 = ( 0 ) - ( ln 2 2 ) = -ln 2 2

ln 2 2
I=-
2
4.Ans. (A) I = ò x x ( ln ex ) dx = ò x x (1 + ln x ) dx

dt
Let t = x = e
x x ln x
Þ = x x (1 + ln x ) dx
dx
Þ I = ò dt = t + C = x x + C

MATHEMATICS /AQ # 5 E-95


TM TARGET: JEE (MAIN + ADVANCED) 2020
ENTHUSIAST & LEADER
Path to success KOTA (RAJASTHAN)
COURSE

1 + 2 + 3 + ..... + 6n
5.Ans. (D) l = Lim
n ®¥
n n

6 6
1 6n r é2 ù
= Lim å = ò x dx = ê x 3/ 2 ú
n ®¥ n n 3
r =1 0 ë û0

2
= .6 6 = 96
3
Þ N = 96

1 + t 8 .2t 2 1 + t 8
6.Ans. (C) f ' ( t ) = = ..... (1)
t2 t

now F ( x ) = ò f ( t ) dt Þ F ' ( x ) = f ( x )
1

F" ( x ) = f ' ( x ) Þ F" ( 2 ) = f ' ( 2 )


from (1)
f ' ( 2 ) = 256 + 1 = 257

2 tan x
7.Ans. (C) Using sin 2x =
1 + tan 2 x

2 tan x
p/ 2 1-
1 + tan 2 x dx
I= ò0 æ 2 tan x ö
2

ç1 + ÷
è 1 + tan x ø
2

(1 - tan x )
p/2 2

= ò0 (1 + tan x )4 .(1 + tan x ) dx


2

(1 - tan x )
p/2 2

= ò0 (1 + tan x )4 .sec x dx
2

put y = tan x Þ dy = sec2 x dx

(1 - y )
¥ 2

\ I=ò dy now put 1 + y = z Þ dy = dz


0 (1 + y )
4

( 2 - z)
¥ 2 ¥
3z 2 - 6z + 4
\ I=ò dz = -
1
z4 3z3 1

1
= Þ a = 1, b = 3 Þ 1 + 3 + 3 = 7
3

( cos x - sin x )
p/ 2 2

Alternatively: I = ò0 ( cos x + sin x )4 dx

1 p/ 2 æ d æ 1 öö
I= ò (14
cos x - sin x ) . ç ç
4244
÷ ÷ dx
3 ç dx ç ( cos x + sin x ) ÷ ÷
3
3 0 è øø
I è1444424444 3
II

integrating by parts

MATHEMATICS /AQ # 5 E-96


TM TARGET: JEE (MAIN + ADVANCED) 2020
ENTHUSIAST & LEADER
Path to success KOTA (RAJASTHAN)
COURSE

é
( sin x + cos x ) ùú
p/ 2
1 ( cos x - sin x ) p/ 2
=- ê + ò0 ( cos x + sin x )3 dx ú
3 ê ( cos x + sin x )3
ë 0 û

1é p/ 2
dx ù
= - ê{( -1) - (1)} + ò ú
3ë 0
1 + sin 2x û

2 tan x
using sin 2x =
1 + tan 2 x

2 1 p / 2 sec 2 x
3 3 ò0 (1 + tan x )2
= - dx

¥
2 1 dt 2 1 t ¥ 2 1
= + é t ù = + [( 0 ) - (1) ]
3 3 ò1 t 2 3 3 ë û 0 3 3
= -

2 1 1
= - = Þ a = 1, b = 3 Þ 1 + 3 + 3 = 7
3 3 3

p/ 2

8.Ans. (B) I = 4 ò ( sin 2 2x - sin 4 2x ) dx;


0

dt
2x = t Þ dx =
2
hence
p p/ 2

2ò ( sin 2 t - sin 4 t ) dt = 4 ò ( cos 2 t - sin 4 t ) dt;


0 0

é p 3p ù 3p p
4ê - ú = p- =
ë 4 16 û 4 4
3/ 2
dx
9.Ans. (A) I = ò
1 + 1 - ( x - 1)
2
1/ 2

put x - 1 = sin q
dx = cos q dq
p/ 6 p/ 6
cos q dq æ 1 ö
= ò
-p / 6
1 + cos q
=2ò
0
ç 1 - 1 + cos q ÷ dq
è ø

qù ù
p/ 6
é p p/ 6 dq ù ép 1
= 2ê - ò ú = 2 ê - .2 tan ú
êë 6 0 2 cos ( q / 2 ) úû
2
ë6 2 2 ûú 0 ûú

ép pù p p
= 2 ê - tan ú = = - 2 tan
ë6 12 û 3 12

1 1 1 1 1
10. Ans.(A) Tr = = = = ×
n× r r ×n r 2 n r/n
×n
n

1 4n 1 dx
( x)
4 4
hence Sum = å =ò =2 = 4 Ans.]
n r =1 r / n 0 x 0

MATHEMATICS /AQ # 5 E-97


TM TARGET: JEE (MAIN + ADVANCED) 2020
ENTHUSIAST & LEADER
Path to success KOTA (RAJASTHAN)
COURSE
11.Ans. (A) Simple
12.Ans. (B) Applying Leibnitz reul, we get
f(x) = x – x2 f(x)
x
f (x) =
1+ x2
1 1
x x 1 8
ò
-1/ 2
1 + x2
dx = ò 1+ x
1/ 2
2
dx = In
2 5

1 2n æ r ö æ r2 ö
13.Ans. (C) y = nlim å ç ÷.sin ç 2 + 1÷
n r =1 è n ø
®¥
èn ø
2

Þ ò x sin(x + 1)dx = sin 2 × sin 3


2

14.Ans. (A)
(e2x - 1)dx e-2x - e-4x
I=ò =ò dx
e4x 2 - 2e -2x + e -4x 2 - 2e -2x + e-4x
Put t2 = 2 – 2e–2x + e–4x
2t dt = (4e–2x – 4e–4x) dx
1
t dt = (e–2x – e– 4x) dx
2
1 t dt
2 1 1
\ I= ò t
= t=
2 2
2 - 2e-2x + e-4x + C

x sin x + cos x æ x cos x (x sin x - cos x) ö


4 3

15.Ans. (C) I = ò e ç 2 2
-
x 2 cos 2 x
÷ dx
è x cos x ø

I = ò x.e(x sin x + cos x ) × x cos xdx

(x sin x - cos x)
- ò e( x sin x +cos x ) × dx
x 2 cos 2 x

I = xex sin x + cos x –


1
òe x cos x ò
x sin x + cos x
dx - ex sin x + cos x × + e x sin x + cos x dx

æ 1 ö
I = ex sin x + cos x ç x - ÷+c.
è x cos x ø

p 3
+ cos -1 x
4

16.Ans. (D) I = ò 2 dx
-1
1 p + cos x + cos -1 (1 - x)
4

3 p 3
4 + cos -1 (1 - x) 4
p + cos-1 x + cos-1 (1 - x)
2
=ò -1
1 p + cos (1 - x) + cos
-1
x
dx Þ 2I = ò p + cos
1
-1
x + cos-1 (1 - x)
dx
4 4

3 1 1 1
= [x]1/ 4 = - = ÞI =
3/ 4

4 4 2 4

MATHEMATICS /AQ # 5 E-98


TM TARGET: JEE (MAIN + ADVANCED) 2020
ENTHUSIAST & LEADER
Path to success KOTA (RAJASTHAN)
COURSE
17. Ans.(A) x2 + y2 – 6y £ 0
centre (0,3) radius 3
x2 = 3y

9x æ 3
x2 ö
\ area = 2 - ç
è
18 - 2 ò0 3 dx ÷ø
3
9p x3 9p 9p
= - 18 + 2 = - 18 + 6 = - 12
2 9 0
2 2

18.Ans. (B) Let (a, b) be the centre of required circle


Þ x2 + y2 – 2ax – 2by + g = 0 is orthogonal to x2 + y2 = k2
Þ g = k2
Þ x2 + y2 – 2ax – 2by + k2 = 0 is passing through (a,b)
Þ locus of (a,b) is (2ax + 2by) = a2 + b2 + k2

1 (a 2 + b 2 + k 2 ) 2
Þ area made by above line and coordinate axes is :
2 4ab

dy dy
19. Ans.(B) The given equation is (3x + ) (y + 2x )=0
dx dx

dy 3x 2
\ either 3x + = 0 Þy=– +2
dx 2
Þ 3x2 + 2y – 4 = 0
dy dy dx
or y + 2x =0 Þ2 y + =0
dx x
1
xy2 = C Þ xy2 = Þ 4.xy2 – 1= 0
4
20.Ans. (C) sin3 x sin (x + a)
= sin3 x(sin x cos a + cos x sin a)
= sin4 x(cos a + cot x sin a)
1
I=ò dx
sin x sin(x + a )
3

1
= ò sin 2
x cos a + cot x sin a
dx

cos ec 2 x
= ò cos a + cot x sin a
dx

Putting cos a + cot x sin a = t and – cosec2 x sin adx = dt,


we have

1 1
I=ò -
sin a ò
dt = - t -1/ 2 dt
sin a t

-1 æ t1/ 2 ö
= ç ÷+C
sin a è 1/ 2 ø

Þ I = -2 cos ec a t + C

= -2 cos eca (cos a + cot x sin a)1/ 2 + C

MATHEMATICS /AQ # 5 E-99


TM TARGET: JEE (MAIN + ADVANCED) 2020
ENTHUSIAST & LEADER
Path to success KOTA (RAJASTHAN)
COURSE
Paragraph for question Nos. 21 to 22
Sol. [21. A , 22. D]
a+ b =1 ....(1)
2a2 + 2b2 = 1 ....(2)
By (1) and (2)
2a2 + 2(1 – a)2 = 1
Þ 2a2 + 2(1+ a2 – 2a) = 1 Þ 4a2 – 4a + 1 = 0
1 1
Þ (2a – 1)2 = 0 Þ a = \b =
2 2

a
\q= =1
b

Again f(x + 2) + f(x) = 2 "x Î [0, 2]


4

Now p = ò f (x)dx - 4
0

2 4

= ò f (x)dx + ò f (x)dx - 4
0 2

[By putting x = t + 2 for second integral]


2 2 2
= ò f (x)dx + ò f (x + 2)dx - 4 = ò (f (x) + f (x + 2))dx - 4
0 0 0

= ò 2dx - 4 = 4 - 4 = 0 = p
0

MULTIPLE CORRECT CHOICE TYPE


23. (*)
x

24. Ans. (A, B, C, D) Given that f (x) = ò | t - 1| dt


0

x 2
Þ f (x) = ò (1 - t)dt,0 £ x £ 1 = x - x
0 2
1 x

Also f (x) = ò (1 - t)dt + ò (t - 1)dt , where 1 £ x £ 2


0 1

2 2
1 x 1 x
= + -x+ = - x +1
2 2 2 2

ì x2
ïï x - , 0 £ x £ 1
Thus, f (x) = í 2 2
ï x - x +1 1 < x £ 2
ïî 2

ì1 - x, 0 £ x £ 1
Þ f '(x) = í
î x - 1, 1 < x £ 2
Thus, f(x) is continuous as well as differentiable at x = 1

MATHEMATICS /AQ # 5 E-100


TM TARGET: JEE (MAIN + ADVANCED) 2020
ENTHUSIAST & LEADER
Path to success KOTA (RAJASTHAN)
COURSE
25.Ans.(A, B) Let g(x) = tan–1x – f(x) ......(1)

1
Diff. (1) w.r.t. x, we get g '(x) = - f '(x)
1+ x2

ex 1 - ex
Also f '(x) = Þ g '(x) = < 0 "x > 0
1+ x 2
1+ x2
Þ g(x) is a decreasing function for x > 0
Þ g(x) £ g(1) " x ³ 1

p
g(1) = [Q f (1) º 0]
4

p
Þ g(x) £ p / 4 " x ³ 1 or tan–1x – f(x) £
4

p
Þ f (x) ³ tan -1 x - "x ³1
4

INTEGER ANSWER TYPE


1
26. Ans. [0208] Let I = ò 207 C7 × x{
200
× (1 - x) 7 dx
1424
0
3 II
I

é 1
ù
ê x 201
7 1 ú
I = C7 ê(1 - x) × + ò (1 - x) 6 × x 201dx ú
207 7

ê 14 4244 201
3 201 0 ú
êë zero 0 úû

7 1
C7 × ò (1 - x) × x dx
207 6 201
=
201 0
I.B.P. again 6 more times

7! 1

C7 ×
207
= 207
ò x dx
201 × 202 × 203 × 204 × 205 × 206 × 207 0

(207)! 7! 1
= 7!(200)! × 201 × 202 × ....... × 207 × 208

(207)! 7! 1 1
= (207)!7! × 208 = 208 = k Þ k = 208 Ans.]

1
-4
27.Ans. [0006] I = ò (x - x ) × x dx
3 1/ 3

1/ 3

1 1
I = ò x(x -2 - 1)1/ 3 × x -4 dx = ò (x -2 - 1)1/ 3 × x -3dx
1/ 3 1/ 3

put x–2 – 1 = t3 Þ –2x–3 dx = 3t2 dt


if x = 1 then t = 0
if x = 1/3 then t = 2
2
32 3 t4 3
\ I = ò t ´ t dt = + ×
2
=+ [16] = +6 Ans.
20 2 4 0
8

MATHEMATICS /AQ # 5 E-101


TM TARGET: JEE (MAIN + ADVANCED) 2020
ENTHUSIAST & LEADER
Path to success KOTA (RAJASTHAN)
COURSE
p/ 2 p/2
I = ò 2 ln(sin x)dx - ò ln(sin 2x)dx
28.Ans. [0002] Let 1440
2443 144 0
2443
J K

p/ 2

Consider K = ò ln(sin 2x)dx


0

(put 2x = t Þ dx = dt /2)
1p 1 p/ 2
K= ò ln(sin t)dt = × 2 ò ln(sin t)dt
20 2 0

p/ 2 p/ 2

K= ò ln(sin t)dt = ò ln(sin x)dx;


0 0

p/ 2 p/ 2

hence I = 2 ò ln(sin x)dx - ò ln(sin x)dx


0 0

p/ 2

I= ò ln(sin x)dx ....(1)


0

p/ 2

; also, I = ò ln(cos x)dx .....(2)


0

add (1) and (2)


p/ 2
æ sin 2x ö
2I = ò ln ç ÷ dx
0 è 2 ø
p/ 2
p
Þ 2I = ò ln(sin 2x)dx - 2 ln2
0

p
\ I = - ln2 Ans.]
2

29.Ans. [0003] - x 2 + 10x - 16 < x - 2


We must have – x2 + 10x – 16 ³ 0
Þ x2 – 10x + 16 £ 0 Þ 2£ x £8
Also,
Right hand side is non negative for x > 2
\ for x > 2 then
– x2 + 10x – 16 < x2 – 4x + 4
Þ 2x2 – 14x + 20 > 0 Þ x2 – 7x + 10 > 0
Þ x > 5 or x < 2 (2)
From (1) and (2),
5 < x £ 8 Þ x = 6, 7,8
30.Ans. [0001] Equation of normal is
1
Y – y= – (X - x)
m

X + mY – (x + my) = 0 .....(1)
Perpendicular distance from (0,0) to equation (1) is

x + my
=y
1+ m2

MATHEMATICS /AQ # 5 E-102


TM TARGET: JEE (MAIN + ADVANCED) 2020
ENTHUSIAST & LEADER
Path to success KOTA (RAJASTHAN)
COURSE

y2 - x 2 dy
Þ (x + my)2 = y2 (1 + m2) Þ x2 + 2mxy = y2 Þ m = Þ 2xy = y2 – x2 . ...(2)
2xy dx

Put y2 = t Þ 2y dy = dt
dx dx
dt dt 1
\ Equation (2) becomes x = t - x2 Þ - t = -x
dx dx x
1
- ò dx
- ln x 1
\ I.F. = e x = e =
x
Now general solution is given by
æ1ö æ1ö
tç ÷ = – x + C Þ y2 ç ÷ = – x + C
èxø èxø
As (1,1) satisfy it , so C = 2
Þ y2 = – x2 + 2x Þ x2 + y2 – 2x = 0
kp
hence required area =
2
\ k = 1 Ans.]

a-x
31.Ans. [0023] ay 2 = x 2 ( a - x ) Þ y = ± x
a

a
Area = 2 x a - x dx
ò0 a
(put x = a cos q,dx = -a sin q dq )
p/ 2
q
= 2 ò a cos q 2 sin a sin q dq
0
2
p/ 2
æ qö q q
= 2 2 a2 ò çè1 - 2sin 2sin 2 cos d q
2

0
2 ÷ø 2 2

q q
put sin = t,cos dq = 2dt
2 2
1/ 2
= 8 2 a2 ò (1 - 2t ) t
2 2
dt
0

1/ 2
= 8 2 a2 ò (t - 2t 4 ) dt
2

1/ 2
æ t 3 2t 5 ö æ 1 2 ö
= 8 2 a2 ç - ÷ = 8 2 a2 ç - ÷
è3 5 ø0 è 6 2 20 2 ø
2
æ 1 1 ö 8a
= 8a 2 ç - ÷ =
è 6 10 ø 15
4

32. [0005] We have ò (3 - f (x))dx = 7


2

4 4

Þ 6 - ò f (x)dx = 7 Þ ò f (x)dx = -1
2 2

MATHEMATICS /AQ # 5 E-103


TM TARGET: JEE (MAIN + ADVANCED) 2020
ENTHUSIAST & LEADER
Path to success KOTA (RAJASTHAN)
COURSE
Now,

-1 2
é4 2
ù
ò2 f (x)dx = - ò-1 f (x)dx = - ê ò
ë -1
f (x)dx + ò
4
f (x)dx ú
û

é4 4
ù
= - ê ò f (x)dx - ò f (x)dx ú = -[4 + 1] = -5 Þ |–5| = 5
ë -1 2 û

MATCH THE COLUMN :


33. Ans. [A – q, B – s, C – p, D – r]
p p
(A) If I = ò0 xln sin x dx = ò0
( p - x)ln sin x dx

p
= pò ln sin x dx - I
0

p p æ p ö
Hence I = ò ln sin x dx = p ç - ln2 ÷
2 0
è 2 ø

-p2
= ln2.
2
Putting x = tan q. It can be written as
p/ 2 p/2
ò
0
ln(tan q + cot q)dq = ò
0
(ln2 - ln(sin 2q))dq

p p/ 2 p p
= ln2 - ò ln sin qdq = ln2 + ln2 = pln2
2 0 2 2
p/ 4
(C) I= ò 0
ln(1 + tan x)dx

p/ 4 æ 1 - tan x ö
= ò ln ç1 + ÷ dx.
0
è 1 + tan x ø

æ pö p/ 4
= ç ÷ ln2 - ò0 ln (1 + tan x ) dx
è4ø

æpö
Þ I = ç ÷ ln2
è8ø

p/ 2 p
(D) Use ò 0
ln sin x = - ln2 get (d).
2

MATHEMATICS /AQ # 5 E-104


TM TM TARGET:JEE
TARGET: JEE(Main
(MAIN ++Advanced)
ADVANCED) 2020
2020
ENTHUSIAST&
ENTHUSIAST & LEADER
LEADER
Path to success KOTA (RAJASTHAN)
COURSE
Path to success KOTA (RAJASTHAN ) COURSE

ADVANCED QUIZ # 6(SOLUTION) INTEGERAL CALCULUAS PART -2 MATHEMATICS


SINGLE CORRECT CHOICE TYPE

1. Ans. (B) Here , ò x 5 (1 + x 3 ) 2 / 3 dx


Let 1 + x3 = t2 and 3x2dx = 2t dt

\ ò x 5 (1 + x 3 ) 2 / 3 dx

2
ò 3ò
= x 3 (1 + x 3 )2 / 3 x 2 dx = (t 2 - 1)(t 2 ) 2 / 3 tdt

2 2
3ò 3ò
= (t 2 - 1)t 7 / 3dx = (t13 / 3 - t 7 / 3 )dx

2 ì 3 16 / 3 3 10 / 3 ü
3 ò î16
= í t - t ý+C
10 þ
1 1
= (1 + x 3 )8 / 3 - (1 + x 3 )5 / 3 + C
8 5

(1 - cos q) 2 / 7
2. Ans. (A) Let I = ò dq
(1 + cos q)9 / 7

(2sin 2 q / 2) 2 / 7 1 (sin q / 2) 4 / 7
I= ò (2 cos q / 2)
2 9/ 2
dq = ò
2 (cos q / 2)18 / 7
dq

q dq
Put = t\ = dt
2 2
(sin t)4 / 7
Þ I=ò dt (Here m + n = –2)
(cos t)18 / 7

= ò (tan t) 4 / 7 sec 2 tdt


Put tan t = u \ sec2t dt = du
u11/ 7 7
Þ I = ò u du = + c = (tan t)11/ 7 + C
4/7

11/ 7 11
11/ 7
7æ qö
= tan ÷ +C
11 çè 2ø

x 9 dx dx
3.Ans. (D) I = ò =ò 6
(x 2 + 4)6 æ 1 ö
x3 ç 4 + 2 ÷
è x ø

æ 1 ö
dç 4+ 2 ÷
1 è x ø

=- 1 dt
ö = - 2 ò t6
6
æ 1
ç4+ 2 ÷
è x ø

-5
æ 1 ö
4+ 2 ÷
1 çè
-5
x ø 1æ 1 ö
=- + c = ç4+ 2 ÷ +c
2 -5 10 è x ø

MATHEMATICS /AQ # 6 E-105


TM TARGET: JEE (MAIN + ADVANCED) 2020
ENTHUSIAST & LEADER
Path to success KOTA (RAJASTHAN)
COURSE

æ ö -1
4.Ans. (C) I = ò e (1 + x + x ) ´ ç 1 + x 2 ÷ dx
-1
tan x 2

è ø

æ x ö
= - ò e tan
-1

ç1 +
x
÷ dx
è 1+ x
2
ø
-1
e tan x
= - ò e tan x dx - ò x
-1
dx
1 + x2

= - ò e tan x dx - xe tan + ò e tan


-1 -1 -1
x x
dx + c = - xe tan -1 x + c

3 + 2 cos x
5.Ans. (A) Let I = ò (2 + 3cos x) 2 dx , Multiplying Nr and Dr by

cosec2x, we get

(3cos ec2 x + 2 cot x cos ec x)


Þ I=ò dx
(2cos ec x + 3cot x) 2

-3cos ec 2 x - 2cot x cos ecx


= -ò dx
(2cos ecx + 3cot x) 2

1 æ sin x ö
= +C = ç ÷+C
2 cos ecx + 3cot x è 2 + 3cos x ø

æ ö p
6.Ans. (A) For x Î ç - 3 ,0 ÷ , 2 cos x - 1 > 0
è ø
0
æ -1 æ 2 ö -1 æ 2 öö
Þ I= ò ç cot ç ÷ + tan ç ÷ ÷ dx
-p /3 è è 2 cos x - 1 ø è 2 cos x - 1 ø ø

0
p p2
= ò 2
-p / 3
dx =
6

dx
7.Ans. (C) = sin -1 (sin t) cos t = t cos t
dt

dy sin t 1
and = .
dt t 2 t

sin t dy sin t tan t


= Þ = =
2t dx 2t t cos t 2t 2

x x
8.Ans. (A) cos x - x ò f (t)dt + ò tf (t)dt
0 0

f '(x) = - sin x - ò f (t)dt - xf (x) + xf (x)


0

f "(x) = - cos x - f (x)


f "(x) + f (x) = - cos x

MATHEMATICS /AQ # 6 E-106


TM TARGET: JEE (MAIN + ADVANCED) 2020
ENTHUSIAST & LEADER
Path to success KOTA (RAJASTHAN)
COURSE
p p
9.Ans. (D) Since, f(x) = sin3x + l sin2x , x Î æç - , ö÷
è 2 2ø

Now, f '(x) = 3sin 2 x cos x + 2l sin x cos x

æ3 ö æ3 ö
Þ f '(x) = 2sin xcox ç sin x + l ÷ Þ f '(x) = sin 2x ç sin x + l ÷
è2 ø è2 ø

Þ f '(x) = 0 only x = 0

3 3
sin x + l > 0 " x Î R Þ l > - sin x
2 2

or l Î æç - 3 ,0 ö÷ È æç 0, 3 ö÷
è 2 ø è 2ø

10.Ans. (C) Let I = ò f (x)g ''(x)dx - ò f ''(x)g(x)dx


Integrating by parts, we have
I = f ( x ) g '( x ) - ò f '( x ) g '( x ) dx - f '( x ) g ( x ) + ò f '( x ) g '( x ) dx

Þ I = f (x)g '(x) - f '(x)g(x)


\ I = f (1)g '(1) - f '(1)g(1)
11.Ans. (C) Since, cot -1 x Î ( 0, p ) " x Î R

ì3 x Î (-¥,cot 3)
ï2 x Î (cot 3,cot 2]
\ cot -1 x = ïí
ï1 x Î (cot 2,cot1]
ïî0 x Î(cot1, ¥)

20
Now, let I = ò [cot -1 x]dx
-10

cot 3 cot 2 cot1 ¥


ÞI= ò 3.dx + ò 2.dx + ò 1.dx + ò 0.dx +
-10 cot 3 cot 2 cot1

= 3cot 3 + 30 + 2 (cot 2 – 3) + cot 1 – cot2 + 0

= 30 + cot 1 + cot 2 + cot 3

1
12.Ans. (D) y = (x - 1) 2 + 1

Area
¥
1
= 2ò dx = 2[tan -1 (x - 1)]1¥ = p sq.units
1 (x - 1) 2
+ 1
MATHEMATICS /AQ # 6 E-107
TM TARGET: JEE (MAIN + ADVANCED) 2020
ENTHUSIAST & LEADER
Path to success KOTA (RAJASTHAN)
COURSE
p
n
13.Ans. (B) Since, In = ò x sin xdx
0

Using Reduction formula, we have


p é p ù
In = [- x n cos x]0p + n ò x n -1 cos xdx Þ I n = p n + n ê (x n -1 sin x) 0p - (n - 1) ò x n - 2 sin xdx ú
0
êë 0 úû

Þ I n = pn - n(n - 1)I n -2
At n = 5 : I5 = p5 – 5.4I3
\ I5 + 20I3 = p5
14. Ans.(C) Here, y = f[g(x)] = f(cos x) = cos2x
Again, 18x2 – 9px + p2 = (3x – p) (6x – p) = 0

p p
Þ x= , Hence, the required area is
3 6

p p
3 3
1
A = ò cos 2 xdx =
2 pò
(1 + cos 2x)dx
p
6 6

p
1æ sin 2x ö 3 1 é p 3ù 1 ép 3ù p
ÞA = çx + ÷p = ê + ú- ê + ú
2è 2 ø 2 ë 3 4 û 2 ë 6 4 û = 12
6

15.Ans. (C) –8 < x < 8 Þ y = 2

1
\ Required area ; A = (1 + 3) ´ 2
2
Þ A = 4sq. unit
16.Ans. (B) Since, an–1y = xn
Differentiating, we have

dy n -1 dx
a n -1 = nx n -1 Þ a = nx n -1
dx dy

æ dy dx ö
Puting this value in the given equation, we have ç put ®- ÷
è dx dy ø

n -1 dy ny dx
– nx .y = x n Þ =- Þ nydy + xdx = 0
dx x dy

\ ny2 + x2= constant

MATHEMATICS /AQ # 6 E-108


TM TARGET: JEE (MAIN + ADVANCED) 2020
ENTHUSIAST & LEADER
Path to success KOTA (RAJASTHAN)
COURSE
17. Ans.(A) Clearly t can be any real number
1 - tan 2 q
Let t = tan q Þ x =
1 + tan 2 q
Þ x = cos 2q, and

2 tan q
y= = sin 2q
1 + tan 2 q

Þ x 2 + y2 = 1

Thus, required area = p sq. units


Paragraph for question Nos. 18 to 20
Sol [18-D, 19-A, 20-C]
x 2
x2
ò f (t)dt =
2
2 òx
+ t 2 f (t)dt

Differentiating w.r.t. x, we get

f(x) = x + (–x2f(x))
x
Þ f(x) [1 + x2] = x Þ y = f(x) =
1 + x2
Þ yx2 – x + y = 0
Since x is real, D ³ 0
é 1 1ù
Þ 1 – 4y2 ³ 0 Þ y Î ê- , ú
ë 2 2û
2

Also f(x) is an odd function, hence ò f (x)dx


-2

1 + x 2 - 2x 2 1 - x 2
f '(x) = = ³0
1+ x2 1+ x2
Þ x2 – 1 £ 0 Þ x Î [ -1,1]
Paragraph for question Nos. 21 to 23
Sol [21-B, 22-D, 23-C]
p/2 p/ 2

f(x) = sinx + sinx ò


-p / 2
f (t)dt + cos x ò
-p / 2
tf (t)dt

æ p/ 2
ö p/ 2
= sin x ç1 + ò f (t)dt ÷ + cos x ò tf (t)dt
è -p / 2 ø -p / 2

= A sin x + B cos x
p/ 2

Thus, A = 1 + ò
-p / 2
f (t)dt

p/ 2 p/ 2
=1+ ò
-p / 2
(A sin t + B cos t)dt = 1 + 2B ò
-p / 2
cos tdt

MATHEMATICS /AQ # 6 E-109


TM TARGET: JEE (MAIN + ADVANCED) 2020
ENTHUSIAST & LEADER
Path to success KOTA (RAJASTHAN)
COURSE
Þ A = 1 + 2B ........(1)
p/2 p/ 2

B= ò
-p / 2
tf (t)dt = ò
-p / 2
t(A sin t + B cos t)dt

p/ 2

= 2A ò t sin tdt
0
= 2A[ - t cos t + sin t]p0 / 2

Þ B = 2A .......(2)
From equations (1) and (2), we get
A = –1/3, B = –2/3
1
Þ f (x) = - (sin x + 2cox)
3

é 5 5ù
Thus, the range of f(x) is ê - 3 , 3 ú
ë û
1
f(x) = - (sin x + 2 cos x)
3

5 5 æ 1ö
=- sin(x + tan -1 2) =- cos ç x - tan -1 ÷
3 3 è 2ø

p p
f(x) is invertible if - £ x + tan -1 2 £
2 2
p -1 p -1
Þ - - tan 2 £ x £ - tan 2
2 2

-1 1
or 0 £ x - tan £p
2
-1 1 1
Þ tan £ x £ p + tan -1
2 2
-1 1
or p £ x - tan £ 2p
2
Þ x Î [p + cot -1 2, 2p + cot -1 2]
p/ 2 p/ 2
1
ò
0
f (x)dx = -
3 ò (sin x + 2cos x)dx
0

1
= - [- cos x + 2sin x]p0 / 2 = –1
3
Paragraph for question Nos. 24 to 26
Sol. [24.C, 25.D, 26. C]
p/ 2
f (x) - l ò sin x cos t f (t)dt = sin x
0

p/ 2
Þ f(x) – l sin x ò0 cos t f (t)dt = sin x
Þ f(x) – A sin x = sin x or
f(x) = (A + 1) sinx, where A = l
p/ 2
Þ A = lò cos t(A + 1) sin t dt
0

p/2
l (A + 1) p / 2 l(A + 1) é - cos 2t ù
=
2 ò0
sin 2tdt =
2 ê 2 ú
ë û0

MATHEMATICS /AQ # 6 E-110


TM TARGET: JEE (MAIN + ADVANCED) 2020
ENTHUSIAST & LEADER
Path to success KOTA (RAJASTHAN)
COURSE
l (A + 1) l
= ÞA =
2 2+ l

æ l ö
Þ f (x) = ç + 1÷ sin x ( l ¹ 2 )
è 2-l ø

æ 2 ö
Þ f (x) = ç ÷ sin x ( l ¹ 2 )
è2-l ø

æ 2 ö
ç ÷ sin x = 2 ( l ¹ 2 )
è2-l ø
Þ sinx = (2 – l) Þ|2 – l| £ 1
Þ -1 £ l - 2 £ 1 Þ1£ l £ 3 ( l ¹ 2)
p/ 2 p/ 2
2 é 2 ù
Þ ò
0
2-l
sin xdx = 3 Þ -ê
ë 2 - l
cos x ú
û0
=3

2
Þ =3 Þ l = 4/3
2-l

MULTIPLE CORRECT CHOICE TYPE


27.Ans.(A, D) A n +1 - A n
p/ 2
sin(2n + 1)x - sin(2n - 1)x
= ò
0
sin x

p/2

= ò 2 cos 2nx dx = 0
0
Þ An + 1 = An

B n + 1 – Bn
p/2 p/2
sin 2 (n + 1)x - sin 2 x sin(2n + 1)x
= ò
0
sin 2 x
dx = ò 0
sin x
dx

= An + 1
28.Ans.(A, C, D)
p/ 2
sin 2 nx
Let I n =
ò sin 2 x
dx
0

Assuming, I = In + 1 – 2In + In – 1
p/ 2
sin 2 (n + 1)x - 2sin 2 nx + sin 2 (n - 1)x
\ I= ò sin 2 x
dx
0

p/2
sin(2n + 1)x sin x - sin(2n - 1)x sin x
ÞI= ò sin 2 x
0

p/2
sin(2n + 1)x - sin(2n - 1)x
ÞI= ò sin x
dx
0

MATHEMATICS /AQ # 6 E-111


TM TARGET: JEE (MAIN + ADVANCED) 2020
ENTHUSIAST & LEADER
Path to success KOTA (RAJASTHAN)
COURSE
p/ 2 p/ 2
2sin x cos 2nx
ÞI= ò sin x
dx = 2 ò cos 2nxdx
0 0
\ In + 1 + In – 1 = 2In for n ³ 1
Þ I1 , I 2 , I3 are in A.P..
So, OPTION (C) is true
p/ 2
p
Now, I1 = ò 1.dx =
2
0

p/ 2 p/ 2
sin 2 2x p
and I2 = ò 2
=4 ò cos 2 xdx = 4 ´ =p
sin x 4
0 0

p p
\ I2 - I1 = p - =
2 2
p p
Then, I1, I2, I3,.......... are in AP with first term and common diference .
2 2
Thus, OPTION (A) is true
æ 16p ö
Now, sin ( I16 ) = sin ç ÷ = sin 8p = 0
è 2 ø
Hence, OPTION (D) is also true
29. Ans. (A, B) We have f(x) = ae2x + bex + cx
ln 4
39
f (0) = -1;f '(ln2) = 31; ò {f (x) - cx}dx =
2
0
Now, f(0) = –1
Þ a + b = -1 .......(1)
and, f '(x) = 2ae 2x + be x + c
Þ f '(ln2) = 31
=8a + 2b + c = 31
ln4
39
Now, ò {f (x) - cx}dx = 2
0

ln 4
39
Þ ò (ae2x + be x )dx =
2
0

ln4
æa ö 39
Þ ç e 2x + be x ÷ =
è2 ø0 2

a a 39
Þ (16) + b(4) - - b =
2 2 2

15 39
Þ a + 3b = Þ 15a + 6b = 39
2 2
Þ 5a + 2b = 13 ..........(3)
Solving (1), (2) and (3), we get
a = 5, b = –6, c = 3

MATHEMATICS /AQ # 6 E-112


TM TARGET: JEE (MAIN + ADVANCED) 2020
ENTHUSIAST & LEADER
Path to success KOTA (RAJASTHAN)
COURSE

xdx + ydy a 2 - x 2 - y2
30.Ans.(AD) Since, xdy - ydx =
x 2 + y2
putting x = r cos q, y = r sin q.
y
Þ x 2 + y 2 = r 2 and = tan q
x
Now, xdx + ydy = rdr
and xdy – ydx = x2 sec2 qdq = r2dq.
Hence, the given equation rduces to the form

rdr a2 - r2 dr
= Þ = a2 - r2
2
r dq r 2 dq

dr
Þ = dq
a2 - r2
Intergrating, we have
r y
sin -1 + c = q = tan -1
a x

æ 1 2 ö
Þ y = x tan ç c + sin -1 x + y2 ÷
è a ø

æ yö
or x 2 + y2 = a sin ç c + tan -1 ÷
è xø
INTEGER ANSWER TYPE
ex ea
31.Ans. [0002] f (x) = \ f (a) =
1+ e x
1 + ea

e- a 1
e- a = =
and f (-a) = 1 1 + ea
1 + e -a 1+ a
e
Þ f(a) + f(–a) = 1
Let f(–a) = a \ f(a) = 1 – a
1-a

Now, I1 = ò
a
xg(x(1 - x))dx

1-a

= ò (1 - x)g((1 - x)(1 - (1 - x))dx


a

1-a

= ò (1 - x)g(x(1 - x))dx
a

1-a
I2
\ 2I1 = ò g(x(1 - x))dx = I
a
2 Þ 2I1 = I 2 Þ
I1
=2

a
f (x) a
32.Ans. [0014] ò f (x) + f (a - x) = 2
0

é a a 2 a3 an ù 7
Þ lim ê + + + .... + ú =
n ®¥ 2 2 2 2û 5
ë

a 14 14
Þ lim = Þ 5a = 14 – 14a Þ a =
n ®¥ 1- a 5 19

MATHEMATICS /AQ # 6 E-113


TM TARGET: JEE (MAIN + ADVANCED) 2020
ENTHUSIAST & LEADER
Path to success KOTA (RAJASTHAN)
COURSE

æ 1ö
33.Ans. [2250] We have F(x) + F ç x + 2 ÷ = 3 ......(1)
è ø

1 æ 1ö
Replace x by x + in (1), we get F ç x + ÷ + F (x + 1) = 3 ......(2)
2 è 2ø
\ From (1) and (2), we get F(x) = F(x + 1) .....(3)
Þ F(x) is periodic function.
1500 1

Now consider I = ò
0
F(x)dx = 1500 ò F(x)dx = 1500
0

é 21 1
ù æ Using property ö
ê ú ç ÷
ê ò F(x)dx + ò F(x)dx ú ç of periodic ÷
êë 0 1
úû ç function ÷
2 è ø

1
Put x = y + in 2nd integral, we get
2

é 12 1
2
ù 1 1
ê æ 1ö ú
I = 1500 ê ò ò0 è 2 ø ú = 1500 ò ç F ( x ) + F ç x + ÷ ÷ dx = 1500 ò 3dx [Using (i)]
F(x)dx + F ç y + ÷ dy 2
æ æ 1 ö ö 2

êë 0 úû 0è è 2 øø 0

æ1ö
Hence I = 1500(3) ç ÷ = 750 × 3 = 2250 Ans.
è2ø
Note that for objective purpose take F(x) = 3/2]
p/ 4
34.Ans. [0192] = ò ( px - 4x 2 )ln(1 + tan x)dx
0

p/ 4
p
= ò ( px - 4x 2 )(ln(1 + tan( - x))dx
4
0

p
4
æ 2 ö
= ò ( px - 4x 2 )ln ç ÷ dx
0 è 1 + tan x ø
p/ 4
2I = ln2 ò (px - 4x 2 )
0

ln2 é px 2 4 3 ù ln2 é p3 p3 ù p3
I= ê - x ú= ê - ú= ln2
2 êë 2 3 úû 2 êë 32 48 úû 192

35.Ans. [0020 ] Area


2 2
= ò (x 2 - (2 - x 2 ))dx + ò (2 - (x
2
- 2))dx
1 2

2 2
= ò (2x 2 - 2)dx + ò (4 - x
2
)dx
1 2

2 2
æ 2x 3 ö æ x3 ö
=ç - 2x ÷ + ç 4x - ÷
ç 3 ÷ ç 3 ÷ø
è ø1 è 2
MATHEMATICS /AQ # 6 E-114
TM TARGET: JEE (MAIN + ADVANCED) 2020
ENTHUSIAST & LEADER
Path to success KOTA (RAJASTHAN)
COURSE

æ4 2 ö æ2 ö æ 8ö æ 2 2ö
= çç - 2 2 ÷÷ - ç - 2 ÷ + ç 8 - ÷ - çç 4 2 - ÷
è 3 ø è3 ø è 3ø è 3 ÷ø Sq. Units

2 2 4 16 10 2 20 - 12 2
=- + + - = sq.units
3 3 3 3 3
1

36. Ans.[0003] ò t 2 f (t)dt = 1 - sin x [Given]


sin x
Differentiating both sides w.r.t. x using Newton Leibnitz formula, we have
- sin2 xf(sinx)cos x = - cosx

Þ sin 2 xf (sin x) cos x = cos x

1
Þ f (sin x) = ·
sin 2 x
1
\ f (x) = 2 (x Î [–1, 1])
x
æ 1 ö
Now, f ç ÷=3
è 3ø
1/ n
é ù p 2p np
37. Ans.[0001] Let lim ê tan tan .....tan ú
n ®¥ ë 2n 2n 2n û

1
\ ln A = lim
n ®¥ n

é p 2p np ù
ê log tan 2n + log tan 2n + ..... + log tan 2n ú
ë û
n
1 pr 1 æp ö
= lim
n ®¥
å n ln tan 2n = ò0 ln tan çè 2 x ÷ø dx
r =1

2 p/ 2
p ò0
= ln tan y dy (1)

é 1 ù
ê Putting 2 px = y \dx = (2 / p)dy ú
ë û
p/2
Now let I = ò ln tan y dy
0

p/ 2 æ1 ö
I=ò ln tan ç p - y ÷ dy (using Property)
0 è2 ø
p/ 2 p/ 2
=ò ln cot y dy = -ò ln tan y dy = - I
0 0

MATHEMATICS /AQ # 6 E-115


TM TARGET: JEE (MAIN + ADVANCED) 2020
ENTHUSIAST & LEADER
Path to success KOTA (RAJASTHAN)
COURSE
or I + 1 = 0 or 2I = 0 or I = 0
\ from equation (1), ln A = 0 \ A = e0 = 1

38. Ans.[0009]

The area betwen the curves y = x and 2y + 3 = x


and x-axis in the 1st quadrant can be shown in the graph
\ Area of shaded portion OABO is given by
9
æ 3 ö 9
9 9 ç x2 ÷ 1 æ x2 ö
æ x -3ö
A=ò xdx - ò ç ÷ dx Þ A = ç 3 ÷ - ç
2 çè 2
- 3x ÷
÷
2 ø çç ÷÷
0 3è ø3
è 2 ø0

æ2 ö 1 ìæ 81 ö æ9 öü
Þ A = ç .27 ÷ - íç - 27 ÷ - ç - 9 ÷ ý
è3 ø 2 îè 2 ø è2 øþ

1
\ A = 18 - {18} = 9 sq. unit
2
MATCH THE COLUMN :
39. Ans. [A-r; B-p; C-s; D-q]
(A)

1
1 2
Required area = ò-1 x x dx = 2 ò x dx =
2

0
3

(B)

2
Required area = ò 0
(y1 - y 2 )dx

MATHEMATICS /AQ # 6 E-116


TM TARGET: JEE (MAIN + ADVANCED) 2020
ENTHUSIAST & LEADER
Path to success KOTA (RAJASTHAN)
COURSE
2
2 é x2 x3 ù
= ò0 [(x + 2) - (x )]dx = ê + 2x - ú
2

ë2 3 û0

8 10
= 2+ 4- = sq. units.
3 3
1
é x3 / 2 x 2 ù
ò( )
1
(C) Required area = x - x dx = ê - ú
0
ë 3 / 2 2 û0

æ 1 1ö 2 1 1
ç - ÷ = - = sq.units.
è 3 / 2 2ø 3 2 6

(D) y = 4 meets the parabola y2 = x at A is (16, 4)

Required area = Area of rectangle OMAC –Area OMA


16
16 x3 / 2
= 4 × 16 – ò0
x dx = 64 -
3/ 2 0

2 3 128 64
= 64 – (4) = 64 - = sq.units.
3 3 3

MATHEMATICS /AQ # 6 E-117


TM TM TARGET:JEE
TARGET: JEE(Main
(MAIN ++Advanced)
ADVANCED) 2020
2020
ENTHUSIAST&
ENTHUSIAST & LEADER
LEADER
Path to success KOTA (RAJASTHAN)
COURSE
Path to success KOTA (RAJASTHAN ) COURSE

ADVANCED QUIZ # 7(SOLUTION) STRAIGHT LINE & CIRCLE PART -1 MATHEMATICS


SINGLE CORRECT CHOICE TYPE

p2 -p 1 p2 - q 2 -(p + q) 0
1. Ans. (D) D = 1 q 2 1
q 1 = q2 - r 2 q+r 0
2 2
r 2
-r 1 r2 -r 1

p - q -1 0
1
= (p + q)(q + r) q - r 1 0
2
r2 -r 1

1
= (p + q)(q + r)[(p - q) + (q - r)]
2
1
= (p + q)(q + r)(p - r)
2
2. Ans. (A) Equation of line
perpendicular to AD is

7x – 4y = l .
It passes through (1,1)
Þ l=3 Þ (A) ]

3. Ans. (A) Clearly circumcentre is (0,0)


Now centroid G is

æ 5 - 13cos q + 13sin q 12 + 13sin q - 13cos q ö


ç , ÷
è 3 3 ø
G divides HO internally 2:1
5 - 13cos q + 13sin q 2(0) + 1(h)
\ =
3 3

12 + 13sin q - 13cos q 2(0) + 1(k)


and =
3 3
h -5
Þ sin q – cos q = ........(1)
13

k - 12
and sin q – cos q = .........(2)
13

h - 5 k - 12
\ From (1) and (2), we get =
13 13
Þ h–k+7=0
\ Locus of orthocentre H(h,k) is x – y + 7 = 0 Ans. ]
4. Ans. (B) C1 : (0, 0); r1 = 12 Þ C2 : (3, 4); r2 = 5
B

dmin = AB = 12 – 10 = 2 Ans. ]
MATHEMATICS /AQ # 7 E-118
TM TARGET: JEE (MAIN + ADVANCED) 2020
ENTHUSIAST & LEADER
Path to success KOTA (RAJASTHAN)
COURSE
5. Ans. (C) Ar. (square of sides 2) = 4 sq. unitds

6.Ans. (A) Line are (x – 1) (y – 2) = 0 Þ x = 1 and y = 2


Centre (1, 2) i.e. – g = 1 and – f = 2
Let the equation of the required circle is x2 + y2 + 2gx + 2fy + c = 0 ..... (1)
As (1) is orthogonal to x2 + y2 + 2x + 4y – 4 = 0
\ –2–8=–4+c Þc=–6
Hence x2 + y2 – 2x – 4y – 6 = 0 Ans.
7.Ans. (D) Equation of the line l is
y – 0 = m (x + 1) ...... (1)
solving it with x2 + y2 = 1 Þ x2 + m2 (x + 1)2 = 1

-2m 2 ± 4m 4 - 4 ( m 4 - 1) -2m 2 ± 2
( m2 + 1) x 2 + 2m 2 x + ( m 2 - 1) = 0, m Î Q x=
2 ( m 2 + 1)
=
2 ( m 2 + 1)

taking– ve sign x = – 1 (corresponding to A)

1 - m2
with + ve sign x=
1 + m2
since m Î Q hence x will be rational.
If x is rational then y is also rational from (1) ]
8.Ans. (A) Let C be the centre of the given circle. The circumcircle of the DRPQ passes through C.
\ (2,3) is the mid point of RC

\ coordinates of C are (–1, –2)


\ Equation of the circle x2 + y2 + 2x + 4y – 20 = 0
9.Ans. (B) Let required line be (y – 0) = m(x – 6), m > 0
Þ mx – y – 6m = 0 .......(1)
As distance of above line from N(1, 3) is 5, so

m - 3 - 6m
= 5 Þ (3 + 5m)2 = 25(1 + m2) Þ 9 + 30m = 25 Þ m = 16 = 8
1+ m 2
30 15

10.Ans.(C) We have, 2 x2 - 3xy - 2 y 2 = 0 `

ie, ( x – 2 y) (2x + y) = 0, which represents a


pair of perpendicular lines throught the origin
p ( 2 ) p (1)
2 2
3p
\ Required area = - = sq unit
4 4 4

MATHEMATICS /AQ # 7 E-119


TM TARGET: JEE (MAIN + ADVANCED) 2020
ENTHUSIAST & LEADER
Path to success KOTA (RAJASTHAN)
COURSE
11.Ans.(A) Clearly, point of intersection of lines x + y + 1 = 0 and x – y + 1 = 0, is (–1, 0) which is circumcentre of triangle ABC.
Let A(h, k) be any point on required locus.
So, (h + l)2 + k2 = 4

Hence, locus of (h, k) is (x + 1) 2 + y2 = 4

Þ x2 + y2 + 2x – 3 = 0

12. Ans. (A) Clearly, point of intersection of L1 and L2 is (0, k) which lies on y-axis
Clearly, from above figure, we get

æ1 3ö
k Îç , ÷
è3 2ø

13.Ans.(C) Image of (5, 0) in x – y + 3 = 0 is (–3, 8)


Hence the circle is (x + 3)2 + (y – 8)2 = 25

x2 + y2 + 6x – 16y + 48 = 0
So, g = 6, f = –16, c = 48

Hence, (g + f + c) = 38

14. Ans. (C)

h2 + k2 = 1 Þ x2 + y2 = 1

15. Ans. (A) We have (a + c)2 + 4b2 – 4ab – 4bc = 0


(a + c)2 + (2b)2 – 4b(a + c) = 0
Þ (2b – (a + c))2 = 0
Þ a, b, c are in A.P.
Þ ax + by + 2b – a = 0 (Put c = 2b – a)
Þ a(x –1) + b(y + 2) = 0 Þ (x – 1) + l(y + 2) =0

So, fixed point is (1, –2)

MATHEMATICS /AQ # 7 E-120


TM TARGET: JEE (MAIN + ADVANCED) 2020
ENTHUSIAST & LEADER
Path to success KOTA (RAJASTHAN)
COURSE
1
16. Ans. (A) PQ = 3 2 Þ xQ = 2 + 3 2 . =5
2

1
and yQ = 1 – 3 2. = – 2 Þ Q º (5, – 2)
2
Distance of Q from the line x + y = 1 is

| 5 - 2 - 1| 1
= 2 Þ QR = 2 2 Þ x = 5 – 2 2. =3
2 R
2

1
and yR – 2 – 2 2. =–4
2

Thus R º (3,–4).

17.Ans.(A) The centre c of the circle = (5,7) and the radius

= 52 + 7 2 + 51 = 5 5
PC = 12 2 + 52 = 13

Þ q = PA = 13 – 5 5 and p = PB = 13 + 5 3

\ G.M of p and q = pq = (13 - 5 5 )(13 + 5 5 )


= 169 - 125 = 2 11.
18.Ans. (A) Circle is (x – r)2 + (y – r)2 = r2
Þ x2 + y2 – 2xr – 2yr + r2 = 0
Hence the circles are
x 2 + y 2 - 2xr1 - 2yr1 + r12 = 0 ...... (1)

x 2 + y 2 - 2xr2 - 2yr2 + r2 2 = 0 ......( 2 )

As (1) and (2) are orthogonal so


2r1r2 + 2r1 r2 = r12 + r2 2
2 2 2
r ær ö ær ö ær ö
4 1 = ç 1 ÷ + 1 Þ ç 1 ÷ - 4ç 1 ÷ + 1 = 0
r2 è r2 ø è r2 ø è r2 ø

r1 4 ± 16 - 4
Þ = = 2± 2 3
r2 2

= 2 + 3 or 2 - 3 (rejected) (r1 > r2) Ans. ]


19. Ans. (C) Lines are y = 1, y = 0
y = – x, y = – x + 2
y = x + 1, y = x – 1
Area of OABCDEO = 2× ar (D OAB) + ar(OBCE)
1 1 3
= 2 ´ ´ ´1 + 1 =
2 2 2

MATHEMATICS /AQ # 7 E-121


TM TARGET: JEE (MAIN + ADVANCED) 2020
ENTHUSIAST & LEADER
Path to success KOTA (RAJASTHAN)
COURSE

20. Ans. (B) Since (0,0) and (1,1) lie on the same side, so
a2 + ab + 1 > 0
Q a Î R Þ D < 0 Þ b2 - 4 < 0
Þ – 2 < b < 2 Þ b = – 1, 0,1
Match the Column :
21. Ans.(A- s; B-r; C-q, D-p)
Equation of circle touching the coordinates axes and centre (r, r) in the first quadrant is
x 2 + y 2 - 2xr - 2yr + r 2 = 0
For r = a or b
hence C1 : x2 + y2 – 2ax – 2ay + a2 ..... (1)

Centre (a, a), radius = a, a > 0


C2 : x2 + y2 – 2bx – 2by + b2 ..... (2)
Centre (b, b), radius b, b > 0
(A) C1 and C2 touch each other
radical axis between (1) and (2) is
(1) – (2) = 0
2 ( b - a ) x + 2 ( b - a ) y - ( b2 - a 2 ) = 0
2x + 2y – (b + a) = 0 ..... (3)
if it touches both C1 and C2 then perpendicular from (a, a) = radius ‘a’
2a + 2a - ( b + a )
=a ..... (4)
8

3a - b = 2 2 a ..... (5)
now origin and (a, a) must lie on the same side of (3)
but (0, 0) gives – ve sign with (3)
hence (a, a) should also give the same sign i.e. 4a – b – a < 0 Þ 3a – b < 0
Hence (5) becomes
b
b - 3a = 2 2 a Þ = 3 + 2 2 Ans. Þ (S)
a
(B) If (1) and (2) are orthogonal then
2g1g 2 + 2f1f 2 = C1 + C2
i.e. 2 ( -a )( - b ) + 2 ( -a )( - b ) = a 2 + b 2
4ab = a2 + b2
2
æbö æbö
ç a ÷ - 4 ç a ÷ +1 = 0
è ø è ø
b
if = t, t 2 - 4t + 1 = 0
a
( t - 2)
2
Þ =3 Þ t - 2 = + 3 or - 3

MATHEMATICS /AQ # 7 E-122


TM TARGET: JEE (MAIN + ADVANCED) 2020
ENTHUSIAST & LEADER
Path to success KOTA (RAJASTHAN)
COURSE

t = 2+ 3
as t > 1 Þ 2 - 3 is not possible
b
\ = 2 + 3 Ans. Þ (R)
a
(C) If common chord is longest then (3) must pass through the centre (a, a) of C
i.e. 4a – b – a = 0
b
3a = b Þ = 3 Ans. Þ (Q)
a
(D) If C2 passes through the centre of C1 then (a, a) must satisfy (2)
i.e. a 2 + a 2 - 2b ( 2a ) + b 2 = 0 Þ 2a 2 - 4ab + b 2 = 0
2
æbö æbö
ç ÷ - 4ç ÷ + 2 = 0
èaø èaø

b
Put =t
a

( t - 2)
2
t 2 - 4t + 2 = 0 Þ = 4 - 2 = 2 Þ t - 2 = 2 or - 2

t = 2 + 2, t ¹ 2 - 2 (as t > 1) Þ (P) ]


22. Ans. [A–S ; B–R ; C–Q]
20 ù
d1 = = 4ú
(A) 5 square
úÞ
20
d2 = = 4ú
5 úû

2 ù
d1 =
(B) 5 úú
Þ interior not 90°
2 ú
d2 = ú

Þ rhombus

(C) 2x2 – 7xy + 3y2 + 5x + 10y – 25 = 0 º (x – 3y + 5) (2x – y –5)


the point of intersection is (4,3)

homogenising f(x,y) = 0 and x + 2y – 5 = 0

we get the homogeneous equation


2x2 – 7xy + 3y2 = 0
hence OAPB is a parallelogram ]

MATHEMATICS /AQ # 7 E-123


TM TARGET: JEE (MAIN + ADVANCED) 2020
ENTHUSIAST & LEADER
Path to success KOTA (RAJASTHAN)
COURSE
23. Ans. [A–pqr; B–qr; C–qrs; D–ps]

(A) x 2 + k 2 x 2 - 20kx + 90 = 0

x 2 (1 + k 2 ) - 20kx + 90 = 0

D£0

400k 2 - 4 ´ 90 (1 + k 2 ) £ 0

10k 2 - 9 - 9k 2 £ 0

k 2 - 9 £ 0 Þ k Î [ -3,3]

æp p ö
(B) 2 ç ´ (-5) + ´ p ÷ = -6 Þ -5p + p 2 + 6 = 0
è 2 2 ø

Þ p 2 - 5p + 6 = 0 Þ p = 2 or 3 Ans.

(C) r12 = l 2 - 4 ³ 0

l Î ( -¥, - 2 ] È [ 2, ¥ ) ..... (1)

r2 2 = 4l 2 - 8 ³ 0

l2 - 2 ³ 0

(
l Î -¥, - 2 ùû È éë 2, ¥ ) ..... (2)

(1) Ç ( 2) is lÎ ( -¥, - 2] È [ 2, ¥ ) Ans.

(D)

Ans. {1, 5} ]

MATHEMATICS /AQ # 7 E-124


TM TM TARGET:JEE
TARGET: JEE(Main
(MAIN ++Advanced)
ADVANCED) 2020
2020
ENTHUSIAST&
ENTHUSIAST & LEADER
LEADER
Path to success KOTA (RAJASTHAN)
COURSE
Path to success KOTA (RAJASTHAN ) COURSE

ADVANCED QUIZ # 08 (SOLUTION) STRAIGHT LINE & CIRCLE PART-2 MATHEMATICS


Paragraph for question Nos. 1 to 3
Ans. [1. A, 2. B, 3. C]
Triangle is right
R = orthocentre
M = circumcentre
2
æ 23 ö 1
RM = ç + 1÷ = 12 Ans.
è 2 ø 2

Incentre
20 ( -8 ) + 15 ( -15 ) + 25 (1)
x=
20 + 15 + 25 æ 23 ö
ç - , -7 ÷
è 2 ø
-160 - 225 + 25 -360
= = = -6
60 60

20 ( 5 ) + 15 ( -19 ) + 25 ( -7 )
y=
60
100 - 285 - 175 -360
= = = -6
60 60
hence incentre (– 6, – 6) (can be used to determine the equation of PD)
(not asked)

D 20.15
r= = = 5 Ans.
s 2.30

æ 23 ö 11
Coordinates of D using section formulae are ç -5, - ÷ and m PR = -
è 2 ø 2
\ equation PD is 11x + 2y + 78 = 0
Þ a + c = 89 Ans. ]
Paragraph for question Nos. 4 to 6
Sol. [4. A, 5. C, 6. B]
Solving x – y = 4 and 2x – y = 5
Þ x = 1 ; y= – 3

\ Coordinates of circumcentre is (1, – 3) Ans.

Equation of AB : y - 3 = -1( x + 2 ) Þ x + y = 1

æ5 3ö
\ M1 º ç 2 , - 2 ÷ Þ B ( 7, - 6 )
è ø
|||1y equation of AC is
1
y-3 = - ( x + 2 ) Þ 2y - 6 = - x - 2 Þ x + 2y = 4
2
æ 14 3ö
Solving 2x – y = 5 and x + 2y = 4 Þ M 2 º ç , ÷
è5 5ø
MATHEMATICS /AQ # 8 E-125
TM TARGET: JEE (MAIN + ADVANCED) 2020
ENTHUSIAST & LEADER
Path to success KOTA (RAJASTHAN)
COURSE
æ 38 9 ö
Þ C º ç ,- ÷
è 5 5ø
equation of BC
y + 6 = 7(x – 7) Þ 7x – y = 55 Ans.

-2 3 1 -9 9 0
Area = 1 7 -6 1=
1
-3/ 5 -21/ 5 0
2 2
38 / 5 -9 / 5 1 38 / 5 -9 / 5 1

1 éæ 21 3 ö ù 1 216 108
= ê ç ´ 9 + 9 ´ ÷ú = ´ =
2 ëè 5 5 øû 2 5 5
Paragraph for question Nos. 7 to 9
Ans.. 7(D) r1 = 2 ; r2 = 1 ; C1 = (0, 3) ; C2 = (6, 0) ; C1C2 = 3 5

clearly the circle with centre C1 and C2 are separated


8.(A) length of TCT = d 2 - (r1 + r2 ) 2 = 45 - 9 = 6
9.(D) C C1 = r + r1
C C2 = r + r2 Þ CC1 – CC2 = r1 – r2 = constant i.e. PS1 – PS2 = a constant hence hyperbula

MULTIPLE CORRECT CHOICE TYPE


10. Ans.(A) (2x – y – 1) (x + 3y + 2) = 0
hence the lines are
2x - y - 1 = 0 ù æ1 5ö
ú Þ Pç ,- ÷
x + 3y + 2 = 0 û è7 7ø
equation of the two lines joining origin and the point
of intersection of 3x – 5y = 2 and f (x, y) = 0 is

(3x - 5y)(3x - 5y)


2x 2 + 5xy - 3y 2 +
2

2(3x - 5y) 2
- =0
4

2x 2 + 5xy - 3y 2 + (3x - 5y)2 - (3x - 5y) 2 = 0

2x 2 + 5xy - 3y 2 = 0

11. Ans. (A, B) L must be angle bisector of L1 & L2


\ L is given by

3x + 4y - 1
5

5x - 12y + 2

13

DP1DA =% DP2 DA

Þ ÐP1AD = ÐP2 AD .

MATHEMATICS /AQ # 8 E-126


TM TARGET: JEE (MAIN + ADVANCED) 2020
ENTHUSIAST & LEADER
Path to success KOTA (RAJASTHAN)
COURSE
12. Ans. (B,C) x2 + 400 = y2 .....(i)
& 900 + (x – 5)2 = (y + 5)2

Þ 900 + x2 – 10x = y2 + 10y


Þ 900 + x2 – y2 = 10 (x + y)
Þ 500 = 10 (x + y)
Þ x + y = 50 ......(ii)
From (i) & (ii)
y– x= 8 .....(iii)
From (ii) & (iii)
So, y = 29 & x = 21
\ D1 = 210 & D 2 = 240
D2 8
= Þ 8D1 = 7D 2 ,
D1 7

D1 210 D 240
Also, P1 = = = 6 and P2 = 2 = =6
s 35 s 40

k -1 k -1
13.Ans. (A,B) h + 1 - h - 1 = 2

é (h - 1) - (h + 1) ù
(k - 1) ê úû = ±2
ë h 2 -1

2(k - 1)
= ±2 Þ k - 1 = ± (h 2 - 1)
h2 -1
(+) ve, k – 1 = h2 – 1 Þ y = x2 Ans Þ (A)
(–) ve, k – 1 = 1 – h2 Þ y = 2 – x2 Ans Þ (B)
3x + 6y
14. Ans. (A,D) We have = 1 .....(1)
k
2x2 + 2xy + 3y2 – 1 = 0 ....(2)
Now homogenising (2) with the help of (1), we get
2
æ 3x + 6y ö
Þ 2x + 2xy + 3y – ç
2 2
÷ =0
è k ø
Þ k2(2x2 + 3x + 3y2) – (3x + 6y)2 = 0
Now coefficient of x2 + coefficient of y2 = 0
Þ (2k2 – 9) + (3k2 – 36) = 0
Þ 5k2 = 45 Þ k2 = 9 Þ k = 3 or – 3 Ans. ]
15. Ans. (B,C) x + y = k; AB =1
-k
p= ; 2 12 - p2 = 1 Þ 4(1 - p 2 ) = 1 Þ 4p 2 = 3
2

k2 3 3 6
4× =3 Þ k2 = Þ k=± =± Ans. ]
2 2 2 2
MATHEMATICS /AQ # 8 E-127
TM TARGET: JEE (MAIN + ADVANCED) 2020
ENTHUSIAST & LEADER
Path to success KOTA (RAJASTHAN)
COURSE
16. Ans. (A, B, C) Let equation of direct common tangent (s) be
y – 0) = m(x –3)
Þ mx – y – 3m = 0
As, p = r
| m(1) - 0 - 3m |
Þ =1
1 + m2
1 1
Þ 4m2 = 1 + m2 Þ m2 = Þm=±
3 3
Clearly, DPQR is an equilateral triangle.
Now, verify alternatives
17. Ans. (A, B, C, D) Simple
INTEGER ANSWER TYPE
18. Ans. [0465] Arranging the lines in descending order
m1 = 5 ; m2 = 3 and m3 = – 1

m1 - m 2 2 1
\ tan A = 1 + m m = 1 + 15 = 8
1 2

m2 - m3 3 + 1
tan B = 1 + m m = 1 - 3 = -2
2 3

m 3 - m1 -1 - 5 3
tan C = 1 + m m = 1 - 5 = 2
3 1

1 9 1 + 256 + 144 401


å tan 2 A = + 4+ = =
64 4 64 64

Þ p + q = 465 Ans.]

19. Ans. [0008] dC C = y + 1 (y > 0)


1 2

x 2 + (y - 3) 2 = y + 1

x 2 + y 2 - 6y + 9 = y 2 + 2y + 1
x2 – 8y = – 8
x2 + 8 = 8y

x2
y= 1 + it is parabola Ans.]
8

20. Ans. [0010] Given h > 0, k > 0


2r = distance between the two parallel lines x – y + 10 = 0 and x – y – 6 = 0
16
2r = =8 2
2
Þ r= 4 2
but h=r= 4 2

h - k + 10
now r= = 4 2
2

MATHEMATICS /AQ # 8 E-128


TM TARGET: JEE (MAIN + ADVANCED) 2020
ENTHUSIAST & LEADER
Path to success KOTA (RAJASTHAN)
COURSE
\ (0, 0) and (h, k) lies on the same side h – k + 10 > 0
\ h – k + 10 = 8
Þ k=2 + 4 2 Þ h+ k=2 + 8 2
\ a + b = 10 Ans.

x y 1
21. Ans. [0025] The family of given circles is (x – 3) (x – 6) + (y – 7) (y – 5) + l 3 7 1 = 0
6 5 1

simplifying
S1 : x2 + y2 + (2l – 9)x + (3l – 12) y + 53 – 27l = 0
S2 : x2 + y2 – 4x – 6y – 3 = 0
common chord is
(2l – 5) x + (3l – 6)y + 56 – 27l = 0
– 5x – 6y + 56 + l(2x + 3y – 27) = 0
5x + 6y = 56
2x + 3y = 27

23 æ 23 ö
solving, x = 2; y = Þ ç 2, ÷
3 è 3 ø

MATHEMATICS /AQ # 8 E-129


TM TM TARGET:JEE
TARGET: JEE(Main
(MAIN ++Advanced)
ADVANCED) 2020
2020
ENTHUSIAST&
ENTHUSIAST & LEADER
LEADER
Path to success KOTA (RAJASTHAN)
COURSE
Path to success KOTA (RAJASTHAN ) COURSE

ADVANCED QUIZ # 09(SOLUTION) CONIC SECTION MATHEMATICS


SINGLE CORRECT CHOICE TYPE

xx1 yy1 x × ae y × b 2 ex y
1.Ans. (B) T : - 2 ; 2 - = 1 or - =1
a2 b a a.b 2 a a

or ex – y = a Þ m = e Ans.
2.Ans. (C) Equation of normal to the ellipse at ‘P’ is 5x sec q – 3y cosec q = 16 .....(i)

Equation of normal to the circle x2 + y2 = 25 at point Q is y = x tanq ......(ii)


Eliminating q from(1) & (2).
We get = x2 + y2 = 64.
3.Ans. (B) Let the centre be (h, k)

|h-k| |h+k|
So, = Þ hk = 0
2 2

\ Lcuos (h, k) of centre of circle is xy = 0


4.Ans. (B) Let P = (at12, 2at1) and Q = (at22,2at2)
at12 t
Given 2
= k 2 Þ 1 = k or t = kt
at 2 t2 1 2

The point of intersection of the tangents at P and Q to the parabola y2 = 4ax is (at1t2, a(t1+t2))
Let a = at1t2 and b = a(t1 + t2)

b2 a 2 (t12 + t 22 + 2t1 t 2 )
=
a at1 t 2

æ t1 t 2 ö b2 æ 1 ö
= a ç t + t + 2÷ Þ a = a ç k + k + 2÷
è 2 1 ø è ø

2
æ 1 ö
\ locus of (a,b) is y = ax ç k +
2
÷ which is a parabola
è kø

5.Ans. (C) Given ( 2t12 , 4t1 ) and ( 2t 22 , 4t 2 ) lie on x + y = 2


\ 2t12 + 4t1 = 2 ......(1) Þ 2t 22 + 4t 2 = 2
(1) – (2) Þ t1 + t2 + 2 = 0
But t1 + t2 + t3 = 0 where t3 is the parameter of the 3rd conormal point
Þ t 3 = +2

Third conormal point is (8,8)

MATHEMATICS /AQ # 9 E-130


TM TARGET: JEE (MAIN + ADVANCED) 2020
ENTHUSIAST & LEADER
Path to success KOTA (RAJASTHAN)
COURSE
6. Ans. (B) Let middle point of chord is P(h, K) then its equation is T = S 1
xh yK h 2 K 2
Þ + = +
a 2 b2 a 2 b2
Þ b2 hx + a 2 Ky = b 2 h 2 + a 2 K 2 ... (i)
Now tangents at extremities intersect on director circle

\ Let Q ( )
a 2 + b 2 cos q, a 2 + b 2 sin q is point of intersection of tangents

\ Equation of chord of contact is

a 2 + b 2 cos q x a 2 + b 2 sin q y
+ = 1 ... (ii)
a2 b2
Equation (i) & (ii) are same so we have

b 2 a 2 + b 2 cos q a 2 a 2 + b 2 sin q a 2b2


= = 2 2
2
b h 2
a K b h + a2K2

a 2b2h
\ cos q =
(a 2
+ b2 ) ( b2h 2 + a 2K 2 )

a 2b2 K
sin q =
a 2 + b2 ( b 2 h 2 + a 2 K 2 )

\ Locus is ( a 2 + b 2 )( b 2 x 2 + a 2 y 2 ) = a 4 b 4 ( x 2 + y 2 )
2

7.Ans. (A) (x - 2)(x - 1)(x + 2) = (x 2 - 2)(x - 1) = x3 – x2 – 2x + 2 a + b + g = -1 - 2 + 2 = -1


8.Ans. (A) Equation of tangent is y = 2x ± 4a 2 + b2
Þ this is normal to the circle x2 + y2 + 4x + 1 = 0
Þ this tangent passes through (–2, 0).
Þ 0 = – 4 ± 4a 2 + b 2 Þ 4a2 + b2 = 16
Þ using A.M ³ G.M. we get

4a 2 + b 2
³ 4a 2 + b 2 Þ ab £ 4.
2

x 2 y2
9.Ans. (C) Equation of hyperbola is - =1
9 16
Equation of tangent is y

= mx + 9m 2 - 16 Þ 9m 2 - 16 = 2 5 Þ m = ±2
Þ a + b = sum of roots = 0 .
10. Ans. (A) Let the point be (a,b) Þ b = a + c
Chord of contact of hyperbola T = 0
xa yb
\ - =1
2 1
xa æx ö
Þ - y(a + c) = 1 Þ ç - y ÷ a - (yc + 1) = 0
2 è 2 ø
Since this passes through point (x1, y1)
x1
\ x1 = 2y1 \ y = 2 .
1

MATHEMATICS /AQ # 9 E-131


TM TARGET: JEE (MAIN + ADVANCED) 2020
ENTHUSIAST & LEADER
Path to success KOTA (RAJASTHAN)
COURSE
2
11.Ans.(A) Tangent y2 = 8(x + 2) is y = m(x + 2) +
m

2 c æ 1ö
c = 2m + Þ = çm+ ÷
m 2 è mø

1 c
Qm + ³2Þ ³2 Þ c³4Þ
m 2
The minimum value of c = 4.
12.Ans.(C) The equation of the normal at (a sec q, b tan q) to the given hyperbola is ax cos q + by cot q = (a2 + b2) This meets the
transverse axis i.e. x- axis at at G ..

æ a 2 + b2 ö
So, the coordinates of G are ç a sec q, 0 ÷
è ø
The coordinates of the vertices A and A’ are A (a,0) and A’ (–a,0) respectively.
\ AG. A’G

æ a 2 + b2 öæ a 2 + b2 ö
= ç - a + sec q ÷ ç a + sec q ÷
è a øè a ø
= (–a + ae2 secq) (a + ae2 sec q)
= a2(e4 sec2q – 1).
13.Ans.(D) For the two ellipses to intersect in 4 distinct points, a > 1
Þ b2 – 5b + 7 > 1
Þ b2 – 5b + 6 > 0 Þ b Î ( -¥, 2) È (3, ¥ )
\ b does not lie in [2,3].

(y - 4x1 )
3/ 2
(4 + 4)
2 3/ 2

14.Ans. (D) A = 1
= =8 2
2a 2
Paragraph for question nos. 15 to 17
x +1 y - 2
15 (C) Let line = = r any point on this line (–1+ r cos q,2 + r sinq)
cos q sin q
This point lie on hyperbola xy = c2
Þ (–1 + r cosq) (2 + r sin q) = c2
Þ sinq cosq r2 +(2 cosq – sinq) r – 2 – c2 = 0
sin q - 2cos q -2 - c2
\ PA + PB = ; PA. PB =
sin q cos q sin q cos q
PA + PB sin q - 2 cos q
PQ = =
2 2sin q cos q
co-ordinate of
æ sin q - 2cos q sin q - 2cos q ö
Q = ç -1 + ,2+ ÷
è 2sin q 2cos q ø
sin q - 2cos q sin q - 2cos q
x=–1+ , y= 2 +
2sin q 2cos q
1 1
Þx+ = - cot q and y - 1 = tan q
2 2
æ 1ö 1
Locus ç x + ÷ (y - 1) = - Þ 2x = y(1 + 2x)
è 2ø 2

MATHEMATICS /AQ # 9 E-132


TM TARGET: JEE (MAIN + ADVANCED) 2020
ENTHUSIAST & LEADER
Path to success KOTA (RAJASTHAN)
COURSE
16 (B) PA,PQ,PB are in GP
2 + c2
Þ PQ2 = PA. PB = – Þ 0 = 2 + c2 + (PQ sin q) . (PQ cosq)
sin q cos q
Þ 0 = 2 + c2 + (y – 2) . (x – 1) Þ xy + y – 2x + c2 = 0
2 PA + PB sin q - 2cos q
17 (A) = =
PQ PA.PB -(2 + c2 )
Þ 4 + 2c2 + PQ sinq – 2 PQ cosq = 0 Þ 4 + 2c2 + (y – 2) – 2 (x + 1) = 0
Þ 2x – y = 2c2
MULTIPLE CORRECT CHOICE TYPE

æ b(1 + cos q) ù
18. Ans.(A,C,D) Coordinates of V º ç -a, sin q ûú
è

æ b(1 - cos q) ù
& V ' º ç a,
è sin q ûú

b(1 + cos q)
l(AV) = &
sin q
b(1 - cos q)
l(A 'V ') = Þ l(AV).l(A 'V ') = b 2
sin q
b(1 + cos q)
Slope of V¢S = a sin q(e - 1) &

b(1 + cos q)
Slope of VS =
a sin q(e + 1)
Prodcut slope of V¢S and

b 2 sin 2 q
VS = = -1
a sin 2 q(e 2 - 1)
2

Þ ÐV 'SV = 90° Also ÐVS'V ' = 90°


Hence V 'S'VS is a cyclic quadrilateral
19.Ans.(A,D) (A) We have x2 – 2x + 9 = 4y Þ 4y = (x – 1)2 + 8 Þ (x – 1)2 = 4(y – 2)
Clearly focus of above parabola is (1, 3)
x 2 y2
(B) We have + =1
25 9
Using property of ellipse, we have SP + S’P = 10, where S and S’ are foci of ellipse.
(C) As given point (1, 2) lies on 2x + y – 4 = 0, so locus of P is a line pair.
(Two lines will be real and distinct)
x 2 y2
(D) We have 2x2 + 3y2 = 6 Þ + =1
3 2

2 1 1
Clearly, e = 1 -
2
= Þ e=
3 3 3

20. Ans.(A, C) Let P be the point ( 2t12 , 4t1 ) and Q be ( 2t 22 ,4t 2 )


Q given focal distance of P = 10
i.e. 2 + 2t 1
2
= 10 Þ t1 = ±2

-1
Now for t1 =2, t2 = [Q t1 t 2 = -1 ]
2
MATHEMATICS /AQ # 9 E-133
TM TARGET: JEE (MAIN + ADVANCED) 2020
ENTHUSIAST & LEADER
Path to success KOTA (RAJASTHAN)
COURSE
æ1 ö 1 æ1 ö
\ coordinates of Q are ç , -2 ÷ and for t1 = -2 , t 2 = . \ coordinates of Q are ç , 2 ÷
è2 ø 2 è2 ø

æ ö 1
21. Ans.(A,C) y2 – 2y = 4x – 3 Þ (y - 1) = 4 ç x - 2 ÷
2

è ø

æ1 ö æ3 ö
so vertex º ç ,1÷ ; focus º ç ,1÷ , Axis is y = 1.
è2 ø è2 ø

5
22.Ans.(A,B) m = slope of tangent = –
12

x 2 y2
If a line m is tangent to the hyperbola - = 1 then point of contact is
a 2 b2

æ a 2m b2 ö
ç± 2 2 ,± ÷
è a m -b 2
a m -b ø
2 2 2

æ 4ö æ 4ö
\ Here point of contact is ç m5, ± ÷ out of these two only ç 5, - ÷ lies on 5x + 12y = 9
è 3ø è 3ø
and obviously this will also lie on 4x + 15y = 0 and 7x + 12y = 19
23.Ans.(C,D) Let point of intersection be (h, k) equation of PQ is
hx + 2ky = 4, touches x2 + y2 = 1
16 h2
2
= 1 + 2 [c2 = a2 (1 + m2)]
4k 4k
x + 4y = 16
2 2

x 2 y2
+ =1
16 4

4 3
e = 1- =
16 2

2´4
Length of latus rectum = = 2 units.
4
INTEGER ANSWER TYPE
24. Ans. [0186]
Equation of parabola, (x – 3)2 = k(y + 11)
which is passing through (7, – 4)
Þ k = 16/7
\ 16y = 7(x – 3)2 – 176
Þ a + h + k = 186

MATHEMATICS /AQ # 9 E-134


TM TARGET: JEE (MAIN + ADVANCED) 2020
ENTHUSIAST & LEADER
Path to success KOTA (RAJASTHAN)
COURSE
MATCH THE COLUMN :
25. (A – r, B – S. C – p, D–q)
Locus of point of intersection of perpendicular
tangent is directrix which is 12x - 5y + 3 = 0.
Locus of foot of perpedicular from foucs upon any tangent is
tangent at the vertex, which is parallel to directrix and
equidistant from directrix and latus rectum line, i.e.,
12x – 5y + l = 0
l -3 l+2 1
where = Þl=
12 + 5 2
12 + 5
2 2 2 2
Hence, equation of tangent at vertex is 24x – 10y + 1 = 0.
Minimum length of focal chord occurs along the latus
rectum line, which is a line passing through the focus
and parallel to directrix, i.e., 12x – 5y – 2 = 0.
Parabola is symmetrical about its axis, which is a line
passing through the focus (1,2) and perependicular to the
directrix, which has equation 5x + 12y – 29 = 0.
26. (A – p, q, r, s; B – p, q, r, s; C – q, r, s; D – p)
æ cos q sin q ö
(A) Equation of tangent at ç , ÷ is
è 2 3 ø
2x cosq + 3y sinq = 1
which is parallel to the given line 8x = 9y
4 3
\ cos q = ± ,sin q = m
5 5

æ 2 1ö æ 2 1ö
Hence, point are ç , - ÷ and ç - , + ÷ .
è5 5ø è 5 5ø
Distance between the points is
16 4 2
+ = Which is less than 1.
25 25 5

(x + 1) 2 (y + 2) 2
(B) The given equation is + = 1 (a = 5, b = 3)
9 25

9 16 4
Þ e = 1- = Þ e = Þ distance between the foci is 2ae = 8
2

25 25 5
(C) Equation of normal at (3 cosq, 2 sinq) is
3x secq – 2y cosec q = 5
which is parallel to the given line 2x + y = 1. Therefore,
3 4
cos q = m ,sin q = ±
5 5

æ -9 8 ö æ9 8ö 16
Hence, points are ç , ÷ and ç , - ÷ . The required sum of distances = .
è 8 5 ø è 5 5 ø 5
(D) Consider any point (t,t + 2), t Î R, on the line x – y + 2 = 0.
The chord of contact of ellipse w.r.t. this point is xt + 2y(t + 2) = 2
Þ (4y – 2) + t(x + 2y) = 0
This line passes through point of intersection of lines
4y – 2 = 0 and x + 2y = 0
\ x=–1
Hence, the point is (–1,1/2), whose distance from (2,1/2) is 3.

MATHEMATICS /AQ # 9 E-135


TM TM TARGET:JEE
TARGET: JEE(Main
(MAIN ++Advanced)
ADVANCED) 2020
2020
ENTHUSIAST&
ENTHUSIAST & LEADER
LEADER
Path to success KOTA (RAJASTHAN)
COURSE
Path to success KOTA (RAJASTHAN ) COURSE

ADVANCED QUIZ # 10 (SOLUTION) LOG AND QUADRATIC EQUATION MATHEMATICS


SINGLE CORRECT CHOICE TYPE

1. Ans. (D) x2 + ax + b = 0 á c Þ x 2 + cx + d = 0á ab
d

c + d = – a ... (1) Þ cd = b ... (2)


a + b = – c ... (3) Þ ab = d ... (4)

(2) × (4) abcd = bd Þ ac = 1 ... (5)

(1) + (3) Þ a + b + c + d = – a – c Þ 2 ( a + c ) + b + d = 0 .... ( 6 )

now a is a root of x 2 + cx + d = 0 Þ a 2 + ac + d = 0

and c is a root of x 2 + ax + b = 0 Þ c 2 + ac + b = 0

add a2 + c2 + 2ac + b + d = 0 Þ a 2 + c 2 + 2ac - 2 ( a + c ) = 0

Þ ( a + c ) - 2 ( a + c ) = 0 Þ ( a + c ) [ a + c - 2] = 0
2

\ a + c = 0 or a + c = 2

Also ac = 1

if a + c = 0 and ac = 1 Þ ( a - c ) = ( a + c ) - 4ac Þ ( a - c )2 = -4 (not possible)


2 2

If a + c = 2 and ac = 1 Þ ( a - c ) = ( a + c ) - 4ac Þ ( a - c )2 = 0
2 2
Þ a = c \ a2 = 1 Þ a = ±1

a = 1; c = 1 or a = -1; c = -1 rejected as a + c = 2 now form (4) Þ ab = d Þ b = d

b + d = – 2(a + c) Þ b + d = – 4 Þ b = d = -2 Þ a + 2b + 3c + 4d = 1 - 4 + 3 - 8 = -8 Ans. ]

2. Ans. (D) Clearly, min. (2x2 – ax + 2) and max. (b – 1 + 2x – x2) will occur at the vertices of the parabolas y = 2x2 – ax + 2 and y =
a2
b – 1 + 2x – x2 respectively, so that min. (2x2 – ax + 2) = 2 - and max. (b – 1 + 2x – x2) = b
8

a2
Hence, 2 - > b Þ 16 – a2 > 8b Þ a2 + 8b – 16 < 0
8
Þ a2 – 4(4 – 2b) < 0
Now, discriminant of 2x2 + ax + (2 – b) = 0,
is a2 – 4(4 – 2b), which is less than zero.
Hence, all roots of the equation 2x2 + ax + 2 – b = 0 are complex roots
3. Ans. (B) We
x2 – 2(4a – 1)x + (15a2 – 2a – 7) > 0 " x Î R
So, disc. < 0 (As coefficient of x2 > 0)
Þ 4(4a - 1) 2 - 4.1(15a 2 - 2a - 7) < 0

Þ (a - 2)(a - 4) < 0 Þ 2<a<4

MATHEMATICS /AQ # 10 E-136


TM TARGET: JEE (MAIN + ADVANCED) 2020
ENTHUSIAST & LEADER
Path to success KOTA (RAJASTHAN)
COURSE
4. Ans. (B) Put x2 = t ³ 0
at2 + bt + c = 0; b2 ³ 4ac

-b c
> 0 and > 0
a a

b c
< 0 and > 0
a a
Þ a > 0, c > 0, b < 0]
5. Ans. (C) We have (p –3) x2 + 12x + (6 + p) > 0 "x Î R
So, p > 3 and Disc. < 0 Þ 144 – 4(p –3) 6 + p) < 0
Þ (p + 9) (p – 6) > 0
Þ p < –9 or p > 6
\ we get, p > 6
Hence, pleast integral = 7

1
6. Ans. (C) a n = log 2002 = log 2002 n
n

\ b = log 2002 ( 2.3.4.5 )

and c = log 2002 (10.11.12.13.14 )

æ 2.3.4.5 ö
\ b - c = log 2002 ç ÷
è 10.11.12.13.14 ø

æ 1 ö
= log 2002 ç ÷ = -1
è 2002 ø
7. Ans. (A) 1 – cos2x + a cos x + a2 – 1 – cos x ³ 0
or cos2x + (1 – a) cos x – a2 £ 0
put cos x = t
t2 + (1 – a) t – a2 £ 0 " t Î [-1,1]
f (t) = t2 + (1 – a) t – a2
f (1) £ 0 and f (–1) £ 0
f (1) £ 0
Þ (a + 2) (a – 1) ³ 0 and f(–1) £ 0 Þ a(a – 1) ³ 0
hence a Î ( -¥, -2] È [1, ¥)

since a < 0 Þ final answer is a Î ( -¥, -2] ]

8. Ans. (A) The equation 4px2 + x(12p + 2q) + 9p + 3q + r = 0 can


be written as
p(2x + 3)2 + q(2x+3) + r = 0 .....(1)
The other equation is px + qx + r = 0
2
.....(2)
By comparing (1) and (2) we can say that if x is a root
of (2) then y = 2x + 3 is a root of (1)
Given that – 3 < a < 1. where l and d is a root of (2)
Þ – 3 < 2 g + 3 < 1 Þ -3 < g < 1 .....(3)
and 2d + 3 > 7 Þ d > 2 (current option (A))
MATHEMATICS /AQ # 10 E-137
TM TARGET: JEE (MAIN + ADVANCED) 2020
ENTHUSIAST & LEADER
Path to success KOTA (RAJASTHAN)
COURSE

ab(a + b)
9. Ans. (C) The equation a(x - b) 2 + b(x - a ) 2 = 0 has the product of its roots given by (a + b) = ab < 0 (Given) [(a + b)x2

– 2x(ab + ab) + ab(a + b) = 0]


i.e. (a + b) x2 – 4abx + ab(a + b) = 0
Also the discriminant
16a2b2 – 4ab(a + b)2 = – 4ab [(a + b)2 – 4ab]
= – 4ab (a - b)2 > 0

So, the roots are of opposite sign and are real. Ans.

10.Ans.(A) Let log 4 A = log 6 B = log 9 ( A + B ) = x


x
B æ 3ö
Þ A = 4x ; B = 6 x and A + B = 9 x ; we have to find =
A çè 2 ÷ø
Þ 4x + 6 x = 9 x Þ 22 x + 2x . 3x = 32 x
let 2x = a; 3x = b
Þ b2 - ab - a 2 = 0

b2 b b 1± 5
- -1 = 0 Þ =
a2 a a 2
x
b 5 +1 æ3ö B
\ = Þ ç ÷ = = 2 cos 36° Ans. ]
a 2 è2ø A

11. Ans. (A) 60a = 3 Þ a = log 60 3


60b = 5 Þ b = log 60 5

1- a - b

let x = 12 2 (1- b )

1 - a - b 1 - ( log 60 15 )
log12 x = =
2 (1 - b ) 2 (1 - log 60 5 )

\ log12 x = log144 4 = log12 2 Þ x = 2 Ans.


12.Ans.(A) We are given ,
a
- = -C = 1 + a + b + c
3
Þ a = 3c, a + b + 1= –2c
Þ 3c + b + 1 = –2c Þ b = –5c – 1
Also given P(O) = 2 Þ c = 2 Þ b = –5 × 2 – 1 = –11
13.Ans.(A) Given equation
1 1
log 2 (3 - x) - log 2 = + log 2 (x + 7)
2(5 - x) 2

(3 - x) 2(5 - x) = 2(x + 7)
Þ x = 1, 8 but x = 8 is rejected. not defined in the given system

MATHEMATICS /AQ # 10 E-138


TM TARGET: JEE (MAIN + ADVANCED) 2020
ENTHUSIAST & LEADER
Path to success KOTA (RAJASTHAN)
COURSE
14.Ans.(A) 2, 6, 2(k – 1) are terms of GP Þ 36 = 4(k – 1) Þ k = 10
Now x2 – x – 6 > 0 Þ (x – 3) (x + 2) > 0 Þ x Î (–¥, –2) È (3, ¥) .........(i)
and |x| < 100 Þ x Î (–100, 100) .........(ii)
Them common x for (i) and (ii) is (–100, –2) È (3, 100) and common integer are {–99, –98, ........... –3, 4, 5,.........99}

\ total integer is 193

15.Ans.(D) Let f(x) = 4x2 – 16 x + l


then f(1) > 0, f(2) < 0
and f(3) > 0
Þ l > 12, l < 16 and l > 12
\ 12 < l < 16

\ Integral values of l = 13, 14, 15

16. Ans. (C) product of root

2c2 l + a 2 m
2
absolute term æcö m
= = = 2ç ÷ +
coefficient of x 2
a l
2
èaø l

= 2(ab)2 – a4 – b4

= 2a 2 b2 - a 4 - b 4 = -(a 2 - b2 ) 2 < 0

Þ roots are opposite in sign


Paragraph for question Nos. 17 to 19
Sol. [17-C ; 18-D ; 19-A]

b
- = -1 Þ b = 2 and f(0) = 2 Þ c = 2
2
f(x) = x2 + 2x + 2 = (x + 1)2 + 1
f(x)min = 1,
Now f(–2) + f(0) + f(1) = (4 – 4 + 2) + (2) + 5 = 9
f(x) = a Þ x 2 + 2x + 2 - a = 0 has two distinct real roots
Þ 4 – 4(2–a) > 0 Þ 1 – 2 + a > 0 Þ a > 1
MULTIPLE CORRECT CHOICE TYPE
20. Ans. (BCD) x = 1 is a root of the equation,
\ other root is also 1

ca - ab
\ =1
ab - bc

2ab
2ab = bc + ac i.e c =
a+b

\ a, c, b are in H.P.

b 2a 2c - ab2 c 2 ab - bc
Since = =1
a 2 c2 b - b2 a 2 c ac - ab

\ ab 2 c 2 , b 2 a 2 c, a 2 c 2 b are in A.P..
MATHEMATICS /AQ # 10 E-139
TM TARGET: JEE (MAIN + ADVANCED) 2020
ENTHUSIAST & LEADER
Path to success KOTA (RAJASTHAN)
COURSE
2
1 æ 1ö
21.Ans. (BD) x + = 14 Þ ç x + ÷ - 2 = 14
2
2
x è xø

2
æ 1ö 1
Þ ç x + ÷ = 16 Þ x + = 4 .....(1)
è xø x

1
Cubing both sides x 3 + + (3 ´ 4) = 64
x3

1
x3 + = 52
x3

æ 5 1 ö æ 3 1 öæ 2 1 ö æ 1ö
ç x + 5 ÷ = ç x + 3 ÷ç x + 2 ÷ -ç x + ÷
è x ø è x øè x ø è xø
= 52 × (14) – 4 = 724 Ans.]
INTEGER ANSWER TYPE
22.Ans.[5050] We have |2x –|2x – 1|| = 1
1 1
Case I : If x ³ Þ |2x – (2x – 1)| = 1, which is true Þ given equation is satisfied " x ³ .
2 2
1
Case II : If x < Þ |2x + (2x – 1)| = 1 Þ |4x – 1| = 1
2
1
Þ 4x – 1 = ± 1 Þ x = 0,
2

æ 1ö
\ x = 0 ç Reject x = ÷
è 2ø

So, solution set is x Î {0} È é 1 , ¥ ö÷


ê2
ë ø
Hence, sum = 1 + 2 + ...... + 100
100 ´ 101
= = 50 ´ 101 = 5050
2

23.Ans.[0004] As 9 x - 2 = 3 x -2
7
æ x -2 |x - 2| -9 5 ö
1

E = ç 3 + (3 )÷
è ø
Tr + 1 = nCr ·xn – r · yr (Put r = 5)
Now, T6 = 567 = 7C5(3|x – 2|)2 (3| x –2| – 9) Þ 27 = 32|x – 2|. (3|x – 2| – 9)
Þ |x – 2| = 4 Þ x = 6 or – 2
x1
24.Ans.[0009] Let f(x) = 5x2 – 2kx + 1
x2
Now we want exactly one x for which f(x) < 0
Hence 1 < |x1 – x2| < 2

|x1 – x2| < 2 Þ (x1 – x2)2 < 4 Þ (x1 + x2)2 – 4x1x2 < 4

MATHEMATICS /AQ # 10 E-140


TM TARGET: JEE (MAIN + ADVANCED) 2020
ENTHUSIAST & LEADER
Path to success KOTA (RAJASTHAN)
COURSE
4k 2 4
- <4
25 5
k2 – 5 < 25 Þ k2 < 30

ÞkÎ .......(1)

and |x1 – x2| > 1 Þ (x1 – x2)2 > 1 Þ (x1 + x2)2 – 4x1x2 > 1
4k 2 4
- >1
25 5

4k 2 9 45
> Þ k2 >
25 5 4

Þ kÎ ..........(2)

(1) and (2) Þ positive integral value of k are 4 and 5.


Hence sum = 9 Ans.
x1
Alternatively :f(x) = 5x2 – 2kx + 1
x2
Since k is positive
Hence vertex of f(x) lies on positive x-axis.
1
Also product of root is
5
1
so both roots of f(x) are positive and smaller root x1 has to be positive fraction because if x1 > 1, then for x1x2 = , x has
5 2
to be fraction which is not possible.
Hence for exactly one integral solution of f(x) < 0, f(1)<0 and f(2) > 0
f(1) = 6 – 2k < 0 Þ k > 3 ........(1)
21
and f(2) = 21 – 4k > 0 Þ k < ........(2)
4
(1) and (2) Þ positive integral values of k are 4 and 5
Hence sum = 9
25. Ans. [0005] Let f(x) = x2 – 2(a + 1) x + 3(a – 3) (a + 1)

Case - I :

f(0) < 0 Þ (a – 3) (a + 1) < 0 Þ –1 < a < 3

Case - II :

(i)) f (0) ³ 0 Þ (a - 3)(a + 1) ³ 0


Þ a = ( -¥, - 1] È [3, ¥)
(ii) D > 0
4(a + 1)2 – 4·3(a – 3)(a + 1)] > 0
a2 + 2a + 1 – 3(a2 – 2a – 3) > 0
– 2a2 + 8a + 10 > 0
MATHEMATICS /AQ # 10 E-141
TM TARGET: JEE (MAIN + ADVANCED) 2020
ENTHUSIAST & LEADER
Path to success KOTA (RAJASTHAN)
COURSE
a2 – 4a – 5 < 0
(a – 5)(a + 1) < 0
\ a Î ( -1,5)

-B
(iii) > 0 (a + 1) > 0
2A
\ a > –1
\ from (i) (i) Ç (ii) Ç (iii) we get a Î [3, 5)
Case - I È Case - II Þ a Î (–1, 3) È [3, 5)
So, there are 5 possible integral values of a i.e., a = 0, 1, 2, 3, 4

3x 2 + 9x + 17
26. Ans. [0041]Let y
3x 2 + 9x + 7

Þ 3x2(y – 1) + 9x(y – 1) + 7y – 17 = 0
since x is real, so,

D³0
Þ 81 (y – 1)2 – 4 × 3 (y – 1) (7y – 17) ³ 0
Þ (y – 1) (y – 41) £ 0 Þ 1 £ y £ 41.

Therefore, the maximum value of y is 41.

MATCH THE COLUMN :


27. Ans. [A–s, B–p,q, C–r]

We have

æ4-x ö
log10 ç ÷
è 10 ø = log log p - 1 1
1+
log10 x
( 10 ( 10 ) ) log10 x

[Note that p > 1]

æ4-xö æ æ log10 p ö ö
log10 x + log10 ç ÷ ç log10 ç ÷÷
Þ è 10 ø = è è 10 ø ø
log10 x log10 x

æ æ 4 - x öö æ log10 p ö
Þ log10 ç x ç ÷ ÷ = log10 ç ÷
è è 10 ø ø è 10 ø

x (4 - x) log10 p
Þ = Þ x 2 - 4x + log10 p = 0
10 10

\ x1 , x 2 = 2 ± 4 - log10 p

Domain: 4 - log10 p > 0 Þ log10 p < 4


\ 1 < p < 104 and p ¹ 103 (if p = 103 Þ x = 1 which is not possible]
Case I : If 1 < p < 104 Þ 2 distinct roots
Case II : If p = 104 Þ there is only one root x = 2
Case III : If p = 103 Þ there is only one root x = 3
Case IV : If p > 104 Þ there is no real roots ]

MATHEMATICS /AQ # 10 E-142


TM TM TARGET:JEE
TARGET: JEE(Main
(MAIN ++Advanced)
ADVANCED) 2020
2020
ENTHUSIAST&
ENTHUSIAST & LEADER
LEADER
Path to success KOTA (RAJASTHAN)
COURSE
Path to success KOTA (RAJASTHAN ) COURSE

ADVANCED QUIZ # 11(SOLUTION) PROGRESSION & BINOMIAL THEOREM MATHEMATICS


SINGLE CORRECT CHOICE TYPE

( k + 2) k - k k + 2
1.Ans. (D) Tk =
k ( k + 2) - k 2 ( k + 2)
2

( k + 2) k -k k +2 1é 1 1 ù
= = ê - ú
2k ( k + 2 ) 2ë k k+2û

1é 1 1 ù 1é 1 1 ù
T1 = - T2 = -
2 êë 1 ú
3û 2 êë 2 ú

1é 1 1 ù
T3 = -
2 êë 3 ú and so on

\ as k ® ¥, sum

1é 1 ù 1+ 2 1+ 2
= ê1+ ú = = Þ a + b + c = 11
2ë 2û 2 2 8
2. Ans. (D) coefficient of A in nth term = 8 + (n – 1) (–2) = 10 – 2n
coefficient of B in nth term = 2 + (n – 1)(–1) = 3 – n
10 – 2n = 2(3 – n) Þ 10 = 6
which is absurd Þ none ]

n 1 2 3 4
3.Ans. (C) å (-1)
n +1
= - + - + ...
5n 5 52 53 54

æ 1 ö æ ,1ö
3
1
[1 ∗ 2çç ÷÷÷ ∗ 3çç ÷÷÷ ∗ .........⁄]
çè 5 ø èç 5 ø
= 5 E555555555555555 F
P

2
æ 1ö æ 1ö
P = 1 + 2 ç - ÷ + 3 ç - ÷ + .......
è 5ø è 5ø
2
-1 æ 1ö æ 1ö
P = +1ç - ÷ + 2 ç - ÷ + .....
5 è 5ø è 5ø
6 1 1 1
P = 1- + 2 - 3
5 5 5 5

6 1 5 25
P= = ÞP=
5 1 6 36
1+
5

1 25 5
Given summation is equal to ´ =
5 36 36

AR 7 8!
4. Ans. (A) AR4 = 7!, AR7 = 8! Þ = =8ÞR = 2
AR 4 7!
AR4 = 7! Þ A.16 = 7! Þ A = 15 ´ 21
(2n - 1)
\ 15.21 = 2205 Þ n = 3
2 -1

MATHEMATICS /AQ # 11 E-143


TM TARGET: JEE (MAIN + ADVANCED) 2020
ENTHUSIAST & LEADER
Path to success KOTA (RAJASTHAN)
COURSE
5. Ans. (C) Let the roots are x1 , x 2 , x 3 , x 4
Þ x1 + x 2 + x 3 + x 4 = 12 and x1x 2 x 3 x 4 = 81

x1 + x 2 + x 3 + x 4
Þ = 3 and ( x1 x 2 x 3 x 4 )1 / 4 = 3
4
since A.M. = G.M. Þ x1 = x 2 = x 3 = x 4 = 3

Þ x 4 - 12x 3 + bx 3 + cx + 81 = ( x - 3 )
4

Þ b = 54, c = -108

6. Ans. (B) Concept : Coefficient of xr in (1 - x ) - n , n Î N is n + r -1


Cr .

2
now given product is = 1 - x . æ 1 - x ö
28 15

ç ÷
1- x è 1- x ø

(1 - x )(1 - x )28 15 2
(1 - x )(1 - 2x )
28 15

= = = (1 - 2x15 - x 28 ) (1 - x )
-3

(1 - x ) (1 - x )
3 3

Hence coefficient of x 28 in (1 - 2x15 - x 28 ) (1 - x ) is


-3

= 30
C 2 - 2. 15 C 2 - 1 = 435 - 210 - 1 = 224 Ans. ]

2 3
7. Ans. (A) 31.20 C0 + 3 .20 C1 + 3 .20 C2 + ...........upto 21 terms
2 3
20 20
20 Cr 20 r +1 C r +1 1 20 r +1 21
= å 3r +1 =å3 = å3 C r +1
r =0 r + 1 r =0 21 21 r =0

1 4 21 - 1 242 - 1 b 21 1
= [(1 + 3)21 - 1] = = Þ a = 42, b = 21 Þ = =
21 21 21 a 42 2

k(n)!
8. Ans. (C) As k n Ck = = n ( n -1 Ck -1 )
(n - k)!(k)!

( Ck ) = n 2 å ( n -1 C k -1 ) = n 2
n 2 n 2
\ åk
2 n

k =1 k =1

é As n n C 2 = 2n C ù
å ( n -1 Ck ) = n 2 (2n - 2 Cn -1 ) ( k)
n -1 2

k=0
êë kå=0

û

So, f (5) = 25(8 C4 ) = 25 ´ 8!


4!4!

25 ´ 8 ´ 7 ´ 6 ´ 5
= = 70 ´ 25 = 1750
24
9. Ans. (D) The last digit in powers of 3 are
3, 9, 7, 1, 3, 9, etc
Such that the same last digit occurs if the power is increased by 4
\ The last digit of 843843 is 7
Now, the last in powers of 2 are
2, 4, 8, 6, 2, 4, etc
Similarly, as 295 = 73 × 4 + 3
Hence, the last digit in the given numbers is 5

MATHEMATICS /AQ # 11 E-144


TM TARGET: JEE (MAIN + ADVANCED) 2020
ENTHUSIAST & LEADER
Path to success KOTA (RAJASTHAN)
COURSE
10.Ans. (A) Power cycle of 6 = 6, 6, 6, 6, 6, ..... = 1
Power cycle of 4 = 4, 6, 4, 6, 4,....... = 2
Now, Units digit in (346)44 = 6
and Units digit in (344)45 = 4
Hence, the digiti in units place of
(346)44 + (344)45 = XX.......X 0
11. Ans. (A) (1 + ax + bx2) [C0 – C1(2x) + C2(2x)2 – C3(2x)3 + C4(2x)4 – ...]
Coefficients of x4 and x3 are both zero.
18
C4.24 – 18C3 23a + 18C2 · 22b = 0
and –18C3 · 23 + 18C2 · 22a – 18C1b 2 = 0
\ 32a – 3b = 240
and 51a – 3b = 544

272
Solving, a = 16 and b =
3

r.100 Cr r.100! ( r - 1)!(101 - r ) !


12.Ans. (C) 100 C = r! 100 - r ! . = 101 - r
r -1 ( ) 100!

100
\ S = å (101 - r ) = 100 + 99 + 98 + ..... + 1 = 5050
r =1

13. Ans. (D) The coefficient of a2 b3 c4 in the expansion of {( 3a + b ) - c}


9

= ( -1) 9C4 ´ the coefficient of a2 b3 in (3a + b)5

9 5
= 9 C4 .32. 5 C 2 = ´ 9´
45 23

9´5´ 6´ 7´8´9
= = 11340
12
Paragraph for question Nos. 14 to 16
Sol. [14-C ; 15-B ; 16-D]
R = (1 + 2x)n
put x = 1 to get sum of all the coefficients
\ 3n = 6561 = 38 Þn=8
1
( )
8
Sol.14 (C) for x = ; R= 2 +1
2

( 2 + 1) + ( 2 - 1)
8 8

consider 144424443
I + f + f¢

= 2 é 8 C0 ( 2) + ......ù = even integer


8

êë úû
since I is integer Þ f + f¢ must be an integer
but 0 < f + f¢ < 2 Þ f + f¢ = 1 Þ f¢ = 1 – f
now n + R – Rf
n + R(1 – f)

( ) ( )
n n
= 8+ 2 +1 × 2 -1 = 8 + 1 = 9 Ans.

MATHEMATICS /AQ # 11 E-145


TM TARGET: JEE (MAIN + ADVANCED) 2020
ENTHUSIAST & LEADER
Path to success KOTA (RAJASTHAN)
COURSE
1
Sol.15 (B) Tr + 1 in (1 + 2x)8 = 8Cr (2x)r = 8Cr when x =
2
now Tr + 1 ³ Tr
8
Tr +1 C
³1 Þ 8 r ³1
Tr C r -1

8! (r - 1)!(9 - r)!
Tr + 1 ³ Tr × ³1
r!(8 - r)! 8!
for r = 1, 2, 3, 4 this is true
i.e. T5 > T4
but for r = 5 T6 < T5
Þ T5 is the greastest term Þ (B)

1
9 2.
(n + 1) | x | 2
r= = = 4.5
1+ | x | 1
1 + 2.
2

r = 4.5 = Rational
\ r = [4.5] = 4
Hence T5- is greatest form
Sol.16 (D) Again Tk + 1 = 8Ck × 2k × xk ; Tk = 8Ck – 1 × 2k – 1 × xk – 1
Tk – 1 = 8Ck – 2 × 2k – 2 × xk – 2
We want to find the term having the greatest coefficient
\ 2k – 1 × 8Ck – 1 ³ 2k × 8Ck .............. (1)
And 2k – 1× 8Ck – 1 ³ 2k – 2 × 8Ck – 2 .............. (2)

8! × 2k -1 2k × 8! 1 2
From (1) ³ Þ (9 - k) ³ k Þ k ³ 18 ³ 3k Þ k = 6.
(k - 1)!(9 - k)! k!(8 - k)!
Again 2k – 1 × 8Ck – 1 ³ 2k – 2 × 8Ck – 2

8! × 2k -1 2k - 2 × 8! 2 1
³ Þ ³
(k - 1)!(9 - k)! (k - 2)!(10 - k)! k - 1 10 - k
Þ 20 – 2k ³ k – 1 Þ 21 ³ 3k Þk£7
Þ6£ k³7
Þ T6 and T7 term has the greatest coefficient
Þ k = 6 or 7
Þ sum = 6 + 7 = 13 Ans.
MULTIPLE CORRECT CHOICE TYPE
17.Ans. (CD) We have
100
x1 + x2 + .........+x100 = (x1 + x100) = –1........(1)
2
50
and x2 + x4 + ........ + x100 = (x + d + x100) = 1.........(2)
2 1
3
\ Solving (1) and (2), we get d =
50
-149
So, x1 =
50
74
Also, x100 =
25
MATHEMATICS /AQ # 11 E-146
TM TARGET: JEE (MAIN + ADVANCED) 2020
ENTHUSIAST & LEADER
Path to success KOTA (RAJASTHAN)
COURSE

47
25 = 94 = 2
Now, sum of in G.P. =
3 47
1-
50
18.Ans. (BCD) Let a and d be the first term and common difference of arithmetic progression. Also, let b and r be the first term
and common ratio of geometric progression.

a + 8d = b ...........(1) ü
ï b
We have a + 12d = br..........(2) ý Also = 80
ï 1 - r
a + 14d = br .........(3) þ
2

(2) – (1) Þ 4d = b(r – 1) ...........(4)


(3) – (2) Þ 2d = br(r – 1) ...........(6)
Now,
1 1
(4) ¸ (3) Þ 2 = Þ r =
r 2
so, 2b = 80 Þ b = 40
æ -1 ö
and 4d = 40 ç ÷ = -20 Þ d = -5
è 2 ø
Also, a – 40 = 40 Þ a = 80

a = 80 ü
d = -5 ïï
ï
Hence, b = 40 ý
ï
1
r= ï
2 ïþ

Now verify alternatives


19. Ans. (ABD) 22 + 42 + 62...... - (12 + 32 + ....)
= 3 + 7 + 11 +........
so term = 25(6 + 49 × 4) = 25(202) = 5050
(x + 1)
(A) y simplifies to x Þ y(x = 100)
2
100.101
= = 5050 Þ (A) is correct
2
(B) f(1) = 1
f(2) = f(1) + 2 = 1 + 2 = 3
f(3) = 3 + f(2) = 1 + 2 + 3 = 6
f(4) = 4 + f(3) = 1 + 2 + 3 + 4 = 10 and so on
f(100) = 1 + 2 + 3 + ......+ 100 = 5050 Þ (B) is correct
let f(n) = Tn
Alternatively : use f(n) – f(n – 1) = n
Put n = 100, 99, ........1 and add
(C) 51 + 61 + 71 + .........+ 341
n = 30; s = 5880. Which is not correct.
100
1 1
(D) åH
i =1
= 100.
H where H is the single H.M. between 1 and 100
i

2 × (1/100) 2
Hence H = =
1 101
1+
100

MATHEMATICS /AQ # 11 E-147


TM TARGET: JEE (MAIN + ADVANCED) 2020
ENTHUSIAST & LEADER
Path to success KOTA (RAJASTHAN)
COURSE
20.Ans. (CD) No of ways to distrbute n different things among three boys.
n n n ænöænöænö
= å å å ç ÷ ç ÷ ç ÷ = 23n
i = 0 j= 0 k = 0 è i ø è j ø è k ø

for n =3, E = 29 = 512; for n = 4, E = 212 = 4096 Þ (C) and (D)


n
21. Ans.(BC) æç x1/ 2 + 1 x -1/ 4 ö÷
è 2 ø
n -r -r
1
Tr +1 = n C r x 2
. ×x 4
2r
1 1
coefficient of the 1st 3 terms are n C0 , n C1 × , n C 2 × 2
2 2
1 1
\ n
C0 + n C2 × = 2 ×n C1 ×
4 2
n(n - 1)
1+ =n
8
n(n - 1)
\ = (n - 1) Þ n = 8 (as n ¹ 1)
8
8- r æ 3r ö
1 - 4r 8 1 ç 4- ÷
\ Tr +1 =8 Cr x r
×
× x = Cr × r × xè 4 ø
2
2 2
terms of x with integer power occur when r = 0, 4, 8 Þ 3 terms hence B/C are correct. ]
22. Ans.(ABCD) Ratio
1 × 3 × 5......(2n - 1) (2n)! 1
= = n = 2n Cn × 2n
2 × 4 × 6......2n (2 × n!) 2 2
verify all the alternatives, noting the fact that C0 + C1 + C2 + ........+ Cn = 2n and
C 20 + C12 + C 22 + ........ + C 2n =2 n C n
23. Ans.(AD) (1 + z)3 where z = x (1 + 2x + 3x2)
1 + 3C1 z + 3C2 z2 + 3C3z3
coefficient of x3 in (1 + z)3
3
C1 (3) + 3C2(4) + 3C3(1) =22
Þ a = 22
now again (1 + y)3
where y = x (1 + 2x + 3x2 + 4x3)
(1 + y)3 = 1 + 3C1 y + 3C2y2 + 3C3y3
\ coefficient of x3 is
3
C1 (3) + 3 C 2 (4) +3 C 3 (1)
= 9 + 12 + 1 = 22
Þ b = 22
Hence a = b Þ a + b = 44 Ans.]
10 20
æ 2 1 ö æ 1ö
24. Ans.(ABCD) ç x + 2
- 2÷ = ç x - ÷
è x ø è xø

1 20 20 – 2r
Tr + 1 = 20Crx20 – r(–1)r
= Crx (–1)r
xr
\ 20 – 2r = 0 Þ r = 10
So, term independent of x = 20C10. Now verify alternatives

MATHEMATICS /AQ # 11 E-148


TM TARGET: JEE (MAIN + ADVANCED) 2020
ENTHUSIAST & LEADER
Path to success KOTA (RAJASTHAN)
COURSE
25.Ans. (AC) 4 C4 · 100 C6 + 4 C3 · 100C7 + 4C 2 · 100 C8
+ 4 C1 · 100 C 9 + 4 C 0 · 100 C10
Out of 104 students of which 100 are boys and 4 are girls, we have to select 10 students.
This can be done in 104C10 = 104C94
Hence, x + y = 114 or 198
C6 + 100 C7 + 3 ( 100 C7 + 100 C8 )
100
Aliter: 14
4244 3

+3 ( 100 C8 + 100 C9 ) +100 C 9 + 100 C10


= 101C7 + 3 101C8 + 3 101C9 + 101C10
(14C4244
101
+ C ) + 2(
3
7
101
8
101
C8 + 101C9 ) +101 C9 + 100 C10

= 102 C8 + 2· 102 C 9 + 102 C10 = 102 C8 + 102 C9 + 102 C9 + 102 C10

= 103 C9 +103 C10 = 104 C10 º 104 C94


Þ x = 104 and y = 10 or x = 104 ond y = 94
Hence, (x + y) = 114 or 198
26. Ans.(ABC) (1 + x)n = C0 + C1x + C2x2 + C3x3 + ....... + Cnxn ......(1)
Puting x = i
(1 + i) n = C0 + C1i - C 2
-C3i + C 4 + C5i - C6 - .....
n
ì æ p p öü
Þ í 2 ç cos + i sin ÷ ý = (C0 - C 2 + C 4 ......) +i ( C1 - C3 + C5 - ......)
î è 4 4 øþ
Equating real and imaginary parts
np
C0 - C2 + C4 .... = 2n / 2 cos OPTION (A)
4
np
C1 - C3 + C5 .... = 2n / 2 sin OPTION (B)
4
Again,
1
C0 + C2 x 2 + C 4 x 4 + .... = é(1 + x) n + (1 - x) n ùû

Putting x = 1, we get
1 n
C0 + C2 + C4 + ............ = (2 + 0) .........(2)
2
Puting x = i, we have
1 1
, {(1 + i) n + (1 - i) n } .........(3)
C0 – C2 + C4 – ...... =
2 2
Adding (2) and (3), we have

1æ n np ö
2 ( C0 + C4 + C8 + .....) = 2 n -1 + ç 2 2 .2 cos ÷
2è 4 ø

n
-1 np
Þ (C 0 + C 4 + C 8 + ....) = 2 n - 2 + 2 2 cos
4
æ n-2 ö
ç ÷ np
\ (C0 + C4 + C8 + ......) = 2 n - 2 + 2è 2 ø
cos
4
MATHEMATICS /AQ # 11 E-149
TM TARGET: JEE (MAIN + ADVANCED) 2020
ENTHUSIAST & LEADER
Path to success KOTA (RAJASTHAN)
COURSE
27.Ans. (ABC) According to Multinomial Theorem, we have
20! a b c
(x + y + z) 20 = å x yz
a!.b!.c!
Case I : Option (A)
Þ a + b + c = 7 + 8 + 7 = 22 > 20
does not exists
\ Coefficient of x7x8y7 = 0
Case II : Option (B)
Total number of distinct terms = 20 + 3 – 1C20 = 22C20 = 231
Case III : Option (C)
Every term is of the form
20!
x 20 - r .y r - k .z k
(20 - r)!.(r - k)!.k!
Þ a = 20 – r, b = r – k, c = k
Þ a + b + c = 20

28. Ans.(ABC) Since, ( 5 5 + 11)


2n +1
= I + f,0 < f <1

( )
2n +1
Let 5 5 + 11 = f ' where 0 < f¢ < 1

Subtracting
I + f - f ' = (5 5 + 11) 2n +1 - (5 5 - 11) 2n +1
ÞI+ f -f ' = 2

é 2n +1 C1 (5 5) 2n (11)1 + 2n +1 C 3 (5 5)2n - 2 (11)3 + .......ù ......(1)


ë û
\ I + f - f ' = EVEN integer
Since 0 < f < 1 and 0 < f¢ < 1, f – f¢ = 0
or f = f¢
Hence from (1), I is an even integer OPTION (A)
Also on taking 11 common we have

I = 22 éë 2n +1 C1 (5 5) 2n + 2n +1 C3 (5 5) 2n -2 112 + ......ùû
Hence I is divisible by 22 OPTION (C)
2n +1
Again (I + f) 2 = éë(5 5 + 11) 2 ùû

= [125 + 121 + 110 5]2n +1

= (246 + 110 5) 2n +1 = 22n +1 (123 + 55 5) 2n +1

INTEGER ANSWER TYPE

æ ö 100 100 k
29. Ans. [0201] Let S = å ç k + 1 ÷ Ck
k =0 è ø

( (k + 1) - 1) 100 100 100 C


Ck = æç å 100 C k ö÷ - å
100 100
=å k

k =0 (k + 1 è k =0 ø k =0 k + 1

1 100 æ k + 1 101 ö 1 100 101


= 2100 - åç Ck +1 ÷ = 2100 - å Ck +1
101 k =0 è k + 1 ø 101 k =0

MATHEMATICS /AQ # 11 E-150


TM TARGET: JEE (MAIN + ADVANCED) 2020
ENTHUSIAST & LEADER
Path to success KOTA (RAJASTHAN)
COURSE

æ 2101 - 1 ö (101)2100 - 2101 + 1


= 2100 - ç ÷=
è 101 ø 101

99(2100 ) + 1 a(2100 ) + b
= = (given)
101 c
So, a = 99, b = 1, c = 101
Hence, (a + b + c)least = 99 + 101 + 1 = 201

15
æ 1 1 ö
15
æ x3 + x + x 4 + 1 ö
30. Ans. [0061] ç x + + x 2
+ ÷ =ç x2
÷
è x x2 ø è ø

a 0 + a1 x + a 2 x 2 + ..... + a 60 x 60
=
x 30
Hence, the total number of terms is 61.
31. Ans. [0081] (x – 2)5 (x + 1)5
= [5C0x5 – 5C1x4 × 2 + ....] [5C0 + 5C1x + ....]
Þ Coefficient of x5
= 5 C 05 C 0 - 5 C1 .25 C1 + 5 C 2 2 2.5 C 2

-5 C 3 23 5 C 3 + 5 C 4 24 5 C 4 -5 C 5 25 5 C 5
= 1 – 5 × 5 × 2 + 10 × 10 × 4
– 10 × 10 × 8 + 5 × 5 × 16 – 32
= –81
Absolute value is 81
32. Ans. [0000] (1 – x)2008 (1+x + x2)2007
= (1 – x) [(1 – x) (1 + x + x2)]2007
= (1 – x) (1 – x3)2007
= (1 – x3)2007 – x(1–x3)2006
All the terms in the expansion of (1 – x3)2007 are of the from x3r and all the terms in the expansion of x(1 – x3)2007 are of the
form x3r + 1 where as x2009 is of the from x3r + 2. Thus, the desired coefficient is 0.

MATCH THE COLUMN :


33. Ans. (A – r; B – p; C – t; D – q)

( 20 + 3 -1) 22 ´ 21
(A) C3 -1 = 22 C2 = = 231
2
n 8
(B) + 1 = + 1 = 4 + 1 = 5.
2 2
(C) Irrational terms = Total no. of terms - rational terms = 101 – 4 = 97
100
(since = 4 + F.T1 ,T25 , T49 ,T73 are rationals)
24
(D) Put x = y = 1
15
æ 1 2ö
Þ ç 1 + + ÷ = 215
è 3 3ø

MATHEMATICS /AQ # 11 E-151


TM TARGET: JEE (MAIN + ADVANCED) 2020
ENTHUSIAST & LEADER
Path to success KOTA (RAJASTHAN)
COURSE
34. Ans. [A– r ; B– s ; C–p ; D–q ]

(A) (5 + 2 ) x - (4 + 5 ) x + 8 + 2
2
5 x1
x2

2x1 x 2 2 8+ 2 5 ( )
H.M. = x + x = = 4 Ans.
1 2 4+ 5 ( ) Þ (R)

1 1
(B) a1 + 9d = 3 and = + 9d1
h10 h1

1 1 1 1 1
2 + 9d = 3; = + 9d1 ; d= 9d1 = - Þ d1 = -
3 2 9 6 54

1 7 1 1 æ 1 ö 1 1 7
\ a 4 = 2 + 3d = 2 + = = + 6ç - ÷ = - =
3 3 h7 2 è 54 ø 2 9 18

7 18
\ a 4 h 7 = ´ = 6 Ans. Þ (S)
3 7
(C) 3x + 4(mx + 1) = 9
3x + 4mx = 5

5
x=
3 + 4m
now intercept for x to be integer m = – 1 or m = – 2 Þ 2 integral values Þ (P)
(D) Product of n geometric means between two numbers is equal to nth power of
single geometric mean between them

( 10 )
6
= (10 ) Þ n = 3
3
Þ

MATHEMATICS /AQ # 11 E-152


TM TM TARGET:JEE
TARGET: JEE(Main
(MAIN ++Advanced)
ADVANCED) 2020
2020
ENTHUSIAST&
ENTHUSIAST & LEADER
LEADER
Path to success KOTA (RAJASTHAN)
COURSE
Path to success KOTA (RAJASTHAN ) COURSE

ADVANCED QUIZ # 12(SOLUTION) COMPLEX NUMBER MATHEMATICS

SINGLE CORRECT CHOICE TYPE


1. Ans. (B) z = - z
zz – 4zi = z
2

Þ z – 4i = z Þ 2z = – 4i
z = – 2i Hence, | z | = 2

z+ z z-z
2. Ans. (B) We have + = 1 Þ |x| +|y| = 1
2 2

Also, |z – i| + |z + i| = 2
(0, 1) and (0, –1)
So, number of solution is 2
i.e., z = i and –i

æ ö 1 1
3. Ans. (C) We have Re ç z ÷ = 2
è ø

æ x - iy ö 1
Þ Re ç 2 2 ÷
= Þ x 2 + y 2 - 2x = 0
è x + y ø 2
Þ (x –1)2 + y2 = 1
Clearly, maximum distance of M(–2, 4) from circle is = 1 + (-2 - 1) 2 + (4 - 0) 2

= 1 + 9 + 16 = 1 + 5 = 6
4. Ans. (D) ia3 + a2 – a + i = 0 Þ a2 (ia + 1) + i(ia + 1) = 0
1 1
Þa= – or a = – i Þ |a| = | - | or |a| = | – i| Þ |a| =1
i i
a lies on a circle of radius 1 with centre at origin
|a – 3 – 4i|max = 1 + 9 + 16 = 6
5. Ans. (D) We have (x – a1) (x – a2) .....(x – a30) = 1 + x + x2 + .... + x30
30

Taking logarithms to the base e å log ( x - a ) = log (1 + x + x


i =1
i
2
+ .... + x 30 )

Differentiating with respect to x


30
1
= 1 + 2x + 3x2 + .... + 30x
2 29
å x-a
i =1 1 + x + x + ..... + x 30
i

Put x = 1
30
1 1 + 2 + ... + 30 30
1
å1- a = = 15 Þ å (a = -15
i =1 1 31 i =1 i - 1)

MATHEMATICS /AQ # 12 E-153


TM TARGET: JEE (MAIN + ADVANCED) 2020
ENTHUSIAST & LEADER
Path to success KOTA (RAJASTHAN)
COURSE

æ 2z 2 - 5z + 3 ö p
6.Ans. (D)arg 3z 2 - z - 2 ÷ = 2
ç
è ø

æ (2z - 3)(z - 1) ö p æ 2z - 3 ö p
arg ç ÷= ; arg ç ÷=
è (3z + 2)(z - 1) ø 2 è 3z + 2 ø 2

æ 3ö æ 3ö
ç 2 z- 2 ÷ p ç z- 2 ÷ p
arg ç . ÷= arg ç ÷=
çç 3 z + 2 ÷÷ 2 ; çç z + 2 ÷÷ 2
è 3ø è 3ø

3 -2
Locus of z is a semicircle described on the line segment joining z = and z = as diameter
2 3

1- an = 1 - i
7. Ans. (D). 1 + a + a2 + ...... + a n -1 = ip
1- a 1 - e 2n

1
1- i 2
= ée ù
ip ip
Qa = i
1
n 2
n
=e 2n = = 2 1
ëê ûú ip = =
1- e 2n
æ p ö
2
2 p p p
2 - 2 cos 2.sin
ç 1 - cos 2n ÷ + sin 2n 2n 4n
è ø

1 p p p
= cosec = cos cosec
2 4n 4 4n

8. Ans. (B) We know that z - z1 2 + z - z 2 2 + z - z3 2 attains the least value, if z coincides with the centorid of the triangle
formed by z1,z2,z3.

0 + 6 + 9i
Here, the centroid of the triangle is = 2 + 3i
3
\ the required minimum value
2 2 2
= 2 + 3i + 2 + 3i - 6 + 2 + 3i - 9i

= 22 + 32 + 42 + 32 + 2 2 + 6 2 = 78
Second Method
Also 3x2 + 3y2 -12x-18y+117
Þ [(x – 2)2 + (y – 3)2] + 78
min value is 78
9. Ans. (D) Given, x4 – 2x2 + 4 = 0
Þ x4 – 2x2 + 1 = –3
Þ (x2 – 1)2 = – 3
Þ x2 = 1 ± i 3
Þ x2 = 2eia/3, 2e–ia/3 {using eiq = cos q + i sin q}
The roots of the equation are

2eip / 6 , - 2eip / 6 , 2eip / 6 , - 2e-ip / 6

and these lie in a circle of radius 2.

p
As the diagonals intersect at an angle of , they are the vertices of an inscribed rectangle
3
MATHEMATICS /AQ # 12 E-154
TM TARGET: JEE (MAIN + ADVANCED) 2020
ENTHUSIAST & LEADER
Path to success KOTA (RAJASTHAN)
COURSE
10. Ans. (B) Given, xn – 1 = (x – 1)(x – a) (x – a2) .......(x – an – 1)
Þ log(xn – 1) = log(x – 1) + log(x – a) + log(x – a2) + ......+log(x – an – 1)
Differentiating w.r.t. x, we have
nx n -1 1 1 1 1
= + + + ....... +
x -1 x -1 x - a x - a
n 2
x - a n -1
Substituting x = 2, we get
n.2n -1 1 1 1 1
= + + + ....... +
2 -1 1 2 - a 2 - a
n 2
2 - an -1

n.2 n -1 n -1
1
Þ -1 = å
2 -1
n
k =1 2 - ak

n.2n -1 - 2 n + 1 n -1 1
Þ =å
2n - 1 k =1 2 - a
k

n -1
1 (n - 2)2n -1 + 1
\ å
k =1 2 - a
k
=
2n - 1

é wr w r +1 w r +2 ù
ê ú
11. Ans. (C) A(r) = ê w r -1 wr w r +1 ú
ê w 2r w 2r + 2 w 2r + 4 úû
ë

1 w w2
r -1
r
w .w .w 12r
w w2 = 0
=
1 w2 w4

12. Ans.(C) For |z – 1| to be maximum, z = -4


z - w - w -4 + 1
Centroid (a) = = = -1
3 3
\ Re (a) = –1.
13. Ans.(D) Equation of the line is i(x + iy) = x – iy Þ ix – x – y + iy = 0 Þ (i – 1)x + (i – 1) y = 0
Þ (i – 1) (x + y) = 0 Þ x + y = 0 Þ y = – x
The image of (2, –1) on y = – x is (1, – 2)
The image is 1 – 2i
14. Ans.(A) ( z + i )3 = ( z - i )3
ie. z3 + 3z 2i + 3zi 2 + i3 = z 3 - 3z 2i + 3zi 2 - i3
1
Þ 6z 2 i + 2i 3 = 0 Þ 3z 2 - 1 = 0 \z = ± are real
3

-1 + i 3 -1 - i 3
15. Ans.(B) We have w = and w2 = is
2 2

( -1 + 3)30 ( -1 - i 3)30 (2w)30 (2w2 )30


\ + = +
(1 - i) 20 (1 + i) 20 (1 - i) 20 (1 + i) 20

ì ü
ï30 ï
2 ï 1 1 ï
= 10 í + 20 ý
ì 1 1 ü
2 ïæ 1- i ö 20
æ 1+ i ö ï = 2 20 í -15p + i5p ý
îe e þ
ï çè 2 ÷ø ç ÷
è 2 ø ïþ
î

= 2 20 {2 cos 5p} = -221

MATHEMATICS /AQ # 12 E-155


TM TARGET: JEE (MAIN + ADVANCED) 2020
ENTHUSIAST & LEADER
Path to success KOTA (RAJASTHAN)
COURSE
Paragraph for Question Nos. 16 to 18
Sol. [16. A, 17. A, 18. C]
(C)

AB = 2ae,
If |OA – OB| = 2a, it is a hyperbola
|OA + OB| = 2a it is an ellipse
AB = |z1 – z2| = 2ae,
Hyperbola Þ ||z1| – |z2|| = 2a
z1 - z 2
Þ =e
z1 - z 2

2ae = ||z1| – |z2|| = 8, |z1| + |z2| = 18

68 17
Hyperbola : e = = ,Ellipse |z1| + |z2| = 2a
8 4

68 17
Ellipse : e = =
18 9
(C) If ‘O’ is outside the ellipse Þ OA + OB > 2a
Þ |z1| + |z2| > 2a
z1 - z 2 17
2ae = |z1 – z2| Þ z + z < e Þ 1 > e >
1 2 9

MULTIPLE CORRECT CHOICE TYPE


(1 - z) 2 1 - z zz - z z - 1
19.Ans. (B, C) Given w = = = =
1 - z2 1 + z zz + z z + 1
æ1- z ö
= - çç ÷÷ = -w (As | z | = 1 Þ zz = 1)
è 1+ z ø
\w + w = 0 Þ w is purely imaginary. Hence w lies on y-axis.
(A) z lies on perpendicular bisector of (2, 4) and (2, –4) which is x-asis
(B) z lies on perpendicular bisector of (3, –4) and (–3, –4) which is y-axis
(C) z lies on perpendicular bisector of (–2, 0) and (2, 0) which is y-axis
(D) z lies on either of the rays emanating from (0, 1) and (0, –1) towards +¥ and –¥ respectively along y-axis, not
complete y-axis.
20. Ans.(A, B)
m
é ù
ê m -1 ú
4n +1 ê
ì æ 2pk ö æ 2pk ö ü ú
å ísin ç ÷ - i cos èç m ø÷ý ú
Let S = å ê k =1 î è m ø
m =1 ê 14444244443þ ú
ê 144444244444 tk

ê S1 ú
ë û

æ 2pk ö æ 2pk ö
Now, let tk = sin ç ÷ - i cos ç ÷
è m ø è m ø

ì æ 2pk ö æ 2pk ö ü
Þ tk = -i ícos ç ÷ + i sin ç ÷ý
î è m ø è m øþ
Apply summation. Operator and check

MATHEMATICS /AQ # 12 E-156


TM TARGET: JEE (MAIN + ADVANCED) 2020
ENTHUSIAST & LEADER
Path to success KOTA (RAJASTHAN)
COURSE
21. Ans.(A, C) iz3 + 3z 2 - z + 3i = iz 2 ( z - 3i ) - 1( z - 3i )

= ( iz 2 - 1) ( z - 3i ) = 0

Þ iz 2 = 1 or z = 3i Þ z = 1or 3

INTEGER ANSWER TYPE


22. Ans. [0004] mBD = –2

1
mAC =
2
parametric equation of AC is

x - 2 y +1 5
= =±
cos q sin q 2

2 1
where cos = ;sin q =
5 5

1
x = 2 ± 1 and y = –1 ±
2

æ -1 ö æ -3 ö
ç 3, ÷ or ç 1, ÷
è 2 ø è 2 ø

x1 x 2 3·4
= =4
y1 y 2 3

23. Ans. [0673] Let z = a + ib hence a, b Î [–2, 2]


(a + 1) + ib
w= ; 2
(a - 1) + ib a + b = 4
2

(a + 1)2 + b 2 a 2 + b 2 + 2a + 1 5 + 2a
| w |= = =
(a - 1) + b
2 2
a + b - 2a + 1
2 2
5 - 2a

5+ 4
| w |max = = 3 = M , when a = 2 (z = 2)
1

5- 4 1
| w |min = = = m , when a = –2 (z = –2)
9 3
Hence (2010 m + M) = 673
24. Ans. [0005] We have
z1 (z12 - 3z 22 ) = 2 ........(1)

z 2 (3z12 - z 22 ) = 11 ........(2)
Multiplying (2) by i and adding it to (1), we get
z13 - 3z 22 z1 + i(3z12 z 2 - z 23 ) = 2 + 11i

Þ (z1 + iz 2 )3 = 2 + 11i ........(3)


Multiplying (2) by i and subtracting it from (1), we get
z13 - 3z 23 z1 - i(3z12 z 2 - z 23 ) = 2 - 11i

MATHEMATICS /AQ # 12 E-157


TM TARGET: JEE (MAIN + ADVANCED) 2020
ENTHUSIAST & LEADER
Path to success KOTA (RAJASTHAN)
COURSE
Þ (z1 - iz 2 )3 = 2 - 11i ........(4)
Multiplying (3) and (4), we get
(z12 + z 22 )3 = 22 + 112 = 125

Þ z12 + z 22 = 5

MATCH THE COLUMN :


25. Ans. [A–q,r ; B–p,s ; C–q,s ; D–p,r]
1 ± -3i 1 + i -3 1 + i -3
(A) z= = or
2 2 2
p p
amp z = or amp z = – Þ Q,R
3 3

-1 ± 3i -1 + i 3 -1 - i 3
(B) z= = or
2 2 2
2p 2p
amp z = or - Þ P,S
3 3
(C) 2z2 = -1 - i 3
-1 - i 3 æ 2p ö æ 2p ö
Þ z2 = = cos ç - ÷ + i sin ç - ÷
2 è 3 ø è 3 ø
æ 2mp - (2p / 3) ö æ 2mp - (2p / 3) ö
z = cos ç ÷ + i sin ç ÷
è 2 ø è 2 ø
æ pö æ pö
m = 0, z = cos ç - ÷ + i sin ç - ÷
è 3ø è 3ø
æ 2p ö æ 2p ö
m = 1, z = cos ç ÷ + i sin ç ÷
è 3 ø è 3 ø
p 2p
Þ amp z = – or Þ Q,S
3 3
(D) 2z2 + 1 – i 3=0
-1 + i 3 æ 2p ö æ 2p ö
z2 = = cos ç ÷ + i sin ç ÷
2 è 3 ø è 3 ø
æ 2mp + (2p / 3) ö æ pö
z = cos ç ÷ + i sin ç ÷
è 2 ø è3ø
æ pö æ 2mp + (2p / 3) ö
m = 0, z = cos ç ÷ + i sin ç ÷
è 3ø è 2 ø
æ 4p ö æ 4p ö æ 2p ö æ 2p ö
m = 1, cos ç ÷ + i sin ç ÷ or cos ç - ÷ + i sin ç - ÷ Þ P,R]
è 3 ø è 3 ø è 3 ø è 3 ø

MATHEMATICS /AQ # 12 E-158


TM TM TARGET:JEE
TARGET: JEE(Main
(MAIN ++Advanced)
ADVANCED) 2020
2020
ENTHUSIAST&
ENTHUSIAST & LEADER
LEADER
Path to success KOTA (RAJASTHAN)
COURSE
Path to success KOTA (RAJASTHAN ) COURSE

ADVANCED QUIZ # 13 (SOLUTION) PERMUATION, COMBINATION & PROBABILITY PART -1 MATHS

SINGLE CORRECT CHOICE TYPE

1 1
1. Ans. (B) P(E) = ; P(Fk) = nCk. 2n
2

n -1
1 n -1 æ1ö
P(E Ç Fk) = × C k -1 ç ÷
2 è 2ø
\ P(E Ç Fk) = P(E). P(Fk)

n -1 1 1 n 1
C k -1 × n
= × Ck × n
2 2 2
2 × n -1C k -1 = n C k Þ n = 2k Þ (B)]

3R
2. Ans. (D) In the 1st case Urn
n white

3
C2 + n C2 1
P(they match) n+3
C2
= ;
2 ( n ³ 2)

6 + n(n - 1) 1
= Þ 2(n2 – n + 6) = n2 + 5n + 6
(n + 3)(n + 2) 2
Þ n2 – 7n + 6 = 0 Þ n = 1 or 6 .....(1)
In the 2nd case
3 3 n n 5
× + × =
n +3 n +3 n +3 n +3 8
solving n2 – 10n + 9 = 0
n = 9 or 1 ......(2)
from (1) and (2) Þ No value of n

60 5
3. Ans. (A) n(S) = 63 ; n(A) = 5 . 4 . 3 Þ =
216 18

4. Ans. (A) x + y + z = 15: x £ 8; y £ 8, z £ 8

Total solution = 17 C 2 = 136


suppose x or y or z is given 9 oranges
x+ y+ z = 6
3
C1 . 8 C 2 = 84 invalid

effective ways = 136 – 84 = 52 Ans. ]

A BCD
5. Ans. (A) 6
F1 F2 F3
;

3! × 4C3 × 3! = 6 × 4 × 6 = 36 × 4 = 144 ]

MATHEMATICS /AQ # 13 E-159


TM TARGET: JEE (MAIN + ADVANCED) 2020
ENTHUSIAST & LEADER
Path to success KOTA (RAJASTHAN)
COURSE
6. Ans. (C)

7. Ans. (C) Let S1 and S2 refuse to be together and S3 and S4 wants to be together only
total ways when S3 and S4 are selected
= (8C2 + 2C1 . 8C1) = 44
total ways when S3 and S4 are not selected
= (8C4 + 2C1 . 8C3) = 182
Thus total ways = 44 + 182 = 226 Ans.]
8. Ans. (B) Let the arrangement be x1 x2 x3 x4 x5 x6 x7 x8. Clearly 5 should occupy the position x4 or x5 . Thus
required number = 2(7!) Ans.]
9. Ans. (B) If all 26 people shake hands, there would be 26C2 handshakes. Of these 13C2 would take place between women and
13 between spouses.
Therefore, there were 26C2 – 13C2 – 13
= 13 . 25 – 13 . 6 – 13 = 234 handshakes.]
10. Ans. (D) A A H H I R S T
7!
A® = 2520
2!
7!
H® = 2520 5040
2!
I A A H H ® 3! = 6 5046
IAAH R H = 2 5048
I AA H R S H T = 1 5049
and I A A H R S T H ® 5050
11. Ans. (D) Out of 5 girls, he has to invite exactly three. This can be done in 5C3 ways .Out of the 4 boys, he may invite either
one or two or three or four or even none of them. This may be done in (2) 4 ways . Thus the total number of wasy is 5C3
× (2)4 = 10 × 16 = 160]

(10000)!
12. Ans. (B) P = 1 .3 . 5 . ....... 9999 =
2 × 4 × 6 × ...... ×10000

(10000)!
= 25000 (5000)! Ans.
Paragraph for question No. 13 to 15
Sol.13. (D) increasing order + decreasing order
9 10
= 378
C5 C5
1442443 1442443
zero can not be included zero can be taken

Sol.14. (A) zero excluded


5!
9
C2 · 2 · (11222 or 22111) = 36 · 2 · 10 = 720
3!2!

é 5! 4! ù 9 é 5! 4!ù
zero included
9
C1 ê - ú + C1 ê 3!·2! - 3! ú
ë 3!·2! 2!·2!û ë û
= 9[10 – 6] + 9[10 – 4] = 36 + 54 = 90
Total = 810
Sol.15. (A)
N = a1 a 2 a3 a4 a5 a6 a7 a8 a9 a10
As sum of of digits = 1 + 2 + 3 + 4 + 5 + 6 + 7 + 8 + 9 + 0 = 45, which is divisible by 3, so no such number exist.]

MATHEMATICS /AQ # 13 E-160


TM TARGET: JEE (MAIN + ADVANCED) 2020
ENTHUSIAST & LEADER
Path to success KOTA (RAJASTHAN)
COURSE
Paragraph for question Nos. 16 to 18
16.Ans.(B) Digits in the ascending order.
Number of numbers beginning with 12 is 3
Number of numbers beginning with 13 is 2
Number of numbers beginning with 14 is 1
Number of numbers beginning with 23 is 2
Number of numbers beginning with 24 is 1
Number of numbers beginning with 3 is 1
Total = 10

There are 10 three digit numbers in which the digits are


in increasing order.
Digits in the descending order
Number of numbers beginning with 54 is 4
Number of numbers beginning with 53 is 3
Number of numbers beginning with 52 is 2
Number of numbers beginning with 51 is 1
Number of numbers beginning with 43 is 3
Number of numbers beginning with 42 is 2
Number of numbers beginning with 41 is 1
Number of numbers beginning with 32 is 2
Number of numbers beginning with 31 is 1
Number of numbers beginning with 2 is 1
Total number 3 digits numbers in which the digits are in descending order = 20
Numbers of 3 digit numbers in which the digits are either
in ascending order or in descending order = 10 + 20 = 30
17. Ans. (A) Consider the subsets A = {1, 2, 3, 4, 5} and B = {0, 1, 2, 4, 5}
All five digit numbers formed using all the digits of A or all
the digits of B without repetition are divisible by 3.
Number of 5 digit numbers that can be formed using the digits
in A =5! = 120
Number of 5 digit numbers that can be formed using all the digits
in B = 4.4.3.2.1
= 96
Total number of 5 digit number divisible by 3
= 120 + 96 = 216
18. Ans. (D) Number of 6 digit numbers = 5 × 5 × 4 × 3 × 2 × 1 = 600
INTEGER ANSWER TYPE
19.Ans. [2750] Unity place can be filled in 5 ways (1,3,5,7,9)
H T U
tenth place in 10 way and hundredth’s place only in 1 way

hence total number = 5 . 10 . 1 = 50


10 × 100(1 + 3 + 5 + 7 + 9) +
sum = 1444 424444 3
sum of hundredth 's place

10 ×1(1 + 3 + 5 + 7 + 9) +
144424443
sum of unit ' s place

5 ×10(1 + 2 + 3 + 4 + 5 + 6 + 7 + 8 + 9)
144444424444443
sum of ten 's place

= 10 100 25 + 10 . 25 + 5 . 10 . 45
. .

= 10 . 25 [100 + 1 + 9] = 10 . 25 . 110 = 27500


Þ K = 2750 Ans.

MATHEMATICS /AQ # 13 E-161


TM TARGET: JEE (MAIN + ADVANCED) 2020
ENTHUSIAST & LEADER
Path to success KOTA (RAJASTHAN)
COURSE
20.Ans. [0483] B : Distribute 20 alike objects in 3 beggars namely x, 2y, 3z

{0 Æ
000 {Æ = 22 C 2 22 × 21
= = 11
1. 21 = 231 Ans.]
20 2 2

A : F1 + F2 + F3 + F4 + F5 + F6 = 5
Problem is similar to distiribution of 5 alike object in 6 beggars.
\ 5 + 6–1C6–1 = 10C5 = 252

\ A + B = 252 + 231 = 483

21. Ans. [0011] at most two children got mobile of the other children
Þ exactly 3R and 2W + all 5 got their own mobile

= 5C3 . 1 + 1 = 11 Ans.

22. Ans. [0008] å Cr = Cr + Cr + ..... + Cr


k r r +1 n

k=r

= 1 + r +1C1 + r + 2C2 + r + 3C3 + .....+nCn – r


r +1
C 0 + r +1C1 + r + 2 C 2 + ..... + n C n - r
E5555555
r+2
F
C1

C2 and so on finally n + 1Cn – r


r+3

now, n + 1Cn – r = n + 1Cr + 1

n
ü
\ f (n) = å n +1 Cr +1 = n +1C1 + n +1C 2 + n +1C3 + ..... + n +1C n +1 ï
r=0 ý
= n +1C0 + n +1C1 + n +1C2 + ..... + n +1Cn +1 - 1ïþ

f(n) = (2n + 1) – 1
f(9) = 210 – 1 = 1023 = 3 . 11 . 31
hence number of divisors are (1 + 1) (1 + 1) (1 + 1) = 8 Ans.
23. Ans. [0816] We have cosec–1 cosec 2> x2 – 3x
Þ x2 – 3x – (p – 2) < 0

3 - 1 + 4p 3 + 1 + 4p
Þ <x<
2 2

\ Positive integral solution of above inequality are 1,2,3


Now 2010 = 2 × 3 × 5 × 67, total number of divisors of 2010 = 16.
So A contains 3 elements and B contains 16 elements. Number of mapping are as follows:

f(1) < f(2) < f(3) Þ 16C3 = 560

f(1) = f(2) < f(3) Þ 16C2 = 120

f(1) < f(2) = f(3) Þ 16C2 = 120

f(1) = f(2) = f(3) Þ 16C1 = 16

Hence total mappings = 816 Ans.

MATHEMATICS /AQ # 13 E-162


TM TARGET: JEE (MAIN + ADVANCED) 2020
ENTHUSIAST & LEADER
Path to success KOTA (RAJASTHAN)
COURSE
24.Ans. [0084] Let a2 = x2 + 6, a3 = x3 + 2, a4 = x4 + 1
So, a1 + a2 + a3 + a4 = 15 Þ a1 + x2 + x3 + x4 = 6, where a1, x2, x3, x4³ 0
So, using begger’s method, we get number of solutions = 9C3 = 84
25.Ans. [0035] Number of 0’s and 1’s in L.H.S. = number of 0’s and 1’s in R.H.S.
(a) If L.H.S. contains all 1’s min. a1 = 1 (given)
In the case a2 = a4 = a6 = a8 = 1
Total ways in the case = 1
(b) In L.H.S. contains three 1’s and R.H.S. also contains three 1’s
(3C2)·(4C3) = 12
(C) L.H.S. have two 1’s and R.H.S. also has two 1’s
(3C1) · (4C2) = 18
(D) L.H.S. has one 1’s and R.H.S. has one 1’s
(1)·(4C1) = 4
Total = 1 + 12 + 18 + 4 = 35
26.Ans. [0840] Total number of arrangements = 7!
Number of arrangements of A, B and C amongst themselves = 3!
7!
\ Number of arrangements when A, B and C follows in order = = 840
3!
MATCH THE COLUMN :
27. Ans. [A–R ; B–S ; C–Q]
(A) Six digits numbers with sum of digits 10
0,1,2,3 occurs at least once remainimg two digits must give the sum 10. Hence possible cases two digits if their sum is 4,
can be
04,13,22
six digit numbers with
6!
(i) 001234 = - 5! = 240
2!

6! 5!
(ii) 012313 = - = 180 – 30 = 150
2!× 2! 2!× 2!

6! 5!
(iii) 012322 = - = 120 – 20 = 100
3! 3!
Total = 490 Ans. Þ (R)

A B 4
(B) (3 – 1) (23 – 1) = 80 . 7 = 560
Q1 < Q1
Q2 < Q2
Q3 < Q3
Q4 <

Ans. Þ (S)

MATHEMATICS /AQ # 13 E-163


TM TARGET: JEE (MAIN + ADVANCED) 2020
ENTHUSIAST & LEADER
Path to success KOTA (RAJASTHAN)
COURSE
(C) LCM of a and b is 22 . 54 . 114
let a = 2r . 5s . 11t and b = 2r1 × 5s1 ×11t1

s s1
4 0
r r1 1
2 0 2
1 3
2
4
1 2 ; |||ly t and t1 can be taken in 9 ways.
3 4
0 2
2 4
5 ways 1 4
0 4
9 ways

Total ways = 5.9.9 = 405 Ans. Þ (Q)]

MATHEMATICS /AQ # 13 E-164


TM TM TARGET:JEE
TARGET: JEE(Main
(MAIN ++Advanced)
ADVANCED) 2020
2020
ENTHUSIAST&
ENTHUSIAST & LEADER
LEADER
Path to success KOTA (RAJASTHAN)
COURSE
Path to success KOTA (RAJASTHAN ) COURSE

ADVANCED QUIZ # 14 (SOLUTION) PERMUATION,COMBINATION & PROBABILITY PART-2 MATHS

SINGLE CORRECT CHOICE TYPE


1. Ans. (C) Required number of ways

7! 120 ´ 6 ´ 7
= ´ 3! = = 630.
3!2!2!2! 8
2. Ans. (D) The middle digit can be fixed in four ways (2,3,5, or 7). while unit digit can be fixed in four ways (0,3,6 or 9). Lastly
hundredth place can be fixed in 9 ways (1,2,.......9). Thus required number of numbers is 9 × 4 × 4 = 144.

6!
3. Ans. (B) Number of numbers that begin with 3 = = 180
2!2!

Number of numbers that begin with 2 = 6! = 120


3!

6!
Number of numbers that begin with 4 = = 60
3!2!

Total = 360
4. Ans. (D) Let a = 2p + 1, b = 2q +1, c = 2r + 1, d = 2s + 1, where p,q,r,s are non negative integers.

\ 2p + 1 + 2q + 1 + 2r + 1 + 2s + 1 = 40 or p + q + r + s = 18.

\ the required number of solutions = 18 + 4 – 1C4 – 1 = 1330

5. Ans. (B) n ( S) = 4 ´ 4 ´ 4 = 64

n (A) = 211 or 312 or 321 or 413 or 431 or 422

6 3 3
\ P (E) = = =
64 32 3 + 29
odds in favour 3 : 29 Ans. ]

6. Ans. (B)

The required number of numbers = 4P4 × 4P4.

7. Ans. (C) A C E R (alphabetical order)


A - - - = 3!
C A E R =1
C A R E =1
\ Rank of word CARE is8

8. Ans. (A) p = 10 ; q = 80 ; r = 32 ]

9. Ans. (B)

MATHEMATICS /AQ # 14 E-165


TM TARGET: JEE (MAIN + ADVANCED) 2020
ENTHUSIAST & LEADER
Path to success KOTA (RAJASTHAN)
COURSE
1 5
10. Ans. (C) Probability of choosing a science subject from first group = ´
3 8

2 3
Probability of choosing a science subject from second group = ´
3 8

1 5 2 3 11
\ Required Probability = ´ + ´ =
3 8 3 8 24

11. Ans. (C) Probability of A winning is


2 4
p æ q ö p æ q ö p
P(A) = +ç ÷ . +ç ÷ .
p+q èp+q ø p+q è p+qø p+q

p
p+q
+.... 2
æ q ö z
1- ç ÷
èp+qø

2
æ q ö
which is an infinite G.P. with common ratio ç ÷
è p+qø

p(p + q) 3
\ P(A) = =
(p + q) 2 - q 2 4
( P(A) = 3P(B) )

Þ 4p(p + q) = 3{(p + q) 2 - q 2 }

Þ 4p(p + q) = 3(p + 2q) · p

Þ 4p + 4q = 3p + 6q

Þ p = 2q

\p:q=2:1
12. Ans. (C) x = Coeff of t10 (t + t2 + ......)4 =Coeff of t6 in (1 – t)–4 = (6+4 – 1)C6 = 9C3 = y
13. Ans. (A) P(E1) = Probability that the first digit is 3
2
= (Q Either 322 or 323 should be drawn)
4

1
=
2

2 1 2 1
P(E2) = = , P(E 3 ) = =
4 2 4 2

1
\P (E1 Ç E 2 ) = Probability that the first two digits are 3 = = P(E1) . P(E2)
4

MATHEMATICS /AQ # 14 E-166


TM TARGET: JEE (MAIN + ADVANCED) 2020
ENTHUSIAST & LEADER
Path to success KOTA (RAJASTHAN)
COURSE
\E1 and E2 are independent Similarly, E2 and E3 are also independent.
\E1,E2,E3 are not mutually exclusive.

1
Also P (E1 Ç E 2 Ç E3 ) =
4

1
¹ P(E1 ).P(E 2 ).P(E 3 ) =
8

14. Ans. (D) Consider the identity


a n - b n = ( a - b ) ( a n -1 + a n - 2 b + ..... + ab n - 2 + b n -1 )

Taking a = 3, b = – 1 we have

3n - ( -1) = 4 é3n -1 - 3n - 2 + ..... + 3 ( -1) + ( -1) ù


n n-2 n -1
ë û
= 4k (k an integer)
Now,

3n + 1 = 3n - ( -1) + 1 + ( -1)
n n

= 4k + 1 + ( -1)
n

3n + 1 leaves the remainder 2 when divided by 4 if n is


even. When n is odd, 3n + 1 is divisible by 4
\ Probability of getting a remainder 1 to 3 = 0
II method:
For all n, 3n is odd Þ 3n + 1 is even
\ When divided by 4, 3n + 1 leaves the reminder 2 or 0.
\ Required probability = 0
\ choice is (D)
15.Ans. (C) Simple

Paragraph for question Nos. 16to 18

16. (D) No. of ways = 4 C1 + 4 C 2 + 4 C 3 + 4 C 4 = 24 - 1 = 15

17. (A) Suppose he attempts question 2 of section A and question 4 and 7 of section B. There are 3 cases that arise
Section – A Section – B
Question – 2 Question – 4 Question – 7
case 1 û ü ü
case 2 ü û ü
case 3 ü ü û

3 1 1 1 14 1 31
Required probability = . . + 2. . . =
4 15 15 4 15 15 900

18. (D) 10 marks can be obtained only if he attempts 1 question of section A and 3 question of section B correctly

3
1 1 1 1 1æ 1 ö
Þ probability = × × × = ç ÷
4 15 15 15 4 è 15 ø

MATHEMATICS /AQ # 14 E-167


TM TARGET: JEE (MAIN + ADVANCED) 2020
ENTHUSIAST & LEADER
Path to success KOTA (RAJASTHAN)
COURSE
MULTIPLE CORRECT CHOICE TYPE
19.Ans. (A,C,D) Answer is 3n
(A) (3n – 1) ; (B) 3n ;
(C) Total – when all 3 digits are even = 6n – 3n ;
(D) n C3 × 3!
20.Ans. (B,C) 210 – 1 ; (A) 210 – 2 ; (B) 210 – 1; (C) 210 – 1 ; (D) 10

(n - 1)! n -1 n -1 n -1
21.Ans. (B, D) (A) p! ; (B) Cp ; (C) using Beggar method, Cp -1 ; (D) Cp

22.Ans. (A, B, D) (C) 1st arm can move in 6 ways and so on....
\ 65 – 1
n!
(B) {n(n – 1) (n – 2) – (n – r + 1) (n – r)!} / (n – r) ! = (n - r)! = Cr × r!]
n

INTEGER ANSWER TYPE

23.Ans. [0084] 4 lines intersect each other in 4 C 2 = 6 points.


6 circles intersect each other in 2 ´ 6 C 2 = 30 points.
Further one line and one circle intersect in 2 points. So 4 lines and
6 circles will intersect 6C1 4C1 × 2 = 48
\ maximum number of points = 6 + 30 + 48 = 84.
24.Ans. [1560] There can be two types of numbers
(i) Any one of the digits 1, 2, 3, 4 appears thrice and the remaining digits only once, i.e. of the type 1, 2, 3, 4, 4, 4, etc.
Number of ways of selection of digit which appears thrice is 4C1.
Then the number of numbers of this type is (6!/3!) × 4C1 = 480.
(ii) Any two of the digits 1, 2, 3, 4 appears twice and the remaining two only once, i.e., of the type 1, 2, 3, 3, 4, 4, etc. The
number of ways of selection of two digits which appear twice is 4C2. Then the number of numbers of this type is
[6!/(2!2! × 4C2]. Therefore, the required number of numbers is 480 + 1080 = 1560.
25.Ans. [0616] The total number of words is 6! = 720. Let us write the letters of word ZENITH alphabetically, i.e., EHINTZ.
For ZENITH word Word starting Number of
start with with words
Z E 5!
H 5!
I 5!
N 5!
T 5!
ZEN ZEH 3!
ZEI 3!
ZENI ZENH 2!
ZENIT ZEHIH 1
Total number
of words before 615
ZENITH
Hence, there are 615 words before ZENITH, so the rank of ZENITH is 616

MATHEMATICS /AQ # 14 E-168


TM TARGET: JEE (MAIN + ADVANCED) 2020
ENTHUSIAST & LEADER
Path to success KOTA (RAJASTHAN)
COURSE
26. Ans. [0226] Let S1 and S2 refuse to the together and S3 and S4 want to be together only. The total number of ways when S3
and S4 are selected is (8C2 × 2C1 × 8C1) = 44. The total ways when S3 and S4 are not selected is (8C4 + 2C1 × 8C3) = 182. Thus,
the total number of ways is 44 + 182 = 226.

27. Ans. [0030] K’s = 2; U’s = 2; T’s = 2

6!
Total = = 90
2!× 2!× 2!

n(E3) = K K U U T T = 3! = 6

n(E2) = 3 [n(A Ç B) – n(A Ç B Ç C)]

é 4! ù
= 3 ê - 6ú =18
ë 2! û

n(E1) = 3 [n(A) – { n(A Ç B) – n(B Ç C)}


– n(A Ç B Ç C)]

é 5! ù
= 3ê - (12 × 2 - 6 ) ú = 3 [30 – 18] = 36
ë 2!× 2! û

Hence required number of ways = 90 – [6 + 18 + 36] = 90 – 60 = 30 Ans.

28. Ans. [0054] Number of even divisors is equal to number of ways in which one or more ‘2’, zero or more ‘3’, zero or more ‘5’
and zero or more ‘7’ can be selected, and is given by (3) (2 + 1) (2 + 1) (1 + 1) = 54

MATCH THE COLUMN :


29.Ans. [A - r ; B - s ; C - q ; D - p]

MATHEMATICS /AQ # 14 E-169


TM TM TARGET:JEE
TARGET: JEE(Main
(MAIN ++Advanced)
ADVANCED) 2020
2020
ENTHUSIAST&
ENTHUSIAST & LEADER
LEADER
Path to success KOTA (RAJASTHAN)
COURSE
Path to success KOTA (RAJASTHAN ) COURSE

ADVANCED QUIZ # 15 (SOLUTION) DETERMINANT & MATRICES MATHEMATICS

SINGLE CORRECT CHOICE TYPE

é 7 -10 17 ù
1. Ans. (B) 3A + 4B = ê 0
T

ë 6 31úû .......(1)

é -1 18 ù
2B - 3A = êê 4 -6 úú
T
.......(2)
êë -5 -7 úû
Taking transpose of (1) so

é 7 0ù
(3A + 4BT ) T = êê -10 6 úú
êë 17 31úû

é 7 0ù
3A T + 4B = êê -10 6 úú
.......(3)
êë 17 31úû
(2) + (3)

é 6 18 ù é 1 3ù
6B = êê -6 0 úú B = êê -1 0 úú
;
êë12 24 úû êë 2 4úû

2. Ans. (C) (I + A)10 = I + 10C1 A + 10C2A2 + .....+10C10A10 = I + 10C1A + 10C2A + ......+ 10C10A = I + (210–1) A
{ Q An = A "n ³ 2 in case of idempotent matrix}

3. Ans. (D) x = ( agABC2 D )


3

3
x = ag.ABC2 D

order of ABC2D is 2 × 2

x = ( a 2 g 2 ) ABC2 D .
3 3

é r r - 1ù
4. Ans. (C) M r = ê
ër - 1 r úû

M r = r 2 - ( r - 1) = 2r - 1
2

n n
Þ å M r = å ( 2r - 1) = n 2
n =1 r =1

5. Ans. (A) Applying C1 Þ C1 + C3 – 2 cos (dx) C2 we get

1 + a 2 - 2a cos(dx) a a2
det = 0 cos px cos(p + d)x
0 sin px sin(p + d)x

= (1 + a2 – 2a cos dx) sin (px + dx – px)


= (1 + a2 – 2a cos dx) sin dx

MATHEMATICS /AQ # 15 E-170


TM TARGET: JEE (MAIN + ADVANCED) 2020
ENTHUSIAST & LEADER
Path to success KOTA (RAJASTHAN)
COURSE
6. Ans. (D) Whe have,
é3 -4ù é3 -4 ù é 5 -8 ù
X 2 = X·X = ê úê ú=ê ú
ë1 -1û ë1 -1û ë 2 -3û

é 5 -8ù
For n = 2, matrices in OPTION (A), (B) and (C) do not match with ê 2 -3ú
ë û
7. Ans. (B) Since, it is given that
1 1 1
a b c = (a - b)(b - c)(c - a)(a + b + c) ....(1)
3 3
a b c3
Now,
1 1 1
(x - a) 2 (x - b) 2 (x - c)2 =0
(x - b)(x - c) (x - c)(x - a) (x - a)(x - b)
Applying
C1 ® C1 (x - a);C 2 ® C2 (x - b);C3 ® C3 (x - c)
1
If (x - a)(x - b)(x - c)

(x - a) (x - b) (x - c)
(x - a)3 (x - b)3 (x - c)3 =0
(x - a)(x - b)(x - c) (x - a)(x - b)(x - c) (x - a)(x - b)(x - c)

(x - a) (x - b) (x - c)
Þ (x - a)3 (x - b)3 (x - c)3 = 0
1 1 1

1 1 1
Þ (x - a) (x - b) (x - c) = 0
(x - a) (x - b)3 (x - c)3
{(x – c) – (x – a)}{(x – a + x – b + c – c)} = 0 {using (1)}
Þ (b – a)(c – a)(a – c) {3x – (a + b+ c)} = 0
1
\ x = (a + b + c) {Q a ¹ b ¹ c}
3
8. Ans. (A) Applying C1 ® C1 + C2, we get

2 cos 2 x sin 2x
2 1 + cos 2 x sin 2x
1 cos 2 x 1 + sin 2x
Applying R2 ® R2 – R1 and R3 ® R3 – R1, we get

2 cos 2 x sin 2x
0 1 0 = 2 + sin 2x
-1 0 1
Since the maximum value of sin2x is 1, and minimum value of sin 2x is (–1)
There fore a = 3 and b = 1

MATHEMATICS /AQ # 15 E-171


TM TARGET: JEE (MAIN + ADVANCED) 2020
ENTHUSIAST & LEADER
Path to success KOTA (RAJASTHAN)
COURSE
9.Ans. (C) Applying C1 ® C1 + C2 + C3 , we have

1 bg ga
D = (ab + bg + ga) 1 ga ab
1 ab bg
Also, since a, b, g Î px3 + qx2 + r = 0
2 0
\ S2 = ab + bg + ga = (-1) p = 0

Hence, D = 0

é a1 a2 a3 ù
10. Ans.(D) êê b1 b2 b 3 úú is a singular matrix
êë c1 c2 c3 úû

é a1 a2 a3 ù
ê b 3 úú = 0
Þ ê b1 b2
êë c1 c2 c3 úû
Þ System of equation has infinite solutions
so above system of plane having only one line of intersection.

PASSAGE TYPE QUESTION

æ 1 ö
11. (B) sin–1 sin ç ÷ = sin -1 sin 6 = 6 - 2p
è abc ø
[6 – 2 p ] = – 1

é 1ù
1 é1 1ù é1 2 ù
12. (A) A = ê 2ú Þ A = ê
2
ú Þ (A 2 ) 2 = ê ú
ê ú ë0 1û ë0 1 û
ë0 1 û

é1 1 + 2ù
Þ (A2)3 = ê 0 1 úû
ë

é 1 5ù é -a -15b ù
Þ (A2)5 = ê 0 1ú = êf (0) -3b ú
ë û ë û
13. (C)

Obviously if g(x) = k has no solution


Þ k Î (0,1]
Þp+q=1
INTEGER ANSWER TYPE

b-c c-a a -b
14. Ans. [0000] Let D = b '- c' c '- a ' a '- b '
b"- c" c"- a " c"- b"

MATHEMATICS /AQ # 15 E-172


TM TARGET: JEE (MAIN + ADVANCED) 2020
ENTHUSIAST & LEADER
Path to success KOTA (RAJASTHAN)
COURSE
Applying C1 ® C1 + C2 + C3

0 c-a a -b
D= 0 c '- a ' a '- b ' = 0
0 c"- a " a "- b"

\m=0
15. Ans. [0006] Applying C1 ® C1 + C 2
2 cos 2 x 4sin 2x
f (x) = 2 1 + cos 2 x 4sin 2x
1 cos 2 x 1 + 4sin 2x

Applying R 2 ® R 2 - R 1 , we have

2 cos 2 x 4sin 2x
f (x) = 0 1 0
1 cos 2 x 1 + 4sin 2x

Þ f (x) = 2 + 4sin 2x
The value of f(x) is maximum when
sin2x = 1
\ maximum value of f(x) = 6

a a+b a +b+c
2a 3a + 2b 4a + 3b + 2c = 64
16. Ans.[0004] Since,
3a 6a + 3b 10a + 6b + 3c

Applying R 2 ® R 2 - 2R1 ; R 3 ® R 3 - 3R1

a a +b a+b+c
Þ 0 a 2a + b = 64
0 3a 7a + 3b
Expanding along C1, we have
a(7a2 + 3ab – 6a2 – 3ab) = 64
Þ a(a 2 ) = 64
\ a=4
17. Ans. [0004] Consider the triangle with vertices B(x1, y1), C(x2, y2) and A(x3, z3), and AB = c, BC = a and AC = b. Then area of
triangle is
x1 y1 1
1
x2 y 2 1 = s(s - a)(s - b)(s - c)
2
x3 y3 1
Squaring and simplifying, we get
2
x1 y1 1
1 æ a + b + c öæ b + c - c öæ c + a - b öæ a + b - c ö
x2 y2 1 =ç ÷ç ÷ç ÷ç ÷
4 è 2 øè 2 øè 2 øè 2 ø
x3 y3 1

2
x1 y1 1
Þ 4 x2 y2 1
x3 y3 1

= (a + b + c)(b + c - a)(c + a - b)(a + b - c)


MATHEMATICS /AQ # 15 E-173
TM TARGET: JEE (MAIN + ADVANCED) 2020
ENTHUSIAST & LEADER
Path to success KOTA (RAJASTHAN)
COURSE

x3 + 1 x 2y x2z
18. Ans. [0003] xy y + 1 y z = 11
2 3 2

xz 2 yz 2 z3 + 1

Multiplying R1 by x, R2 by y and R3 by z, we get

x4 + x x3y x 3z
1
xy3 y4 + y y3 z = 11
xyz
xz3 yz3 z4 + z

Taking x, y, z common from C1, C2, C3, respectively, we get

x 3 + 1 x3 x3
y3 y3 + 1 y3 = 11
z 3
z 3
z +1
3

Using R1 ® R1 + R2 + R3, we have

1 1 1
(x + y + z + 1) y3
3 3 3
y +1
3
y3 = 11
z3 z3 z3 + 1

Using C2 ® C2 – C1 and C3 ® C3 – C1, we get

1 0 0
(x + y + z + 1) y3
3 3 3
1 0 = 11
z3 0 1

Hence,
x3 + y3 + z3 = 10
Therefore, the ordered triples are (2, 1, 1), (1, 2, 1), (1, 1, 2)
19. Ans. [0001] Since the product matrix is 3 × 3 matrix and the pre-multiplier of A is a 3 × 2 matrix, therefore, A is 2 × 3 matrix. Let.
él m nù
A =ê ú . Then the given equation becomes
ëx y z û

é 2 -1ù é -1 -8 -10 ù
ê 1 0 ú é l m nù = ê 1 -2 -5 ú
ê ú êx y z ú ê ú
êë -3 4 úû ë û êë 9 22 15 úû

é 2l - x 2m - y 2n - z ù
Þ êê l m n ú
ú
êë -3l + 4x -3m + 4y -3n + 4z úû

é -1 -8 -10 ù
= êê 1 -2 -5 úú
êë 9 22 15 úû
Þ 2l – x = –1, 2m – y = –8, 2n – z = –10, l = 1, m = –2, n = –5
Þ x = 3, y = 4, z = 0, l = 1, m = –2, n = –5
é l m n ù é1 -2 -5 ù
ÞA=ê ú=ê ú
ë x y z û ë3 4 0 û

MATHEMATICS /AQ # 15 E-174


TM TARGET: JEE (MAIN + ADVANCED) 2020
ENTHUSIAST & LEADER
Path to success KOTA (RAJASTHAN)
COURSE
20. Ans. [0004] We know that in a square matrix of order n,
|adj A| = |A|n – 1
2
Þ | adj(adj.A) | = | AdjA |n -1 =| A |(n -1)
Þ n2 – 2n – 8 = 0
Þ n = 4 as n = –2 is not possible
MATCH THE COLUMN :
21. Ans.(A-p, q, t; B-s; C-p, r; D-r)
Here 24 matrices are possible.
Values of determinants corresponding to these matrices are as follows :
1 0 1 0
= 2 (4 matrices), = 4 (4 matrices),
4 2 2 4

2 0
= 8 (4 matrices)
1 4
And 12 more matrices are there, values of whose determinants are –2, – 4, – 8.
(A) Possible non-negative values of det. (A) are 2, 4, 8.
(B) Sum of these 24 determinants is 0.
(C) Mod. (det(A)) is least \ A = ± 2

( n -1)3
Þ adj(adj(adj ( A )) = A = ±2
(D) Least value of det. (A) is – 8

-1 1 16
Now, 4 A = 16 A = -8 = -2]

22. Ans.[A–q ;B–s ; C– p; D–r]


R = PT QK P

= P T ( PAP T ) P
k

= P T PAP
1444
T
PAP T
..........PAP
424444 3P
T

k times

= A as PP = I as P is orthogonal
k T

é1 + 2K -4K ù
= Ak = ê
ë K 1 - 2K úû

|||1y T = PT SK P = BK

é b ( a K - 1) ù
ê K ú
B = êa
K
a - 1 ú]
êë 0 1 úû

MATHEMATICS /AQ # 15 E-175


TM TM TARGET:
TARGET:JEE
JEE(Main
(MAIN+ +Advanced)
ADVANCED)2020
2020
Path to success KOTA (RAJASTHAN) ENTHUSIAST
ENTHUSIAST&&LEADER
LEADER
Path to success KOTA (RAJASTHAN ) COURSE
COURSE

ADVANCED QUIZ # 16(SOLUTION) VECTOR & 3D MATHEMATICS


SINGLE CORRECT CHOICE TYPE
1.Ans. ( ˆ ´ (2bˆ - c)
(A) (2aˆ - b) )
ˆ Þ (4aˆ - bˆ - 2aˆ ´ cˆ + bˆ ´ c)
ˆ × (2cˆ - a) ˆ × (2cˆ - a) ˆ ˆ ˆ ù - é bca
ˆ Þ 8 éë abc û ë
ˆ ˆ ˆ ù = 7 éabc
û
ˆˆ ˆù
ë û = 7 Ans.

x -1 y - 0 z + 2
2.Ans. (C) L is = = hence L is along the vector – ˆi + 3jˆ
-1 3 0

x -1 y - 0
only in (C) the line is = ; z = 5 and (B) is coincident line, which is || to the vector – ˆi + 3jˆ ]
-1 3
r r r r r r r r r r
3.Ans. (D) p ´ ( p ´ q ) = ( p × q ) p - ( p × p ) q = -4q
r r r r r r r r r r r r
\ V = -4p ´ ( p ´ q ) = – 4 éë( p × q ) p - ( p × p ) q ùû = + (4) (4) q = 16q Ans.]
4. Ans. (B) For L1 : (3 – t) – (2 + t) + 2 (5t) = 9
8t = 8 Þ t=1
hence A(2,3,5)
for L2 : (1 + 2t) + 2(4t) – (2–3t) + 1 = 0
13 t = 0 Þ t = 0
hence B(1,0,2)
x y -1 z
for L3 : = = =t
1 1 1/ 2

3t 9t
\ 4(t) – (t + 1) +
2
=8 Þ 2 =9 Þ t =2

11 + 16 - 19 = 1 19 + 11 - 16 14
hence C(2,3,1) now cos C = Þ cosB = =
2 × 11 × 4 11 2 × 19 × 11 2 × 19 × 11

19 + 16 - 11 24 3
cos A = = = hence A,B,C is acute triangle. Ans.]
2 × 19 × 4 2 × 19 × 4 19
5. Ans. (A) Verify each alternative
x -3 y -0 z-0
in (A) = = ; dr’s of the plane are 2, –1, – 5
4 -2 -10

2 -1 - 5
hence = =
4 -2 -10
r
6. Ans. (D) a = -iˆ + 2ˆj + 0kˆ
r
b = -iˆ + 0ˆj + 3kˆ
r ˆ ˆ ˆ
c = i + j + 4k

ˆi ˆj kˆ
r r r
n = a ´ b = -1 2 0 = ˆi(6) - j(-3) + k(0
ˆ + 2)
-1 0 3

r r
= 6iˆ + 3jˆ + 2kˆ a´b = 7

r r r
c × (a ´ b) (iˆ + ˆj + 4k)
ˆ × (6iˆ + 3jˆ + 2k)
ˆ 6 + 3 + 8 17
r
d = | Projection of c on nr | = ar ´ br = = = ]
7 7 7
MATHEMATICS /Q # 16 E-176
TM
TARGET: JEE (MAIN + ADVANCED) 2020
Path to success KOTA (RAJASTHAN)
ENTHUSIAST & LEADER
COURSE
7.Ans. (A) Centriod of tetra hedron having vertices (x1,y1,z1) , (x2, y2, z2), (x3, y3, z3) and (x4, y4, z4) is

æ x1 + x 2 + x 3 + x 4 y1 + y 2 + y3 + y 4 z1 + z 2 + z 3 + z 4 ö a + 1+ 2 + 0
G ºç , , ÷ so =1 Þ a =1
è 4 4 4 ø 4

2 + b +1+ 0 3+ 2+c+ 0
4
=2 Þ b=5 Þ 4
=3 Þ c=7

so a2 + b2 + c2 =1 + 25 + 49 = 75

x - 3 y -1 y - 2
8. Ans. (B) L1 = x - 1 = y - 2 = z - 3 Þ L 2 = = =
3 1 2 1 2 3
and point on line (1) can be given by P º (3l + 1, l + 2, 2l + 3)
any point on line (2) can be given by Q º (u + 3, 2u + 1,3u + 2) , so point of intersection of L1 & L2 is R

R º ( 4,3,5 ) Equation of plane passing through point R is a (x – 4) + b(y – 3) + c (z – 5) = 0


Plane is at greatest distance from O if OR is perpendicular to plane, then normal of plane is parallel to OP, Therefore
direction ratio of the normal to the plane are proportional to the direction ratios of OP
a b c
= =l so equation of plane
4-0 3-0 5-0

4l ( x - 4 ) + 3l ( y - 3 ) + 5l ( z - 5 ) = 0 , So answer is 4x + 3y + 5z = 50

9.Ans. (D) Þ AN.BC = 0 Þ 2x – 2 + 8y – 2z + 8 = 0

x + 4y – z + 3 = 0 .......(1)
l BC = BN

l(2iˆ + 8jˆ - 2k)


ˆ = xiˆ + (y + 11)ˆj + (z - 3)kˆ Þ x = 2l, y + 11 = 8l, z – 3 = – 2l

2l + 4 (–11 + 8l) – (3 – 2l) + 3 = 0 Þ 2l – 44 + 32l – 3 + 2l + 3 = 0

11 22 11 5 22iˆ - 11jˆ + 5kˆ


36l = 44 Þ l= Þ x= ,y = - ,z = Foot is
9 9 9 9 9
10.Ans. (B) The equation of the plane passing through the point of intersection of the given palnes is
(2x + 3y + z – 1) + k (x + 5y – 2z + 7) =0 Þ x(2 + k) + y (3 + 5k) + z(1 – 2k) + (7k – 1) = 0
This plane is parallel to x-axis
Therefore 1(2 + k) + 0 (3 + 5k) + 0 (1 – 2k) = 0, k = – 2
Hence the required equation is – 7y + 5z – 15 = 0 Þ 7y – 5z + 15 = 0.

11. Ans. (A) We have x.b + y.a = 0


Now (x + b) 2 + (y + a) 2
= 2[| x |2 + | a |2]
= 2[1 + 4] = 10

12. Ans. (B) aˆ ´ bˆ = cˆ Þ a.b


ˆ ˆ = 0 Þ a, ˆ cˆ are mutually perpendicular vectors.
ˆ b,
é xr bˆ cˆ ù = 3 Þ xr × bˆ ´ cˆ = 3 Þ xr × aˆ = 3 Þ [ xr cˆ aˆ ] = 4 Þ xr × bˆ = 4 Þ é xr aˆ bˆ ù = 2 Þ xr × cˆ = 2
( )
ë û ë û
Second Method
rˆ ù
Let xr = Aar + Bbˆ + Ccˆ taking dot with bˆ ´ c,
ˆ iˆ ´ aˆ and aˆ ´ bˆ , ë
ˆ = A éaˆ bˆ cˆ ù 3 = A×1 Þ A = 3
é xbc
û ë û
Similerly B = 4; C = 2

MATHEMATICS /Q # 16 E-177
TM
TARGET: JEE (MAIN + ADVANCED) 2020
Path to success KOTA (RAJASTHAN)
ENTHUSIAST & LEADER
COURSE
13. Ans. (B) Lines intersect at (2,0,2)
equation of the plane is

x-2 y z-2
-1 1 0 =0
1 -1 2

Þ x + y - 2 = 0 Ans.]

a +0+0
14. Ans. (B) Q Þ a =3
3

0+ b+ 0
= k Þ b = 3k
3
k 2 x + ky + z = 3k 2

x y z 1
3 0 0 1
=0
so equation of plane passing through point A(3,0,0), B(0,3k, 0) and C(0,0, 3k2) is Þ 0 3k 0 1
0 0 3k 2 1

x y z
or + + = 1 Þ k 2 x + ky + z = 3k 2 Þ k2x + ky + z = 3k2
3 3k 3k 2
15.Ans. (B) Let the foot of perpendicular be (4l, 3l – 3, 5l – 10), so direction ratios of perpendicular line are
(4l – 7, 3l – 4, 5l – 12)

(7, 1, 2)

(4l, 3l-3, 5l-10)

Also 4(4l – 7) + 3 (3l – 4) + 5(5l – 12) = 0


Þ l = 2, then foot of perpendicular is (8, 3, 0), so position vector of image is 9 ˆi + 5jˆ - 2kˆ
Paragraph for question No. 16 to 18
Sol. [16. A, 17. B, 18. C]

x -1 y z +1
We have = = = t ( say )
2 1 -2
uuur
Now AP = 2tiˆ + ( t - 1) ˆj - 2 ( t + 1) kˆ

uuur r -1
As AP.v = 0 Þ t =
3

2 -1
Again a1 + 1 = Þ a1 =
3 3 0 ,

-2 -5
a2 +1 = Þ a2 =
3 3

MATHEMATICS /Q # 16 E-178
TM
TARGET: JEE (MAIN + ADVANCED) 2020
Path to success KOTA (RAJASTHAN)
ENTHUSIAST & LEADER
COURSE
-2 -5
a3 + 1 = Þ a3 =
3 3

æ -1 -5 -5 ö
Hence Q is ç , , ÷ ;
è 3 3 3 ø

1 25 25 17
Hence OQ = + + = Ans. (iii)
9 9 9 3
uuuur uuur r
Equation of the plane containing the point A and L is given by éë PA, RA V ùû = 0

x -1 y -1 z -1
Þ 0 1 2 =0 Þ ( x - 1)( x - 2 ) + 2 ( 2 ( y - 1) - ( z - 1) ) = 0 Þ - 4 ( x - 1) + 4 ( y - 1) - 2 ( z - 1) = 0
2 1 -2

Þ 2 ( x - 1) - 2 ( y - 1) + ( z - 1) = 0 Þ 2x - 2y + z = 1 ..... (1)

1 1 4 16 16
Distance of origin from (1) is = Ans. (i) Finally AP = + + = 4 = 2 Ans. (ii) ]
9 3 9 9 9
Paragraph for question Nos. 19 to 21
z + 4i
19. (B) Let z = x + iy =1
z - 1 - 2i

Þ x 2 + ( y + 4 ) = ( x - 1) + ( y - 2 ) Þ 8y + 16 = -2x - 4y + 5 Þ 2x + 12y + 11 = 0
2 2 2

20. (D) AB = 1 + 6i , Representation of B¢ relative to A is


(1 + 6i)i = – 6 + i Þ Representation of B¢ relative to O is -6 + i - 4i = -6 - 3i

B¢ is – 6 – 3i Area of DOA ' B =


( 3 ´ 4 + 24 )
21. (C) A¢ is 4 – 4i, = 18
2
MULTIPLE CORRECT CHOICE TYPE
22. Ans. (A, C, D) [Hint: Note that D is 90°
hence ÐA = 90
D ABC and D DEF are both right isosceles triangle.
Now verify all alternatives ]
23. Ans. (A, B, C, D) simple
x - 2 y + 3 z +1
24.Ans. (A, B) Equation of the line AB through A is Þ = = Now,, 6 2 + 2 2 + 32 = 7
6 2 3
\ Equation of line AB is

x - 2 y + 3 z +1
= = =r
6/7 2/ 7 3/ 7
where, r is the distance of any point P on the line from the point A
But AP = ± 14 = r

6 2 3
\ x = 2 ± .14, -3 ± .14, -1 ± .14 (14, 1, 5) or (–10, –7, –7)
7 7 7

25.Ans. (A,B) Since both the given lines pass through the point with position vector ˆi + 2ˆj - k,
ˆ the required plane also passes

through ˆi + 2ˆj - kˆ and is parallel to the vectors ˆi + 2ˆj - kˆ and ˆi + ˆj + 3kˆ

MATHEMATICS /Q # 16 E-179
TM
TARGET: JEE (MAIN + ADVANCED) 2020
Path to success KOTA (RAJASTHAN)
ENTHUSIAST & LEADER
COURSE
r
If d = aiˆ + bjˆ - ckˆ is normal to the required plane, then a + 2b – c = 0 and a + b + 3c = 0.

a b c r
Þ = = Þ d = 7iˆ - 4jˆ - kˆ
7 -4 -1

So the required plane through ˆi + 2ˆj - kˆ and is normal to 7iˆ - 4ˆj - kˆ

ˆ ù × 7iˆ - 4jˆ - kˆ = 0 Þ rr × (7iˆ - 4ˆj - k)


Hence the required equation is éë r̂ - (iˆ + 2jˆ - k) ( )
ˆ = 1 ´ 7 + 2( -4) + ( -1)(-1) = 0
û

Also, since the required plane passes through ˆi + 2ˆj - k,


ˆ i.e. the point (1,2,–1) and the direction ratios of the plane are 7,

– 4, – 1, the equation of the plane in cartesian from can be written as 7(x – 1) – 4(y – 2) – (z + 1) = 0
INTEGER ANSWER TYPE
r r
26.Ans. [1125] Let V1 = 3iˆ + 4ˆj + 10kˆ and V2 = f (x)iˆ + g(x)ˆj + h(x)kˆ
r r r r r r
U(x) = V1 × V2 = V1 V2 cos q £ V1 V2 = 9 + 16 + 100

f 2 + g 2 + h 2 £ 3 125 = 1125 Þ N = 1125 = Ans.]


27.Ans. [0004] We have
uuur 2 uuur 2 uuur 2 uuur 2 uuur 2 uuur 2 uuur 2 r r r r
( b - ar ) + ( cr - b ) + ( d - cr ) + ( ar - d )
2 2 2 2
AB + BC + CD + DA - AC - BD = l PQ

r r 2 r r uuur
( ) ( )
2 2
- (c - a ) - d - b = l PQ
r rr rr rr
on simplifying gives = ar 2 + b 2 + cr 2 - 2a.b - 2b.c + 2a.c
r
(ar + cr - b )
2
..... (1)

r
r ar + cr r b
again P = ;Q = ;
2 2
r r r r
uuur ar + cr - b uuur ( a + c - b)
2
r r r uuur
( ) ( ) ( )
2 2 2
PQ = Þ 4 PQ = a+c-b Þ PQ = ..... (2)
2 4
from (1) and (2)
l = 4.
MATCH THE COLUMN :
28. Ans. [A–s ; B–r ; C–q]
1 r r r r r r
a ´b + b´c + c´a
Area of DABC 2
(A) Area of DAOC = 1 r r
a´c
2
r r r
now a + 2b + 3c = 0
r r r r r r r r r
cross with b, a ´ b + 3c ´ b = 0 Þ a ´ b = 3(b ´ c)
r r r r r r r 3 r r
cross with a, 2a ´ b + 3a ´ c = 0 Þ a ´ b = (c ´ a)
2
r r 3 r r r r
\ a ´ b = (c ´ a) = 3(b ´ c)
2
r r r
Let (c ´ a) = p

MATHEMATICS /Q # 16 E-180
TM
TARGET: JEE (MAIN + ADVANCED) 2020
Path to success KOTA (RAJASTHAN)
ENTHUSIAST & LEADER
COURSE
r r
r r r r r r r r 3p p r
+ +p r
r r 3p r r p a ´b + b´c + c´a 3p
a´b = b´ c = \ ratio = r r 2 2 = pr = 3
2
;
2 c´a
= r
p

Ans. Þ (S)
r r r
r r r r r r r r r r r r a+b+c r ( )
(B) L.H.S = d - a + d - b + h - c + 3(g - h) = 2d - a + b + c + 3 (
3
- 2h )
r r r r uuur
( )
= 2d - 2h = 2 d - h = 2HD Þ l = 2 Ans. Þ (R)

r r r r
r r r r b ×a b ×d
(C) ( )(
b´c × a´d = r r r r
c ×a c×d
)
|||ly compute others which gives (1) Þ (Q)]

29. Ans.[A–p; B–s ; C–p; D–p ]


r r rr
(A) a + b = 1 Þ a 2 + b 2 + 2a.b = 1

r r r rr
r r 1
2a + b .a (
2 + a.b ) 2 - (1/ 2 ) 3
a.b = - Þ cos q = r r r = r r = = Þ q = 30°
2 a 2a + b 4a + 4a. b + b
2 2
4 - 2 +1 2

(B) a cos A = b cos B Þ 2R sin A cos A = 2R sin B cos B


sin 2A = sin 2B Þ A = B Þ 2A = p – 2B Þ A + B = p/2 Þ 90° A
(C) Using consine rule

4 + x2 -1 x2 + 3 1 æ 3ö
cos A = where AB = x = = çx + ÷
4x 4x 4è xø
x 2

1 éæ ù 2

= êç x - ÷ + 2 3 ú
4 êçè x ÷ø ú B 1 C
ë û

3 3
hence minimum value of cos A = i.e. cos A ³
2 2
\ Amax = 30°
(D) BD = x cot 75°
and DC = x cot q
add

2x = x (cot 75º + cot q)

Þ (
cot q = 2 - 2 - 3 = 3 Þ q = 30°] )

MATHEMATICS /Q # 16 E-181
TM TM TARGET:
TARGET:JEE
JEE(Main
(MAIN+ +Advanced)
ADVANCED)2020
2020
Path to success KOTA (RAJASTHAN ) ENTHUSIAST
ENTHUSIAST&&LEADER
LEADER
Path to success KOTA (RAJASTHAN) COURSE
COURSE

ADVANCED QUIZ # 17(SOLUTION) TRI, TE & SOLUTION OF TRIANGLE M AT H S


SINGLE CORRECT CHOICE TYPE

sin(3r q) sin q sin 3q sin 9q sin(3n q)


1.Ans. (A) We have Q = å cos(3r +1 q) = cos 3q + cos9q + cos 27 q +..........+ cos(3n +1 q)

sin q 2 sin q cos q sin 2q 1 é sin(3q - q) ù 1


As, = = = = (tan 3q - tan q)
cos 3q 2 cos q cos 3q 2 cos q cos 3q 2 êë cos q cos 3q úû 2
1
\ Q = [(tan 3q - tan q) + (tan 9q - tan 3q) + ......... +
2
P
(tan 3n +1 q - tan 3n q)] Þ Q= ÞP = 2Q
2

p
2.Ans. (D) Since 1 degree = radians, converting the given equation into radians given
180

æ xp ö xp
sin x = sin ç ÷Þ x+ =p
è 180 ø 180

p 180 p
x= =
p 180 + p
1+
180

æ ö æ 3 ö 11 3 11
3.Ans. (D) Clearly, 3 £ x £ 3 3 , so x - + - x = ç x - ÷ + ç - x ÷ = 4
2 2 è 2 ø è 2 ø

4.Ans. (C) We have Hence f (q)]max = 1 and f (q)]min. = -3


\ -3 £ f (q) £ 1

pæ 2 pö 2p 5 -1
tan 2 ç1 + cos ÷ 2 + 1 + cos 3+
5è 5ø 5 = 3 + cos 72 ° 3 + sin18 ° 4 = 11 + 5
5.Ans. (A) Let E = = = =
p p 2p 1 - cos 72 ° 1 - sin18 ° 5 -1 5 - 5
tan 2 .sin 2 1 - cos 1-
5 5 5 4

12p 14p 4p p 13p p 4p p pæ 13p 4p ö


6.Ans. (A) cos + cos + 2 cos cos = 2 cos cos + 2 cos cos = 2cos ç cos + cos ÷ = 0
17 17 17 17 17 17 17 17 17 è 17 17 ø

æ p ö æ 3p ö
7.Ans. (B) Note : sin q ¹ cos q Þ qÏ ç 0, 2 ÷ È ç p, 2 ÷ ;
è ø è ø

p 3p æp ö
q¹ , , 0, p, 2p and equality holds if qÎç , p ÷
2 2 è2 ø

8.Ans. (D) The given equation can be written as cosx (cosx – 2) + 2sinx(cosx – 2) = 0
x
2 tan
1 2 = -1
or (cosx + 2 sinx) (cosx – 2) = 0 Þ cosx + 2sinx = 0 Þ tan x = tan x = - or
2 x 2
1 - tan 2
2

MATHEMATICS /AQ # 17 E-182


TM
TARGET: JEE (MAIN + ADVANCED) 2020
Path to success KOTA (RAJASTHAN )
ENTHUSIAST & LEADER
COURSE
2t -1 x
Þ = Þ t 2 - 4t - 1 = 0 Þ tan = 2 ± 5
1- t 2
2 2

9.Ans. (A) Let A = ( 3sin x - 4 cos x - 10 )( 3sin x + 4cos x -10 ) = 9sin 2 x - 16cos2 x - 10 ( 3sin x - 4 cos x ) - 10

( 3sin x + 4cos x ) + 100 = 25sin 2 x - 60sin x + 84 = ( 5sin x - 6 ) + 48


2

\ Amin occurs when six = 1 Þ Amin = 49

10.Ans.(B) cos A = -3 Þ sin A = 4 & tan A = - 4


5 5 3

4 4 -8 -16
\ sin A + tan A = - = & sin A.tan A = Þ Required quadratic equation is
5 3 15 15

æ -8 ö æ -16 ö
x2 - ç ÷ x + ç ÷ = 0 or 15x + 8x – 16 = 0
2

è 15 ø è 15 ø

sin130° cos80°
1 + cos80°
11.Ans.(B) E = æç cos 50° + cos80° ö÷ + cos80° = sin 50°.sin80° + sin 80° = = cot 40° = tan 50°
sin 80°
è sin 50° sin 80° ø sin 80°
12.Ans.(D) The given equation can be written as
sin x cos x [sin2x + sinx cosx + cos2x] = 1

-2 ± 4 + 16
or sinx cosx[1 + sinx cosx] =1 or sin2x[2 + sin2x] = 4 Þ sin 2x = = -1 ± 5
2
which is not possible.

( cosec x - 1)
2
2
( cot x )4 p
13.Ans.(C) f ( x ) = Þ f (x) = = 0 Þ cot x = 0 Þ x = ( 2n - 1)
( cosec x - 1) + 1 - cot x + cot x
2 1 + cot x
2
2

p 3p 5p 199p p p 100
\x= , , ......., Sum = [1 + 3 + 5 + ....... + 199] = . .200 = 5000p
2 2 2 2 2 2 2

7q 3q 7q q 7q q
14.Ans.(D) 2 cos cos - 2 cos cos = – 4cos éêsin .sin qùú Þ l = – 4.
2 2 2 2 2 ë 2 û

15. Ans.(B) tan A. tan B. tan C = tan A + tan B + tan C Þ A + B + C = p


Hence 3x + 5x + 10x = p Þ x = 10°

cos x + sin x. tan 55° 1 + tan x. tan 55° = 1 1


\ = = =1
cos x.tan 55° - sin x tan 55° - tan x tan ( 55° - x ) tan 45°

æ a+bö æ a -b ö æ a -b a+b ö æ a +b ö
16.Ans.(B) x - y = 2 cos ç 2 ÷ cos ç 2 ÷ - 2. ç cos 2 - cos 2 ÷ cos ç 2 ÷ - cos(a + b)
è ø è ø è ø è ø

æ a +b ö é æ a -b ö a -b a +bù 2 æ a+bö
x - y = 2cos ç ÷ Þ ê cos ç 2 ÷ - cos 2 + cos 2 ú - cos(a + b) Þ x - y = 2 cos ç ÷ - cos ( a + b )
è 2 ø ë è ø û è 2 ø
= 1 + cos(a + b) - cos(a + b) = 1

1
17.Ans.(D) (3cosx – 1)2 + (secy – 2)2 = 0 Þ cosx = Þ x = x1, x2 and secy = 2 Þ y = y1, y2 Þ four ordered pair
3

MATHEMATICS /AQ # 17 E-183


TM
TARGET: JEE (MAIN + ADVANCED) 2020
Path to success KOTA (RAJASTHAN )
ENTHUSIAST & LEADER
COURSE
3 3
18.Ans.(A) 1 + 4cos3x – 3cosx = 2(2cos2x –1) Let cosx = t, 1 + 4t3–3t = 4t2 – 2 Þ (t–1) (t + )(t - )=0
2 2

3 3 p 5p
cosx = 1 or or - Þ x = 2np or 2np ± 6 or 2np ± 6
2 2
A is not the subset of general solution of the given equation

1 s + ( s - a ) + (s - b ) + (s - c)
2 2
4s 2 - 2s ( a + b + c ) + a 2 + b 2 + c 2 a 2 + b 2 + c 2
2 2
1 1 1
19.Ans. (B) 2 + 2 + 2 + 2 = = =
r r1 r2 r3 D 2
D2 D2
20.Ans. (D) a2,b2,c2 are in A..P. Þ 2b2 = a2 + c2 i.e. , a2 + c2 – b2 = b2

b2 a 2 + c2 sin 3B
2 ac cos B = b2 Þ cos B = = , Now = 3 - 4sin 2 B
2ac 4ac sin B

4(a 2 + c 2 )2 (a 2 - c 2 ) 2
= 4 cos2 B – 1 = 2 2 –1 =
16a c 4a 2 c 2

sin 45° sin C A


21.Ans. (C) Using sine law, =
6 3

3 3 6
Þ sinC = Þ C = 60° Þ A = 75°
2 45°
B C
b2 + c 2 - a 2 1 (b + c) 2 - 2bc - 7 A
22.Ans. (C) Using cosine rule, cos60° = Þ = Þ bc = 6 and b + c = 5
2bc 2 2bc
Þ b = 3, c = 2, a = 7 c 60° b

3
ratio = B a C
2
1 AD
23.Ans. (A) tan 22 = .......... (i)
2 BC + x

1 AD æ 1 ö AD
tan 67 = Þ tan ç 90 - 22 ÷ =
2 x è 2ø x

1 AD
= .......... (ii)
2 -1 x

1
Form (i) 2 -1 = ........... (iii)
BC x
+
AD AD

BC 1 1 BC BC
+ = Þ = 2 + 1 - 2 + 1 Þ AD =
AD 2 +1 2 -1 AD 2

b2 + c 2 - a 2 a 2 + c 2 - b 2 a 2 + b2 - c 2 a 2 + b 2 + c2 (a + b + c)2 - 2(ab + bc + ca)


24. Ans. (A) cos A + cos B + cos C = + + = =
a b c 2abc 2abc 2abc 2abc 2abc

162 - 2 ´ 34 47
= =
2 ´ 50 25

MATHEMATICS /AQ # 17 E-184


TM
TARGET: JEE (MAIN + ADVANCED) 2020
Path to success KOTA (RAJASTHAN )
ENTHUSIAST & LEADER
COURSE
Paragraph for question Nos. 25 to17

b Þ b + c - a + 2 æç c + a - b + a + b - c ö÷ = a + b
2 2 2 2 2 2 2 2 2 2 2
cos A æ cos B cos C ö a
25 (C) + 2ç + ÷ = + 2abc 2abc 2abc
a è b c ø bc ac è ø abc

p
Þ 3a 2 + b 2 + c 2 = 2 ( a 2 + b 2 ) Þ a2 + c2 = b2 Þ ÐB = 2

1
ac ac ( a + c - b )
Area of DABC 2 =
26. (B) Inradius r = =
(a + c)
2
s 1 - b2
(a + b + c)
2

ac ( a + c - b ) a +c-b
= = (\ a2 + c2 = b2)
2ac 2

p p p
27. (B) Now A, C, B are in A.P. Þ 2C = A + B = p – C Þ C = ; A = , B =
3 6 2

p 2p 3 3
Hence å sin 2A = sin 3 + sin p + sin 3
=
2
+ 0+
2

MULTIPLE CORRECT CHOICE TYPE

æ 2 ö 1
28.Ans. (AD) We have ç cos x + cos 2 x ÷ ³ 2 , 1 + tan22y ³ 1, 2 £ 3 + sin 3z £ 4
è ø

1
So, the only possibility is cos2x + = 2 , 1 + tan22y = 1, 3 + sin 3z = 2 Þ cosx = ±1 Þ x = np, n Î I
cos 2 x
mp p
Also, tan 2y = 0 Þ y = , m Î I and sin 3z = –1 Þ z = (4k –1) , k Î I
2 2
INTEGER ANSWER TYPE

æ 4 + log t ö
29.Ans.[0003] Let cos q = t , so we get ( log 2 t )2 + log 4 / t (16t) = 2 Þ ( log 2 t )2 + log 2 (16t) = 2 Þ ( log 2 t ) + ç 2 - log t ÷ = 2
2 2

æ4ö è 2 ø
log 2 ç ÷
ètø

æ 4+ z ö 1
Let log 2 t = z , so z + ç ÷ = 2 Þ z(z2 – 2z – 3) = 0 Þ z = 0, –1, 3 Þ t = 1, ,8
2

è 2-z ø 2

1 p æ p 5p ö
So, cos q = 1 or cos q = Þ q = 2np ± or q= 2mp, n, m Î I. Hence, number of solutions are 3 ç i.e, q = , , 0 ÷
2 3 è 3 3 ø

30.Ans.[4] As 3x2 – 8x + 7 > 0 " x Î R and log2(x – 2) is defined provided x > 2


\ Taking intersection, we get x Î (2, ¥) Now using properties of log functions, we get

1 3x 2 - 8x + 7 3x 2 - 8x + 7
log 2 (3x 2 - 8x + 7) - log 2 (x - 2) ³ 1 Þ log 2 ³ 2 Þ ³ 4 Þ 3x2 – 8x + 7 ³ 4x2 – 16x + 16
2 (x - 2) 2 (x - 2) 2

Þ x2 – 8x + 9 £ 0 Þ 4 - 7 £ x £ 4 + 7 Þ 1.36 £ x £ 6.64
\ Possible integral values of x are 3, 4, 5, 6 (As x > 2)
Hence number of integral values of x is 4

MATHEMATICS /AQ # 17 E-185


TM
TARGET: JEE (MAIN + ADVANCED) 2020
Path to success KOTA (RAJASTHAN )
ENTHUSIAST & LEADER
COURSE
1
31.Ans.[0000] As 2cot a + = 2 cot a + cos ec 2 a = 2 cot a + 1 + cot 2 a = (1 + cot a) 2 =|1 + cot a |
sin 2 a

æ 3p ö
Also, a Î ç , p ÷ , so |1+ cot a| = –(1 + cot a)
è 4 ø

\ we get tan b = -1 Þ no solution exist.

32.Ans.[0002] We have 5(sinx + 2 sin 2x sin x + cos 3x + sin 3x) = (1 + 2sin 2x) (3 + cos 2x)
Þ 5(sin x + cos x – cos 3x + cos 3x + sin 3x) = (1 + 2 sin 2x) (3 + cos 2x)
Þ 5 cos x (1 + 2 sin 2x) = (1 + 2 sin 2x) (3 + cos 2x)

-1
\ Either (1 + 2 sin 2x) = 0 Þ sin 2x = (Not possible, think !) or 5 cos x = 3 + cos 2x = 3 + 2cos 2x – 1
2
Þ 2 cos2x – 5 cos x + 2 = 0 Þ 2 cos2x – 4 cos x – cos x + 2 = 0 Þ 2 cos x (cos x – 2) – (cos x – 2) = 0

1
Þ cos x = 2 or cos x = (But cos x ¹ 2)
2

p 5p
\ x= or . As x Î [0, 2p] so we have only 2 solutions
3 3

2s(s - a) b(s - b) c(s - c) a(s - a)


33.Ans. [0020] = + - Þ 2s2 – 2as = bs – b2 + cs – c2 – as + a2
D D D D
2s2 = (a + b + c)s – b2 – c2 + a2

Þ a 2 = b 2 + c2 Þ DABC is rt. angled at A Þ A is orthocentre

1
\ Distance between centroid and circumcentre = ´ 60 = 20
3

b 2 + c2 - a 2
34.Ans. [1225] cos A = A
2bc
c b

B a/2 D a/2 C

1 p b 2 + c2 - a 2
Þ = cos = Þ b 2 + c 2 - a 2 = bc .......................................... (1)
2 3 2bc

In triangle ABD, AD = c +
2 2 a2 a
- 2c ´ cos B. Þ 4 AD 2 = 4c 2 + a 2 - 4 ca
c2 + a 2 - b 2( )
= 2c2 + 2b2 - a 2
4 2 2ca

= 2c 2 + 2b 2 - ( b 2 + c 2 - bc ) [(using (1)] = b 2 + c 2 + bc. = 2500 + 900 + 1500 = 4900 Þ AD2 = 1225

MATHEMATICS /AQ # 17 E-186


TM
TARGET: JEE (MAIN + ADVANCED) 2020
Path to success KOTA (RAJASTHAN )
ENTHUSIAST & LEADER
COURSE
MATCH THE COLUMN :

35. Ans.(A – r; B – p; c – q; D – s)

sin 2 x - cos x = 1/ 4 Þ 1 - cos x - cos x = 1/ 4 Þ 4 cos x + 4 cos x - 3 = 0 Þ ( 2cos x + 3)( 2 cos x - 1) = 0


2 2
(A)

Þ cos x = 1/ 2 [cos x ¹ -3/ 2] Þ x = p / 3, 5p / 3 as 0 £ x £ 2p so number of solution 2.

æp ö æx ö
(B) sin 2x = cos 3x Þ cos ç - 2x ÷ = cos 3x Þ 3x = 2np ± ç - 2x ÷ Þ x = 2n p – p/2 or 5x = 2np + p/2
è2 ø è2 ø

p 5p 9p 13p 17 p p 3p p
Þ x= , , , , , , ( is coming two times), Total number of solutions is 6.
10 10 10 10 10 2 2 2

tan 2 x + cot 2 x = 2 Þ tan x - 2 tan x + 1 = 0 Þ tan x = 1 Þ tan x = ± 1


4 2 2
(C)

Þ x = p / 4, 3p / 4, 5p / 4, 7p / 4 and the number of solutions is 4

(D) sin x = 1/ 2, cos x = 3 / 2 Þ x = p / 6 only one solution

36.Ans. (A-p; B-q, s; C-p, q.)


c(a + b) sinA+ sinB
(A) sinA, sin B are roots of c2x2 – c (a + b)x + ab = 0 \ sinA + sinB = = sin A + sin B Þ = sinA+sinB
c2 sinC
Hence sin C = 1 Þ C = 90º From sine formula,

sin A sin B sin C 1


= = = 2R = Þ sin C = 1 Þ C = 90° [using equation (1)]
a b c c

40 2 + (40 3) 2 - a 2
(B) Using cosine formula, cos 30° = Þ a = 40
2 ´ 40 ´ 4 3
\ AB = BC = 40 \ DABC is isosceles
Also, ÐA = ÐC = 30° Þ ÐB = 120°
\ DABC is an obtuse angled D.
(C) In DABC,

1 - cos A cos B
cos A cos B + sin A sin B sin C = 1 Þ sin C = ....(i)
sin A sin B

Since 0 < sin C £ 1 [Q ÐC is angle of the triangle]

1 - cos A cos B
Þ 0< £ 1 Þ 1 – cosA cosB £ sin A sin B Þ 1 £ sin A sin B + cos A cos B
sin A sin B

Þ 1 £ cos(A - B)or cos(A - B) ³ 1 Þ A – B = 0 Þ A = B ....(ii)

1 - cos 2 A
Also from (1) and (2), sin C =
sin 2 A
sin C = 1 Þ C = 90° ....(iii)
From equations (2) and (3), DABC is a right angled isosceles triangle.

MATHEMATICS /AQ # 17 E-187


TM
TARGET: JEE (MAIN + ADVANCED) 2020
Path to success KOTA (RAJASTHAN )
ENTHUSIAST & LEADER
COURSE
IMPORTANT NOTES

MATHEMATICS /AQ # 17 E-188


TM
TARGET: JEE (MAIN + ADVANCED) 2020
Path to success KOTA (RAJASTHAN )
ENTHUSIAST & LEADER
COURSE
IMPORTANT NOTES

MATHEMATICS /AQ # 17 E-189


TM
TARGET: JEE (MAIN + ADVANCED) 2020
Path to success KOTA (RAJASTHAN )
ENTHUSIAST & LEADER
COURSE
IMPORTANT NOTES

MATHEMATICS /AQ # 17 E-190

You might also like